You are on page 1of 339

OLIMPIADAS

INTERNACIONALES
DE QUIMICA
OLECCtON DE EJERCICIOS DE COMPETENCIA
JURADO INTERNACIONAL DE LAS OLIMPIADAS DE QUIMICA Y COMISION

CHECOESLOVACA PARA LA COOPERACION CON UNESCO

COLECCION DE EJERCICIOS DE COMPETENCIA DE LAS

OLIMPIADAS INTERNACIONALES DE QUIMICA

BRATISLAVA - PRAGA
PRIMERA EDICION:
Diez Primeras Olimpiadas:
Recopilación: Antón Sirota_y Jitka Machacková
Arbitros: Doc. Ing. Tibor Sramko, CSc. e Ing. Emil Adamkovic
Traducido y Editado en inglés por RNdr. Antón Sirota, CSc. Asesoría
lingüística: S. Bachraty
Publicado con el consentimiento del Ministerio de Educación de la
República Socialista Eslava (N°102 114/ 1980-20 de 26.6.1980) Impreso en
Bratislava por la Central House of Pioneers and Youth Junio, 1980.
Tiraje 250 copias.
Décimoprime ra a Decimoquinta Olimpiadas:
Recopilación: RNDr. Frantisek Zemánek, RNDr. Antón Sirpta, CSc;
Sr. Jaromír Krejci; doc. RNDr. Miloslav Cerny'.CSc.
doc. RNDr. Pavel Petrovic, CSc. Asesoría
lingüística: Ing. K. Stulik, CSc. M. Stuliková Asesoría técnica:
J. TrávniSek
Publicado con la autorización del Ministerio de Educación de la República
Socialista Checa (N014907/84-211 de 10. 4.1984) Impreso en Ostrava
por la Central House of Pioneers and Youth Junio, 1984. Tiraje 150
copias.

SEGUNDA EDICION:
Publicada según contrato 107.012.4 (16.7.1984) entre la Unesco y la
Comisión Checoeslovaca para la Cooperación con Unesco. Impresa en
Bratislava. Central House of Pioneers and Youth. Junio, 1985.
Tiraje 350 copias
(C) Copyright: Secretariado Ejecutivo. Jurado Internacional de las
Olimpiadas de Química.

NOTA: Los autores son responsables por la selección y presenta-


ción de los hechos contenidos en esta publicación, así como de
las opiniones expresadas en ella, las que no son,
necesariamente, las de la Unesco y no comprometen a la
Organización.
Traducción al español: Guido A. Concha, Ph.D.

Edición en español publicada por la Oficina Regional de Educación de la


Unesco para América Latina y el Caribe (OREALC). Casilla 3187.
Santiago de Chile, 1985
INDICE

PRESENTACION i
PREFACIO DE LA PRIMERA EDICION 1
PREFACIO DE LA SEGUNDA EDICION 7
ABREVIATURAS Y SIMBOLOS MATEMATICOS USADOS
EN LA COLECCION 11

EJERCICIOS DE COMPETENCIA:

I Olimpiada. Praga 13
II Olimpiada. Katowice
III Olimpiada. Budapest 23
IV Olimpiada. Moscú 37
V Olimpiada. Sof ia
VI Olimpiada. Bucarest 53
VII Olimpiada. Veszprem 73
VIII Olimpiada. Halle
IX Olimpiada. Bratislava 91
X Olimpiada. Torun 113
XI Olimpiada. Leningrado
XII Olimpiada. Linz 143
XIII Olimpiada. Burgas 165
XIV Olimpiada. Estocolmo
XV Olimpiada. Timisoara 201
241
269
311
345
387

APENDICE

Cantidades y sus Unidades usadas en la Colección 425


PRESENTACION

Entre las actividades científicas y tecnológicas juveniles, cabe


destacar el éxito que revisten las Olimpiadas. En diveros países de América
Latina existen a la fecha programas sistemáticos para su organización y
desarrollo, generalmente anual. Tal ocurre con las Olimpiadas Nacionales
de Matemáticas, que constituyen una experiencia que tiene ya tradición en
numerosos países de la región, algunos de los cuales han participado con brillo
en Olimpiadas Internacionales.

Tanto la experiencia alcanzada como los resultados obtenidos han


motivado que las Olimpiadas se extiendan al campo de interés de otras
disciplinas científicas. Es el caso de la Química que, como otras de gran
contenido experimental, ofrece un interesante desafío y originalidad en la
preparación de sus ejercicios y actividades.

Las Olimpiadas Internacionales de Química, que han estado en


constante expansión desde que se iniciaron en 1968, nos ofrecen un singular
repertorio de ejercicios lo que, sin duda, debe constituir una herramienta
valiosa para los promotores de este tipo de actividades y para los profesores
de química, en general, una fuente de inspiración para su trabajo cotidiano.

La Oficina Regional de Educación de la Unesco para América Latina


y el Caribe, OREALC, tomó a su cargo la traducción y edición en español de
la Colección de Ejercicios de Competencia de todas las Olimpiadas
Internacionales de Química, considerando el interés que, por esta disciplina
y por las actividades científicas y tecnológicas juveniles, existe en la re-
gión. Ahora, la ofrecemos al personal especializado y esperamos que sirva
de apoyo tanto a quienes preparan futuras olimpiadas de química como a aquellos
que ya las están desarrollando.

OREALC. Santiago de Chile, 1985


PREFACIO

La enseñanza colectiva tradicional en las escuelas que usan


currículos y libros de texto uniformes no crean un ambiente favorable para
los alumnos dotados y talentosos que desean a prender y adquirir conocimientos
y habilidades extras a las pre sentadas dentro del currlculo del colegio. Así,
sucede a menudo que estos estudiantes aprenden con este material con excelen
tes resultados,pero con el mínimo de esfuerzo, sin tener la opor tunidad de
desarrollarse completamente y usar sus habilidades y conocimientos.

La enseñanza en la escuela básica es, por lo tanto, su plementada


por un conjunto de actividades e-xtra-escolares especializadas,, Comparada
con la enseñanza escolar, las activida -des extra-escolares difieren de ella,
especialmente en la parti^ cipación voluntaria de los alumnos, como también
en su forma y contenidos, los cuales son más variables y ajustados a los inte
reses de los estudiantes.

Las actividades extra-escolares en el campo de la química, las


cuales han sido especialmente promovidas en los últimos años, tienen las
siguientes metas:

a) Identificar a los alumnos que son dotados e interesados en química,


dándoles las motivaciones correspondientes para actividades en su tiempo
libre, influenciando y canalizando sus intereses en la dirección social
deseada.

b) Desarrollar las habilidades intelectuales de los alumnos entrenando sus-


capacidades para pensar en términos químicos y dominar nuevos
descubrimientos de una manera activa, creativa e independiente.

c) Influenciar positivamente la modernización de la enseñanza de la química


en" las escuelas.

Los jóvenes muchas veces toman parte en diferentes com petencias


debido a que muchos de ellos lo hacen para poder mostrar sus conocimientos
y habilidades. Una de las formas de actividades extra-escolares en
Checoeslovaquia es, por lo tanto, la Olimpiada de Química. Es una competencia
en química organizada anualmente en diferentes etapas y niveles para alumnos
de escuelas básicas y secundarias. El participante de la Olimpiada de Química
tiene que solucionar un número dado de ejercicios de com petencia teóricos
y experimentales en un período de tiempo deter minado. La Olimpiada tiene un
carácter selectivo en todas las e tapas y rondas, i.o. solamente aquellos que
han resuelto con éxi_ to los ejercicios prosiguen a la ronda siguiente„

La Olimpiada Química tiene en Checoeslovaquia una tradi_ ción


permanente. La experiencia y los resultados obtenidos de es_ tas competencias
en Checoeslovaquia, pero también en otros paí -ses, muestran que debieran ser
consideradas como una parte impor tante de las actividades extra-escolares,
siendo de gran valor instructivo y educacional. La Olimpiada Química es para
iniciar e incrementar el interés de los alumnos por la química y ayudar
activamente a encontrar alumnos dotados y cuidar su desarrollo futuro. Permite
enseñar a los alumnos a trabajar en una forma crea tiva e independiente, a
desarrollar sus habilidades y razonamien to y también mostrarles como aplicar
sus conocimientos.

La Primera Olimpiada Internacional de Química se efec -tuó en Praga


en 1968; su realización fue iniciada por el Comité Central para Olimpiadas
Químicas de Checoeslovaquia. Solamente tres países participaron en
ella/Checoeslovaquia, Hungría y Polo nia/. El primer encuentro internacional
de los competidores a -lumnos y los organizadores de las Olimpiadas Químicas
nacionales formó las Bases para una Buena tradición, cuando cada año, alum-
nos de diferentes países se juntaron para competir y mostrar sus habilidades
y conocimientos, pero además de esto, ellos hacen nue vos- amigos y aprenden
acerca de la organización del país, su pueblo y costumbres.

Once Olimpiadas Químicas Internacionales /OQIs/ se han llevado


a efecto a la fecha; dos de ellas en Checoeslovaquia/1968 , 1977/,
Polonia/1969, 1978/, Hungría/1970, 1975/ y la Unión Sovié tica/1972, 1979/;
una en Bulgaria/1973/, Rumania/1974/, y la República Democrática
Alemana/1976/. El número de países participantes en las OQIs ha aumentado y
junto a los países arriba mencionados, también estudiantes de Austria,
Bélgica, la República Federal de Alemania, Finlandia, Suecia, Turquía y
Yugoslavia han participado en la competencia.

OQIs son organizadas anualmente después del término del año


académico, como norma en Julio, en uno de los países partici_ pantes. El
principal organizador es el Ministro de Educación del país anfitrión, pero
también las OQIs patrocinadas por Organizaciones Juveniles, Sociedades
Químicas y en forma limitada por la industria química. Todos los gastos de
estadía de los partici -pantes y del desarrollo de la competencia son cubiertos
por el país organizador.

Cada equipo nacional consiste en cuatro estudiantes quie nes en la


época de envío de sus solicitudes a la competencia,son estudiantes regulares
de una escuela secundaria sin ninguna espe cialización química. Los
participantes son en su mayoría pupi -los de los cursos más altos de la escuela
secundaria. Como norma ellos son ganadores en sus competencias nacionales.

Los estudiantes competidores son acompañados por dos maes_ tros o


pedagogos quienes formarán parte del Jurado Internacional durante la
competencia.
La competencia es dirigida por el Comité Organizador / del país
organizador/ y por el Jurado Internacional. El Jurado Internacional evalúa
los resultados de los trabajos de los estudiantes y proclama los resultados
finales.

La OIQs consta de dos partes: teórica y experimental.La competencia


dura dos días, pero considerando las excursiones y visitas hacen un total de
ocho días. La parte teórica de la com petencia debería ser de un mínimo de
cuatro horas con un máximo de cinco horas. Después de un descanso de un día,
prosigue la parte experimental la cual dura también cuatro horas.

Los ejercicios para la competencia están a cargo del Or ganizador.


Profesores y ayudantes de las universidades, así como maestros de las escuelas
secundarias preparan las tareas y sus soluciones. Para crear las mismas
condiciones para todos los es_ tudiantes en competencia, algunos meses antes
de la OIQ el Comité Organizador prepara facsímiles de ejercicios/ en uno de
los lenguajes oficiales/ de manera que los estudiantes en competen -cia puedan
prepararse en forma apropiada para la competencia. Es to es muy importante
debido a las diferencias del currículo de química en los diferentes países.

El país organizador está obligado a preparar ejercicios para la


competencia en cuatro versiones: Inglés, Francés, Alemán y Ruso. Los maestros
acompañantes traducen las tareas a su lengua materna. Los idiomas arTiba
mencionados pueden también ser usados en las sesiones del Jurado
Internacional.

Los ejercicios resueltos por los competidores son eva -luados y las
soluciones son fotocopiadas. Los originales son en tregados a los autores de
los ejercicios, mientras las copias son evaluadas por los profesores
acompañantes de los estudiantes. Am bas evaluaciones son luego comparadas.

Los estudiantes en competencia pueden ganar un máximo de cien


puntos, sesenta puntos para la parte teórica y cuarenta pun tos para la parte
experimental. De acuerdo al número de puntos obtenidos hay usualmente entre
tres y cinco primeros premios/diploma y medalla de oro/ y más segundos y
terceros premios/diploma y medalla de plata y bronce respectivamente/. Más
aún, los ganadores reciben algunos regalos.

Debido a que el numero de países participantes en la OIQs está


aumentando y las competencias se hacen más difíciles , se juzgó necesario
solucionar los problemas concernientes a la regulación de la competencia, sus
contenidos y la cooperación in ternacional. Estos problemas fueron discutidos
en el Encuentro Consultivo Internacional llevado a efecto en Stirin cerca de
Pra gra/Checoeslovaquia/ en Mayo de 1976. Todos los participantes de diferentes
países de Europa llegaron a la conclusión de que las OIQs son un factor
estimulante tanto para los alumnos dota dos como para los profesores de química
y reafirmaron su interés en llevar a efecto OIQs en el futuro. Junto con esto,
se discutieron las normas de la competencia y se aprobaron también las formas
concretas de cooperación internacional. El representan -te de UNESCO sugirió
algunas posibilidades de cooperación entre UNESCO y aquellos países en los
cuales en años futuros se organi_ cen OIQs.

Las conclusiones del arriba mencionado encuentro ya encontraron su


realización cuando la novena OIQs fue organizada en Bratislava
/Checoeslovaquia/. Fue la primera OIQ en la cual UNESCO ha participado. Un
seminario sobre las actividades extra escolares, el cual fue una parte de la
novena OIQ, también fue organizado bajo los auspicios de UNESCO. Fue muy
apreciado que UNESCO haya participado también en la décima OIQ, llevada a cabo
en 1978 /Torun, Polonia/. Creemos que la cooperación con UNESCO en este campo
puede ser muy útil, pero se requiere examinar sus varias formas y usarlas.
Esta Colección de Ejercicios puede ser vir como un ejemplo de tal cooperación.

Habiendo participado en el Simposio Internacional so -bre Educación


Química/, en nuestra exposición plenaria sobre actividades extra*-escolares
en química, promocionamos las OIQ. En la discusión que siguió se presentaron
preguntas respecto del ni_ vel y tipo de los ejercicios de competencia.
Recibimos pedidos de enviar estos ejercicios a muchos participantes del
simposio como también a los Consejeros para la Educación Química en diferen
tes países.

Para los países que quisieran tomar parte en el OIQ, el problema


es cómo obtener los trabajos de competencia de previas OIQs para ser capaces
de preparar adecuadamente a sus estudian -tes para esta competencia. Es un
hecho, que una colección de es tos trabajos no ha sido publicada hasta ahora.
Muchos ejerci cios fueron publicados separadamente en diferentes idiomas en
va rias magazines nacionales, pero eran difíciles de obtener. En es te punto
hemos sido solicitados por un representante de UNESCO para compilar los
trabajos de competencia de las anteriores OIQ.
Nuestra iniciativa ha sido muy bien recibida por todos los miembros del Jurado
Internacional de la décimo primera OIQ.

La Colección terminada contiene los ejercicios de compe tencia de


las primeras diez Olimpiadas Internacionales de Química. En la elaboración
de esta Colección tuvimos ciertas dificul tades porque nuestro propósito no
fue solamente hacer uso de reproducciones anteriores sino que darles una forma
tal que puedan usarse como ejercicios sistemáticos en la educación química.
Con secuentemente,fue necesario actualizarlas y hacerles algunos cam bios.
En particular, conciernen a las soluciones de los ejerci -cios las que a menudo
en las competencias están disponibles en forma abreviada sólo para el Jurado
Internacional. Nos disculpa mos y explicamos algunos cambios de métodos
inevitables en la formulación y solución de algunos ejercicios "más que nada
en los trabajos de las primeras competencias". Desafortunadamente, no son
conocidos todos los autores de los trabajos de las competencias y esa es la
razón porque los nombres de los autores de los trabajos individuales no
aparecen.

En esta colección se han usado las Unidades SI y se ha introducido


un método más moderno de cálculos químicos. Solamen te hay algunas excepciones
hechas en un esfuerzo para ceñirse al texto original; en este caso las
cantidades y las unidades usadas no son SI.

Un reestudio de los ejercicios de competencia de las diez primeras


OIQs debiera servir a competidores y maestros como una fuente de futuras ideas
en su preparación para esta difícil competencia. Para aquellos que han
participado en algunas de es tas Olimpiadas, la colección serviría de ayuda
como un archivo y material de documentación.

Esperamos que esta colección nos ayudará a todos en nue£ tro


esfuerzo para mover la frontera de la enseñanza de la quíirvi ca unos pocos
pasos hacia adelante para ayudar a satisfacer las necesidades de la humanidad
y su ambiente.
- 11 -

PREFACIO A LA SEGUNDA EDICION

Han transcurrido cinco años desde la publicación de la Colección


de Trabajos de Competencia de las diez primeras Olimpiadas Internacionales de
Química. En vista de que su publicación ha tenido una activa respuesta
internacional, nos hemos abocado a la segunda edición de la Colección. Esperamos
que al abarcar un lapso más breve de tiempo la publicación de los quince trabajos
de competencia de las OIQ y de sus soluciones los hará más pertinentes,
proporcionando así una visión más clara del nivel actual de esta competencia
internacional de tan ta importancia entre estudiantes de química talentosos.
Durante los últimos cinco años la Olimpiada Internacional de
Química ha progresado cuantitativa y cualitativamente. Muchos países, cuyos
estudiantes no habían participado anteriormente en las OIQ, se han incorporado
a la competencia. Por primera vez las OIQ se han organizado exitosamente en
Austria y Sue cia. Asimismo, el nivel de la competencia se ha venido elevando
en forma sostenida.
Los estudiantes que participan en las OIQ han mostrado un excelente
conocimiento de las áreas individuales de la química y son adecuadamente hábiles
en la solución de trabajos prácticos en el laboratorio.
La Olimpiada de Química ejerce una influencia positiva en las normas
generales de la enseñanza de la química en las escuelas de los países que
participan en ella. Los trabajos de competencia constituyen la labor de docentes
de gran experiencia y la preparación para la Olimpiada de Química ofrece a los
estudiantes grandes posibilidades para mejorar sus talentos.
En los últimos años, la Unesco ha tenido una participación
considerable en el desarrollo de las Olimpiadas Internacionales de Química.
La base de esta cooperación se estableció en el Primer Seminario Internacional
sobre Actividades Extraescola-res de Química para Estudiantes de Educación
Secundaria, que se realizó en Checoeslovaquia en 1976 por iniciativa de
organizadores de ese país. En la actualidad, la Unesco ha participado di-
rectamente en la preparación de tres Olimpiadas Internacionales de Química.
También, sobre la base de la cooperación con la Unesco, se publicó la primera
Colección de Trabajos de Competen cia de las Olimpiadas Internacionales de
Química y se dedicó un número especial de la revista "Educación Científica y
Técnica Extraescolar" (N°20-21) a esta importante competencia interna-
cional .
En 1982, con la cooperación de la Unesco, se organizó en
Checoeslovaquia el Segundo Seminario Internacional sobre Acti vidades
Extraescolares de Química para Estudiantes de Educación Secundaria con el
objeto de ampliar la cooperación y de resolver, eventualmente, algunos puntos
- 12 -

relacionados con las Olimpiadas Internacionales de Química. Los


participantes en el Seminario evaluaron los resultados altamente positivos
logrados a partir del Primer Seminario Internacional y apuntaron al futuro de-
sarrollo de las OIQ. Entre los aspectos positivos del Seminario se cuentan
sus Actas, en las que se recomendó que el Jurado Internacional de las Olimpiadas
de Química fuera la máxima entidad ejecutiva de esta competencia a partir de
la 14a. OIQ y también se sugirió que se nombrara su Secretaría Ejecutiva. El
Jurado Internacional aprobó esta propuesta en la 14a. OIQ que, como su entidad
de trabajo, inició sus actividades en 1982. Entre sus principales tareas
se encuentran la promoción de una permanente cooperación y coordinación de
actividades entre los países que participan en las OIQ, la puesta en práctica
de las decisiones del Jurado Internacional de las OIQ, la compilación,
publicación y distribución de documentos para las necesidades de los partici-
pantes en las OIQ. También esta Colección ha sido publicada de acuerdo con
las tareas arriba mencionadas.
Al recopilar esta Colección hemos respetado en el mayor grado
posible la composición original de los trabajos de competencia y sus soluciones.
Debido a que no conocíamos a todos los miembros de los grupos que han elaborado
los trabajos durante las respectivas OIQ no habríamos podido mencionarlos en
esta Colección. No obstante, permítasenos por favor expresarles núestros
profundos agradecimientos. Con leves excepciones hemos respetado las
cantidades y las unidades utilizadas por los autores y los textos han sido
modificados solamente cuando estas unidades no correspondían al Sistema SI.
Esperamos que esta Colección de los Trabajos de Competencia de las
OIQ sea algo más que sólo material de archivo y documentación, que será útil
para los alumnos que compiten y que servirá de ayuda en la preparación de una
olimpiada de química; confiamos en que proporcionará tópicos adecuados a los
expositores para la elaboración de los nuevos trabajos y que dará a todos los
partidarios de las OIQ una visión del nivel de los trabajos de competencia en
las Olimpiadas Internacionales de Química.
Debemos nuestros agradecimientos a todos aquellos que nos han
proporcionado los materiales necesarios y que nos han ayudado a publicar esta
Colección; al señor K. Stulík y a la señora Stulíková por su asesoría
lingüística y al señor J. Trávnicek por su ayuda técnica.

Praga-Bratislava, 1985

Pavel Petrovic
- 13 -

Secretario General Jurado Internacional


de las OIQs

Central House of Pioneers and Youth Mierové nam.


1
811 06 Bratislava, Czechoslovakia
ABREVIATURAS Y SIMBOLOS MATEMATICOS USADOS EN LA COLECCION

OIQs : Olimpiadas Internacionales de Química.

OIQ : Olimpiada Internacional de Química

STP : Temperatura estándar /T / y presión

M : molar

N : normal

igual a, se iguala

: corresponde a

*\j : aproximadamente igual a

"o : proporcional a

< : menor que

> : mayor que

: implica

Nota: El traductor ha respetado el marco general del original.


Aunque el sistema IUPAC requiere del uso de paréntesis en las fórmulas
escritas de Química, se han mantenido los signos / impresos en el texto
de las primeras Diez Olimpiadas Internacionales de Química.
PRIMERA

OLIMPIADA INTERNACIONAL DE QUIMICA

PRAGA 1968

CHECOESLOVAQUIA

Una mezcla de hidrógeno y cloro mantenida en un frasco cerrado


a temperatura constante fue irradiada con luz dispersa. Después de un cierto
tiempo el contenido de cloro disminuyó en un 20% con respecto a la mezcla
original y la mezcla resultante tenía la siguiente composición: 60% volumen
de cloro, 10% volumen de hidrógeno y 30% volumen de cloruro de hidrógeno.

Problemas:

1. ¿Cuál era la composición de la mezcla original?

2. ¿Cómo se producen cloro, hidrógeno y cloruro de hidrógeno?

------------------------- S O L U C I O N --------------------------

1. H2 + Cl2 = 2 HC1
30 partes en volumen de cloruro de hidrógeno pueden formarse solamente
por la reacción de 15 partes en volumen de hidrógeno y 15 partes en volumen
de cloro.

Por lo tanto, la composición original de la muestra tiene que ser:


Cl2: 60 + 15 = 75% H2 :
10 + 15 = 25%
- 15 -

EJERCICIO N° 15 1» OIQ

2. Cloro e hidrógeno se producen por electrólisis de una solución acuosa de


NaCl:
NaCl , , = Na , / + Cl . „
,
/aq/ /aq/ /aq/

ánodo: 2C1~ - 2e = Cl,


cátodo: 2Na + 2e = 2Na
2Na++ 2H20 = 2NaOH + H2
El cloruro de hidrógeno se produce por la reacción del hidró geno con
el cloro.
- 17

EJERCICIO N° 16 Io OIQ

Escriba las ecuaciones para las siguientes reacciones:

1. Oxidación de cloruro de cromo/III/ con bromo en solución alcalina /KOH/.

2. Oxidación de nitrito de potasio con permanganato de potasio en solución


acida /HjSO^/.

3. Acción del cloro sobre lechada de cal /Ca/OH/2/en una mezcla de reacción
en frío.

-------------------------- S O L U C I O N -------------------------

1. 2CrCl3 + 3Br2 + 16 KOH = 2K2Cr04 + 6KBr + 6KC1 + 8H20

2. 5KN02 + 2KMn04 + 3H2S04 = 2MnS04 + K2S04 + 5KN03 + 3H20

3. Cl2 + Ca/OH/2 = CaOCl2 + H20


- 17 -

EJERCICIO N° 3 Io QIQ

El gas que escapa de un alto horno tiene la composi


ción:

12.0% volumen de C02; 28.0% volumen de CO;


3.0% volumen de ; 0.6% volumen de CK^;
0.2% volumen de C.H.; 56.2% volumen de N,
2 4 2

Problemas:
1. Calcular el consumo teórico de aire /en m3/ que se necesita
3
para la combustión total de 200 m del mencionado gas si am bos, el gas
y el aire están medidos a la misma temperatura. /El contenido de oxígeno
en el aire es alrededor de 20% en volumen/.

2. Determine la composición de los productos de la combustión si el gas


fue quemado con "un 20% en exceso de aire.

-------------------------- S O L U C I O N ------------------------
°2
1. 2CO + 02 = 2C02
2H2 + 02 = 2H20 1.5
CH. + 20- = CO., + 2H.0 1.2
4 2 2 2
C2H4 + 302 = 2C02 + 2H20 0.6
17.3 partes x 5 = 86.5 partes de
aire
3 3 200 m de gas .... 2 x 86.5 = 173 m de
aire
+ 20% 34.6 3
' 207.6 m de aire

2. 207.6 : 5 = 41.52 partes de 02 : 2 = 20.76 partes de 02 por 100

ni3 de gas
20.76 x 4 = 83.04 partes de N2 para 100 m3 de gas
- 18 -

Balance : CO. > H. N2 °2



/partes en 12, ,00 3, .00 56. . 20 20. ,7
6
volumen/ 28. ,00 1, . 20 83, .04 -17 ,3
, 0
0. ,60 0. .40 139. , 24 3, .4
6
0. , 40 4, ,60

41. ,00

Total : 41.00 + 4.60 + 139.24 + 3.46 = 188.30 de partes en vo lumen de


los componentes.

% C02 = TsTÍlu- ' 100 = 21"77

% H2° = IWM ' 100 = 2-44


139 24

% N2 = TM73T • 100 = 73'95

% °2 = T5of ' 100 = 1'24

21.77 + 2.44 + 73.95 + 1.84 = 100.00%


19

EJERCICIO N° 4 Io OIQ

Un volumen de 31.7 cm3 de 0.1- normal de NaOH se necesita para


neutralizar 0.19 g de un ácido orgánico cuyo vapor es treinta veces más denso
que el gas hidrógeno.

Problema:

1. Escriba el nombre del ácido y su fórmula estructúralo /El ácido


involucrado es un ácido orgánico común.

S O L U C I O N

a/ El supuesto ácido puede ser: HA, H^A, H^A, etc.

3 3 - 3
n/NaOH/ = c x V = o.l mol/dm x 0.0317 dm = 3.17 x 10 mol

t< ., , 3.17 x 10"3 .


n/ácido/ = -------------- mol

donde v = l, 2, 3,.„.„..

m/ácido/ -3.17 x 10~3 ,


íiVz—ra—-7 ~ -------------- mol
M/ácido/ v

M/ácido/ = ------- °'19 g-, --- = v x 60 g/mol /l/


3.17 x 10" mol

b/ De la ley de los gases ideales se puede obtener:

Pi. _ _____
P2 M2

M/H2/ = 2 g/mol
M/ácido/ = 30 x 2 = 60 g/mol
20 -

Por comparación con /l/: v = 1

El ácido involucrado es un ácido monoprótico y su masa molar es 6 0 g/mo1.

El ácido es ácido acético: H 0


I '/
H-C-C I \
H OH
SEGUNDA

OLIMPIADA INTERNACIONAL DE
QUIMICA

KATOWICE 1969

POLONIA
- 22 -

EJERCICIO N° 1 2o OIQ

Una cantidad de 20 g de sulfato de potasio se disolvió en 150 cm3


de agua. La solución fue electrolizada. Después de la electrólisis, el
contenido de sulfato de potasio en la solu -ción era 15% en masa.

Problema:

1. ¿Qué volúmenes de hidrógeno y oxígeno se obtuvieron a una tem peratura


de 20°C y una presión de 101325 Pa?

S O L U C I O N

En la electrólisis solamente se descompone el agua y la cantidad


total de sulfato de potasio en el electrolito de la solución es constante.

La masa de agua en la solución:


-3
a/ antes de la electrólisis /presumiendo que p = 1 g.a /: m/IT20/ = 150 g

b/ después de la electrólisis:
20
m/H20/ = m/solución/ - m/K2S04 = - 20 = 113.3 g

La masa de agua descompuesta por la electrólisis: m/H20/ = 150

- 113.3 = 3607 g, i.e. m/H20/ = 2.04 mol.

Dado que, 2H20 -»■ 2H2 + 02


23

por le tanto,

n/H2/ = 2.04
mol

r./02/ = 1.02 mol

V/H2 n/H2/ . R.T 2_Q4 mol _ 8.314 J.mol"1 . K_1 . 293.15K


101325 Pa

~ 0.049 m3 ~ 49 dm3

V/O,/ = -i- V/H.,/ ~í 0.0245 m ^ 24.5


dm
1 n
3 3
2, -----i- V/H2/ - "•",c ~ 0/1 c
- 24 -

EJERCICIO N° 2 2o OIQ

Un compuesto A tiene 38.67% de potasio, 13,85% de nitro geno y 47.48%


de oxígeno. Al calentarlo se convierte en el compuesto B que tiene 45.85% de
potasio, 16.47% de nitrógeno y 37.66% de oxígeno.

Problema:

1. ¿Cuál es, son las fórmulas moleculares de los compuestos? Escriba la


ecuación química correspondiente.

S O L U C I O N

Compuesto A:
„ » 38.67 13. 85 47.48
KxNyOz x : y : z = -^-^ : : =

= 0.989 : 0.989 : 2.968 = 1 : 1 : 3

A: KN03

Compuesto B:

45.85 16.47 37.66


K N O p : q
p
: q rr •M • 39.1 ' 14 * 16

= 1.173 : 1.176 : 2.354 = 1 : 1

B: KN02

Ecuación: 2KN03 = 2KN02 + 02


- 25 -

EJERCICIO N° 3 2o OIQ

Una muestra de 10 cm3 de un hidrocarburo gaseoso se mez


ció con 70 cm3 de oxígeno y la mezcla se encendió por medio de
una chispa eléctrica. Cuando la reacción terminó y los vapores
de agua fueron licuados, el volumen final de los gases disminuyó
d
e
3

a 65 cm3.A esta mezcla se la hizo reaccionar con una solución de hidróxido


de potasio y el volumen de los gases disminuyó a 45 cm3

Problema:
1. ¿Cuál es la fórmula molecular del gas desconocido si el volumen de los gases
fue medido en condiciones estándar de temperatura y presión /STP/?

S O L U C I O N

El hidrocarburo gaseoso desconocido tiene la fórmula generaliC^H^


„/r _ 0,010 dm3 _______
- 2274 Q.010
R
x y~ „22.4
. *¿ dm .mol
.-I - mol

Balance de oxígeno:
Antes de la reacción: 70 cm3, i.e.
mol

< * i s< A C 3 0.045 ^ ,


Después de la reacción: 45 cm , i.e. —22~4

Consumidos en la reacción: ^2^4 1110^ ^e °2

De acuerdo a la ecuación:

CxHy +/X + 4/ °2 = XC°2 + 2 H2°

0 Q2Q , . t
0.020
De aquí, '2 mol de 02 reaccionaron con carbono y 22 4
- 26 -

mol de CC>2 se formaron /C + O., - CO^/,

0. 005 , , . ,., < 0.010 , ,


—4 mo-'- "e <->2 se coln^>lnaron con hidrogeno y ~~22—4 TOO-'- ^ e

agua se obtuvieron / 2H2 + 0^ = 2H2 /. n/C/ =

n/C02/ = mol n/H/ + 2n/H 0/ = mol

x : y = n/C/ : n/H/ = 0.0 20 : 0.0 20 = 1 : 1

De las posibles soluciones C2H2, C^ñ^, C^H^, C^H^,..,.

solamente C2H2 satisface a las condiciones dadas en el ejercicio,


1. e. el hidrocarburo desconocido es acetileno.
- 27 -

EJERCICIO N° 4 2o OIQ

El carburo ele calcio y el agua son las materias primas básicas


para la producción de:

a/ etanol

b/ ácido acético

c/ etileno y polietileno

d/ cloruro de -vinilo

e/ benceno

Problema:

1. De las reacciones químicas básicas para cada reacción por la cual pueden
obtenerse los compuestos arriba mencionados.

S O L U C I O N

Reacción básica : CaC2 + 2 H20 = Ca/OH/2 + C2H2 Del

acetileno puede obtenerse:

a/ etanol fíg so /catallzadox/


CH 3 CH + H,,0 ------ rp-^TT , ■ -i ¿* j ---- ► CH = CH-OH --------
2 H-2S04 diluido 2 i hol vinílico
a co

reordenamiento ^ CH3_ch = 0 reducción ^ C^.CH^QH

a.cetaldehído etanol

b/ ácido acético
HgSO^/cata1i z ador/
CH3-CH- + H20 lñQ ------- ► CH2 = CH-OH
2 4
- 28 -

reordenamiento , = Q oxidación , CU^C00E

acetaldehído ácido acético

c/ etileno, polietileno

CH E CH + H2 catalizador , CH = CH

etileno

nCH, = CH, Presión, Temper. , /


2 2 catalizador 2 2 n

d/ cloruro de vinilo

CH E CH + HC1 ------- <■ CH2 = CH - Cl


cloruro de vinilo

400 - 500°C^

e/ benceno
- 29 -

EJERCICIO N° OIQ

1/ Tres tubos de ensayo numerados /1-3/ contienen mezclas de dos sustancias


de los siguientes pares /4 variantes/:

1. ZnS04 - NaBr NaCl - Ca/N03/2 MgS04 - NH4CI

2. A1C13 - KBr CaCl2-NaN03 ZnCl2 - /NH4/2S04

3. KN03 - NaC03 KC1 - NH4C1 - Ba/N03/2


MgS04

4. MgCl, KNO. K2C03-ZnS04 Al/N03/3-NaCl

b/ Los tubos de ensayo numerados 4 y 5 contienen cada uno las siguientes


sustancias:

glucosa, sacarosa, urea, acetato de sodio, ácido oxálico. Problema:

1. Por medio de los reactivos disponibles sobre el mesón del la boratorio


determine el contenido de cada tubo de ensayoa De las razones para ambos
test efectuados, sus respuestas y describa la ecuación química con la
ecuación correspondiente.

Nota:
Para la identificación de las sustancias dadas en el ejercicio, los
siguientes reactivos estaban disponibles para los alum nos competidores: 1
N HC1, 3 N HC1, 1 N H2S04 concentrado, FeS04, 2 N NaOH, 20% NaOH, 2 N NH4C1,
2 N CuS04, 2 N BaCl2, 0.1 N AgNO-j, 0.1% KMn04, agua destilada, fenolftaleína
y anaranjado de metilo. Además, estaban disponibles otros elementos de
laboratorio, tales como alambre de platino, vidrio de cobalto, etc.
Permita reaccionar 10 cm3 de una solución 3 N HCl con la muestra
metálica(A los competidores se les dio exactamente pesadas las muestras de
magnesio, zinc o aluminio) y recoja el hi_ drógeno desprendido en la reacción
en una probeta graduada so -bre el agua. Ejecute el experimento con los
dispositivos de a-cuerdo a las instrucciones dadas.

Para simplificar el problema, calcule la masa de la muestra


metálica del volumen de hidrógeno presumiendo que este fue medido en
condiciones estándares /STP/.
TERCERA

OLIMPIADA INTERNACIONAL DE QUIMICA

BUDAPEST 1970

HUNGRIA
31

EJERCICIO N° 1 3o OIQ

Una cantidad de 25 g de un gas /densidad p = 2o05 g dm a STP/ fue


quemado obteniéndose 44 g de dióxido de carbono y 27 g de agua.

1. ¿Cuál es la fórmula estructural del gas /compuesto/?

------------------------- S O L U C I O N --------------------------

El gas desconocido - X

De la ley de los gases ideales: X/X/ = P/*A R; T,, _ 46 g-mol

n/X/ = -2-3-2 ----- — =0.5 mol


46 g.mol

4 4a
n/CO./ = -------- 7—i— = i mo1

2 44 g./mol

n/C/ = 1 mol

n/C/ = 12 g

n/H-O/ = ? -- = 1.5 mol


2 18 g/mol

n/H/ = 3

mol n/H/ =

3 g

El compuesto contiene también oxígeno, luego m/C/ + m/H/

= 1 2 g - + 3 g = 1 5 g < 2 3 g m/O/ = 23 g - 15 g = 8 g

n/O/ =0.5 mol


- 32 -

m/C/ : m/H/ : n/O/ = 1 : 3 : 0 . 5 = 2 : 6 : 1 La

fórmula empírica del compuesto es C,H,0.

dimetil éter

El etanol es un líquido en las condiciones dadas y por lo tanto el gas


desconocido es dimetil éter.
- 33 -

EJERCICIO N° 2 3o OIQ

Una muestra de soda cristalina con una masa de 1.287g se le hizo


reaccionar con un exceso de ácido hidroclórico y se produjeron 100„8 cm de
un gas /medido a STP/.

Otra muestra de una soda cristalina diferente con una ma sa de 0.715


g fue descompuesta por 50 cm3 de 0.2N de ácido sulfú rico. Después de la total
descomposición de la soda, el exceso de ácido sulfúrico fue neutralizado con
50 cm3 de una solución
0. 1 N de hidróxido de sodio / por valoración utilizando anaranja
do de metilo como indicador/.

Problemas:

1. ¿Cuántas moléculas de agua en relación a una molécula de Na2C03


contiene la primera muestra de soda?

2. ¿Tienen ambas muestras de soda la misma composición?

Masas atómicas relativas: Ar/Na/ = 23; A /H/ = 1 ;

A^/C/ = 12; Ar/0/ = 16

S O L U C I O
N
Muestra A: Na2C03 . xH20 m/A/ =

1.2 87 g

pV = nRT: n/C02/ = -j?fy- = 0.00 45 mol = m/A/

-1

M/A/ = M/Na2C03/ + . tt/H"20/

M/A/ - M/Na2C03/ = /286 - 106/ q-mol'1


= 1Q
M/H20/ 18 g.mol'1
34 -

Muestra A : Na2C03 . 10 HjO

Muestra B : Na2C03 . x HjO

m/B/ = 0.715 g

H2S04 + 2NaOH = Na2S04 + 2H20

n/NaOH/ = c.V = 0.1 mol.óm3 . 0.05 dm3 = 0,005 mol

Exceso de H_SO. : n/H-SO./ = 0.0025 mol


2 4 2 4

Cantidad de sustancias conbinadas con la muestra B:

n/H2S04/ = 0.00 25 mol = n/B/

= "n/W = S:0025 mol =


9 286 ^

Muestra B : Na2C03 . 10 B20


- 35 -

EJERCICIO N° 3 3o OIQ

Se mezcló monóxido de carbono con una proporción mayor de 1.5 veces


su volumen con vapor de agua.

¿Cuál será la composición /en % masa y % volumen/ de la mezcla


gaseosa en el equilibrio? Especifique si el 80% de monóxi do de carbono se
convirtió en dióxido de carbono.

S O L U C I O N

CO + H20^-C02 + H2

Presunción:

n/CO/ = 1 mol

n/H20/ = 1.5 mol

Después de la reacción:

ñ/CO/ = 0.2 mol

ñ/H20/ = 0.7 mol

ñ/C02/ = 0.8 mol

ñ/H2/ = 0.8 mol

V. = n. Sil i
i p

V, n. i i
Y
i

Y/CO/ = £• 1
2.3 mol
moj- = 0.08 ... 8 volumen % de CO

Y/H o/ =
2
l-'l -m°j- = 0.28
2.5 mol ¿
... 28 volumen % de H,0

V
36

EJERCICIO N° 4 3o OIQ

Y/C02/ = ™°\ =0.32 ... 32 volumen % de C02

Y/H2/ = 2*5 mol = °-32 32 volumen % de H


2

Antes de la reacción:

m/CO/ = n.M = 1 mol „ 28 g„mol_1 = 28 g m/H20/ =

1.5 mol . 18 g.mol-1 = 27 g

Después de la reacción:

1
m/CO/ = 0 . mol . 28 g.mo1 = 5 . g
2 6
m/H20/ = 0 . mol . 18 g.mol 1 =12 . g
7 6
m/C02/ = 0 . mol . 44 g.mol 1 =35 . g
8 2
m/H2 = 0 . mol . 2 g.mol 1 = 1 . g
8 6

w. =
m.

l'J^ ^
1
i
w/CO/ = 0.102 .o. 10.2 masa % de CO
55.0 g %

w/Ho0/ = ? ^ 0 . 229 ... 22 .9 masa % de H,0


' 2 ' 55.0 g i- 2
w/CO / = H' l 3 <v 0 .6 40 ... 0
2
6 4.0 masa % de CO.
55.0 g <\i 2
w/H-/ = e-¿,67i9- > 0 .029 . „. 2.9 masa % de
H.
¿ b b . u g ' v ¿
- 37 -

Una aleación contiene rubidio y uno de los otros -meta -


les alcalinos. Una muestra de 4.6 g de la aleación se le hizo
reaccionar con agua liberándose 2.241 litros de hidrógeno a STP.

Problema:
38

EJERCICIO N° 4 3o
OIQ

n/Rb/ . M/Rb/ + (0.2 - n/Rb/) . M/Li/ = 4.6 g n/Rb/ .85.5

+ (0.2 - n/Rb/) . 7 = 4.6 g

n/Rb/ 0.040 8 mol

n/Li/ £ 0.1592 mol

% Rb = 0-0408 mol . 85,5 g^ol"1 ^ 4.6 g

% Li = 0-1592 mol 7 g. mol'2 1QQ % 24


4 o 6 g 'v.

1¿Cuál de los metales alcalinos es el otro metal


alcalino de la aleación?
2o ¿Qué composición en % de masa tiene la aleación?
Masas atómicas relativas : A^/Li/ = 7; Ar/Na/ = 23;
A^/K/ = 39; Ar/Rb/ = 85.5; Ar/Cs/ = 133.
S O L U C I O N
M - metal alcalino
Reacción : 2M + 2H20 = 2M0H + H2 n/H2/ = 0.1 mol
n/M/ = 0.2 mol
Masa molar media:
M = — = 4
>* ? . = 23 g.mol-1 n 0.2 mol 3
Con respecto a las masas molares de todos los metales
alcalinos, solamente el litio puede ser considerado,
i.e. la aleación está formada por rubidio y litio.
n/Rb/ + n/Li/ = 0.2 mol m/Rb/ = m/Li/ = 4.6 g
n/RB/ . M/Rb/ + n/Li/ . M/Li/ = 4.6 g

2¿Cuál de los metales alcalinos es el otro metal


alcalino de la aleación?
2o ¿Qué composición en % de masa tiene la aleación?
Masas atómicas relativas : A^/Li/ = 7; Ar/Na/ = 23;
A^/K/ = 39; Ar/Rb/ = 85.5; Ar/Cs/ = 133.
S O L U C I O N
M - metal alcalino
Reacción : 2M + 2H20 = 2M0H + H2 n/H2/ = 0.1 mol
n/M/ = 0.2 mol
Masa molar media:
M = — = 4
>* ? . = 23 g.mol-1 n 0.2 mol 3
Con respecto a las masas molares de todos los metales
alcalinos, solamente el litio puede ser considerado,
i.e. la aleación está formada por rubidio y litio.
n/Rb/ + n/Li/ = 0.2 mol m/Rb/ = m/Li/ = 4.6 g
n/RB/ . M/Rb/ + n/Li/ . M/Li/ = 4.6 g
39 -

EJERCICIO N° 5 3o OIQ

Una cantidad de 20 g de óxido de cobre/II/ fue tratado con una


cantidad estequiométrica de una solución tibia de ácido sul fúrico al 20% para
producir una solución de sulfato de cobre/II/.

Problema:

lo ¿Cuántos gramos de sulfato de cobre/II/ cristalino /CuSO^ . 5H.,0/ han


cristalizado cuando la solución se enfrió a 20°C?

Masas atómicas relativas: A^/Cu/ = 6 3.5; A /S/ = 32;


Ar/0/ = 16 ; Ar/H/ = 1

Solubilidad del CuS04 en 100 g de H2o


s = 20.9 g de CuS04 en 100 g de H20

S O L U C I O N

CuO + H2S04 = CuS04 + H20

^°/= - ÍT-f^or^ 0 - 2516 ^

n/H/ SO/ = n/CuSO/ = 0.2516

Solución de CuS04 obtenida por la reacción: m/solución de CuS04/


= mCuSO/ + m/solución de H2S04/

n/H SO,/ . M/H-SO /


=m/CuO/+w/H2S04/ -------------------
on ^ 0,2516 mol . 98 g mol-1
= 20 g + ------------ 0 -------------

= 143.28 g

Fracción de masa de CuS04: a/ en la

solución obtenida:
- 40 -

EJERCICIO N° 6 3o OK

m/CuS04/ m/CuS04/ „ M/CuS04/


= = 0o28
w/CuS04/ = -m/soiucign CuS04/ -m/soluci6n CuS04/

5/ en la solución saturada de CuS04 a 20 °C:

w/cusv = IIFTH = °-173


c/ en CuS04 . 5 HjO:

M/CuS04
W/CUS04/ = = °'639
M/CUS04 . 5 H20

Ecuación del balance de masa para el CuSC>4:

0.639 m1 + 0.17 m2 = 0.28 m ; donde: m^ - masa

del CuS04 . 5 í^O;

m2 - masa de la solución saturada de CuS04 a 20°C;

m - masa de la solución de CuS04 obtenida por la reacción a una temperatura


más alta.

0.639 m1 = 0.173 . /143.28 - m1/ = 0„28 . 143.28

m1 = 32.9 g

La cantidad producida de la cristalización es 32.9 g de CuSC>4.511,0


- 41 -

El óxido de un cierto metal contiene 25% en masa de oxl geno. Otro


óxido del mismo metal contiene 50.48% en -masa de oxí geno .

Problema:

1„ ¿Cuál es la masa atómica relativa del metal?

S O L U C I O
N
Oxido 1: M.O 2 x

w/M/ . w/O/

0,774 5 0.2255
X = /!/
M/M/ 16
5 4.95
M/M/

Oxido 2: M -0 2 y
m/M/
y =
M TW
w/O/
M 7Ó 7
0.4952 0 .5848
y = /2/
M/M/ 16
15.695
M/M/

Cuando /l/ se divide


y 3.5
x por /2/

54.95
15.695

x
- 42 -

EJERCICIO N° 6 3o OK

Por sustitución de x = 2 en la ecuación /l/ : •1


M/M/ = 54.95 g.mol M Oxido 2 = Mn207
= Mn Oxido J. = MnO
- 43 -

EJERCICIOS N° 7 - 8 /experimental/ 3o OIQ

Ejercicio N° 7

Una muestra desconocida es "una mezcla de 1,2-molar H^SO^ y 1.47


molar HC1.

Por medio de las soluciones y aparatos de laboratorio disponibles


determine:

1. La cantidad total de sustancia /por valoración/ del ácido pre senté en


1 dm de la solución.

2. La masa de ácido sulfúrico y de ácido clorhídrico presentes en 1 dm3 de


la muestra.

Ejercicio N° 8

Por medio de los materiales y reactivos disponibles eje cute un


análisis cualitativo de las sustancias dadas en los tu -bos de ensayos
enumerados y escriba sus fórmulas químicas.

De 10 ecuaciones para las reacciones químicas por las cuales se


probaron estas sustancias:

5 ecuaciones para reacciones de precipitación;

2 ecuaciones para reacciones involucradas con libera -ción de un


gas.

3 ecuaciones para reacciones de óxido-reducción.


CUARTA

OLIMPIADA INTERNACIONAL DE QUIMICA

MOSCU 1972 UNION SOVIETICA


- 45 -

EJERCICIO N° 1 4 o OIQ

Una mezcla de dos elementos sólidos con una masa de 1 , 5 2 g fue


tratada con un exceso de ácido clorhídrico. Un volumen de
0. 896 dm3 de un gas fue liberado en este proceso y quedó un resi^
dúo de 0 . 5 6 g el cual se disolvió en exceso de ácido.

En otro experimento, 1 . 5 2 g de la misma mezcla se hicie ron


reaccionar con un exceso de una solución al 1 0 % de hidróxido de sodio. En este
caso 0 . 8 9 6 dm3 de un gas también se liberaron pero quedaron 0 , 9 6 g de un residuo
sin disolver.

En el tercer experimento, 1 . 5 2 g de la mezcla inicial se


calentó a alta temperatura sin exceso de aire. De esta manera se
formó un compuesto el cual es totalmente soluble en ácido clorhí
3
drico y se liberaron 0 . 4 4 8 dm de un gas desconocido. Todo el gas obtenido se
introdujo en un recipiente cerrado lleno con ox^í geno. Después de la reacción
del gas desconocido con el oxíge -no, la presión del recipiente disminuyó
aproximadamente diez veces.

1. Escriba las ecuaciones para las ecuaciones arriba dadas y prue


be su exactitud por cálculo.

En la resolución del problema considere que el volumen del gas fue


medido a STP y las masas atómicas redondeadas a núme ros enteros.

-------------------------- S O L U C I O N ---------------------------
a/ Reacción con ácido clorhídrico:
3
1 . 5 2 g - 0 . 5 6 g = 0 . 9 6 g de un metal que reaccionó, 0 . 8 9 6 dm de hidrógeno
se formaron

masa del metal que se combina = 1 1 . 2 . ¡? * 9 Q , = 1 2 g


- 46 -

Posibles soluciones: Masa

atómica
relativa "Valencia Elemento Satisfase?
12 I C no
24 II Mg si
36 III Cl no

Reacción: Mg + 2HC1 = MgCl2 + H2

b/ Reacción con hidróxido de sodio:

1.52 g - 0.96 g = 0.56 g de un elemento que reaccionó, 0.896 dm3


de hidrógeno se formaron
0 56
masa del elemento que se combina = 11.2 . o ° 896 ~ 7 ^

Posibles soluciones: Masa

atómica
relativa "Valencia Elemento Satisfase?

7 I Li no
14 II N no
21 III Ne no
28 IV Si si
: Si 2NaOH + H20 = Na2Si03 + 2H2
+
Reacción

c/ Combinando ambos elementos: 2 Mg +

Si = Mg2Si

Q.96 g Mg + 0o56 g Si = 1.52 g de siliciuro Mg^Si^ w/Mg/ =

£96_|-0.63, w/sl/. i^=0.37 Q.63 0.37 ,


* •• y = -24" : ir = 2 ■■ 1

siliciuro : ~Mg2Si
- 47 -

d/ Reacción del siliciuro con ácido: Mg2 + 4HC1

= 2 MgCl2 + SiH4

n/Mg2Si/ = lo52 9 _a = 0.0 2 mol 76 g.mol


n/SiH,/ = Q°448
J1"3 ---- . = 0.02 mol
22.4 din .mol""1

e/ Reacción del silano con el oxigeno: SiH4 + 2

02 = Si02 + 2H20

V = 1 dm3
n
Presumiendo que T = Const; P2 = ---- . P^
nl

n /O / = ------------------ = 0.0446 mol


¿ 2 2 . A dm . mol-1

Consumo de oxígeno en la reacción : n/O^/ = 0.04 mol El resto

del oxígeno está en el recipiente cerrado:

n2 = Q.Q446 mol - 0.04 mol = 0.0046 mol

= 0.0046 mol v 2 0.Q4 mol •


yl % u - x Fl
- 48 -

EJERCICIO N° 2 4o OIQ

Una mezcla de hierro metálico con óxidos de Fe/II/yFe /III/


recientemente preparados fue calentada en un recipiente ce rrado en una
atmósfera de hidrógeno. Una cantidad de 4.72 g de la muestra cuando
reaccionó produjo 3.92 g de hierro y 0.90 g de agua.

Cuando la misma cantidad de la muestra se le permitió reaccionar


con un exceso de una solución de sulfato de cobre/II/, se obtuvieron 4.96 g
de una mezcla sólida.

Problema:

1. Calcular la cantidad de 703% de ácido clorhídrico /p = 1.03 g.cm /que se


necesita para la total disolución de los 4.72 g de la mezcla inicial.

2. ¿Qué volumen de un gas a STP es liberado?

Masas atómicas relativas: A^/Q/ = 16; A^/S/ = 32; Ar/Cl/=32.5

Aj/Fe/ = 56; A^Cu/ = 64.

------------------------- S O L U C I O N ------------------------------

1. Reducción por Hidrógeno:

FeO + H2 = Fe + H20

n/Fe/ = n/FeO/; n/H20/ = n/FeO/

Fe203 + 3 H2 = 2 Fe + 3 H20

n/Fe/ = 2 n/Fe203/; n/H20/ = 3 n/Fe.^/

La masa de hierro después de la reducción: 3.9 2 g

La cantidad total de sustancia de hierro después de la reducción:


n/Fe/ + n/FeO/ + 2 n/Fe 0^/ = —3,92 9 _^ = 0.07 mol /l/
56.g.mol
- 49 -

b/ Reacción con sulfato de cobre /II/:

Fe + CuSO. = Cu + FeSO, 4 4

Aumento de la masa: 4„96 g - 4072 g = 0„24 g

Después de la reacción de 1 mol de Fe, un aumento de masa se ría:

M/Cu/ -M/Fe/ = 64 g.mol""1 - 56 g.mol-1 = 8 g.mol-1 Cantidad de

la sustancia de hierro en la mezcla:

n/Fe/ = °'24 g _± = 0„03 mol /2/ 8 g.triol

c/ Formación de agua después de la reducción: 0.90 g H20,

i.e. 0.05 mol

0.05 mol = n/FeO/ + 3 n/Fe^/ /3/

Por resolución de las ecuaciones /l/, /2/ y /3/: n/FeO/ =

0.02 mol n/Fe203/ = 0.01 mol

d/ Consumo de ácido:

Te + 2 HC1 = FeCl2 + H2 FeO + 2 HC1

= FeCl2 + H20

Fe203 + 6 HC1 = 2 FeCl3 + 3 H20

n/HCl/ = 2 n/Fe/ + 2 n/FeO/ + 6 n/Fe^/ =

= 0.06 mol + 0.04 mol + 0 o 0 6 mol = 0.16 mol Una parte de hierro

reacciona de acuerdo a la ecuación: Fe + 2 FeCl3 = 3 FeCl2

n/Fe/ = -— n/FeCl3/ = n/Fe203/ n/Fe/

= 0 . 0 1 mol
- 50 -

Significa que el consumo de ácido disminuye en 0.02 mol. El consumo

total de ácido: n/HCl/ = 0.14 mol

V/ 7.3% HC1/ = n'M = °'14 mo1 • 36'5 * cm3


68
w p
' 0.073 . 1.03 g.cm"3

Volumen de hidrógeno: Fe + 2

HC1 = FeCl2

Hierro en la muestra: Q»03 mol

Hierro que reaccionó con FeCl^ : 0.01 mol

Hierro que reaccionó con ácido : 0.02 mol

De aquí, 0.02 mol de hidrógeno, i.e. se formaron 0.448 dm3 de hidrógeno»


- 51 -

EJERCICIO N9 3 4o OIQ

Un volumen de 200 cm de una solución 2-normal de cloru _3


ro de sodio / p = 1.0 g.cm / fue electrolizada con agitación per manente en
una celda electrolítica con electrodos de cobre. La e lectrólisis se detuvo
cuando 22.4 dm /a STP/ de un gas fue libe rado en el cátodo.

Problema:

1. Calcular el porciento de masa de NaCl en la solución después de la


electrólisis.

Masas atómicas relativas: A^/H/ = 1; Ar/0/ = 16; Ar/Na/=23;


Ar/Cl/ 35.5; Ar/Cu/ = 64.

-------------------------- S O L U C I O N ---------------------------

Los cálculos se hacen sobre la presunción de que las siguientes reacciones


tienen lugar:

2 NaCl = 2 Na+ + 2 Cl"

Cátodo : 2 Na+ + 2 e + 2 Na

Anodo : 2 Cl~ - 2 e = Cl2

Cl2 + Cu = CuCl2

Dado que la solución del electrolito está siendo agitada perma -nentemente,
se deben considerar las siguientes reacciones:

Cu Cl2 + 2 NaOH = Cu/OH/2 + 2 NaCl


Presumiendo que todo el cloro reacciona con el cobre, la masa de NaCl en el
electrolito permanece constante durante la electrólisis.

m/NaCl/ = n.M = c.V.M =


= 2 mol.ctaf3 . 0.2 dm3 . 58.5 g.mol-1 = 23.4 g
- 52 -

V/H2/ = 2 2 . 4 dm3, i.e, n/H2/ = 1 mol

La cantidad de agua disminuye en la solución por:

N/H2 0 / = 2 mol; n/H2 0 / = 3 6 g

Antes de la electrólisis:
m/ solución de NaCl/ = V»p = 2 0 0 cm3 . 1 . 1 0 g.cm-3 = 2 2 0 g

% NaCl = G
. 100 = 100 = 10.64
220 g

Después de la electrólisis:

m/solución de NaCl/ = 2 2 0 g - 3 6 g = 1 8 4 g

% NaCl = T I '- -?-


3 4 A 100 = 12.72
184 g
- 53 -

EJERCICIO N° 4 4o OIQ

Cantidades de 50 g de una solución de hidróxido de so -dio al 4%


y 50 g de una solución de ácido clorhídrico al 1.825 fueron mezcladas a 20°C
en un recipiente aislado térmicamente.La temperatura de la solución obtenida
de esta forma aumentó a 23.4 °C. Luego, se agregaron 70 g de una solución de
ácido sulfúrico a 20°C de temperatura a la solución arriba descrita.

Problema:

1. Calcular la temperatura final de la solución resultante.

2. Determine la cantidad del residuo seco que queda después de la evaporación


de la solución.

En el cálculo del primer problema use el valor de la capacidad calórica:


c = 4.19 J.g-1 . K-1.

Masas atómicas relativas: A^/H/ = 1; Ar/0/ = 16; Ar/Na/=2 3;


AJS/ =32; Ar/Cl/= 35 „5

S O L U C I O N

a/ NaOH + HC1 = NaCl + H20


nNaOH/ = TO/5OlW gSH/a0H/'W = ^ * °°-í = °°05 mo1
N
' 40 g.mol

50
n/HCl/ = ?• °- 018 g5 = 0.025 mol 36.5
g.mol

sin reaccionar:

n/NaOH/ = 0.0 25 mol

b/ Cuando se forma 1 mol de agua el calor de neutralización

te' = «.c . ¿t = lOOg . 4.19 J.g"1 . K1 . 3.4 K =


neutr. n/*20/ " 0.025 mol
- 54 -

= - 57000 J.mol"1

c/ NaOH + H.SO, = NaHSO, + H-0


2 4 4 2

La temperatura de la solución resultante se calcula de a-cuerdo a la


ecuación:

m^o c^ c t^ + m2.c 2 o t 2 = m.c.t

iti^.tj + m 2 o t 2 = m.t.

_ m1.t1 + m2„t2 ^ loo. 23.4 + 70 . 20 _

d/ La temperatura aumenta debido a la reacción de NaOH con H,,S04 como


sigue:
_ n/H20/ • AHneutr. m. c
t = =

0.025 mol . 57000 J. mol"1 _


- ------------------------------------ ¿ Kv

170g . 4.19 J.g . K La temperatura final de la solución

es: t = 22 + 2 = 24°C

2, e/ Se presume que las siguientes reacciones ocurren cuando la solución se


ha evaporado:

NaCl + NaHS04 = Na2S04 + HC1

Na2S04 es el residuo seco.

m/Na2S04/ = n.M = 0.025 mol . 142 g. mol"1 = 3.55 g


EJERCICIO N° 5 - 55 - OIQ

40

Solamente un producto se obtuvo por la reacción de bromo con un


hidrocarburo desconocido. Su densidad es 5.207 veces mayor que la del aire.

Problema:

1. Determine la fórmula estructural del hidrocarburo desconocido. Masas


atómicas relativas: A^/H/ = 1; A^/C/ = 12; Ar/Br/ = 80.

-------------------------- S O L U C I O N ---------------------------

La masa molecular relativa del hidrocarburo puede calcularse del valor de la


densidad:

Mr/RBr/ = 29 . 5.207 = 151

Los derivados mono-bromurados pueden solamente ser toma dos en


consideración porque la masa molecular relativa de los de rivados
di-bromurados debieran ser mayor que:

M /RBr_/ > 1 6 0 r
2

Mr/RH/ = 151 - 80 + 1 = 72

La correspondiente fórmula resumida: C.H 5 12

Por las condiciones dadas /el único producto/


el hidrocarburo desconocido es el 2,2-dimetil propano:

C - CH
\

CH,
- 56 -

EJERCICIO N° 6 4o OIQ

Un compuesto orgánico A tiene 41.38% de carbono, 3.45% de hidrógeno


y el resto es oxígeno. El compuesto A calentado con etanol en presencia
de un ácido produce una nueva sustancia B la cual contiene 55.81% de carbono,
6.97% de hidrógeno y oxígeno.

Cuando al compuesto inicial A se le permite reaccionar con ácido


bromhídrico da el producto C el cual en agua hirvien-te de la sustancia D que
contiene 35.82% de carbono 4.4 8% de hi_ drógeno y oxígeno.

Una cantidad de 2.6 8 g de la sustancia D se requiere para reaccionar


con 20 cm3 de una solución 2 N de hidróxido de po tasio.

Problemas:

1. Determine las fórmulas estructurales de las sustancias: A, B, C y D.

Use el resultado experimental que el compuesto A produce agua al ser


calentado.

2. Escriba las ecuaciones químicas para las reacciones arriba escritas .

Masas atómicas relativas: A^/H/ = 1; Ar/C/ = 12; Ar/0/ = 16;


Ar/K/ = 39.

S O L U C I O N

1. Fórmulas estequiométricas de los compuestos:

A: C3HyOz *syiz = %|i : 3J45 , 55^ . ^

rr „ TT „ 55.81 6 .97 : 37.2 _


B: C H O m:n;p = —: -= ------- -, g ■ ■ -
2:3:4
m- n p r 12 1 lo
- 57 -

a: b: c = 4:6:5

D: C H.O 35 .82 59.70


a b
e 12 16
4.48

20 cm3 de 2 N KOH = oc04 mol KOH = 0.04/v mol de sustancia D = 2.68 g de sustancia
D, -v = 1, 2, 3, ....

1 mol de compuesto D = -v „ 67 g

Mr/D/ = 6 7, 6 134, ó 201, etc.

Debido a ambas fórmulas estequiométricas y la masa molecular relativa del


compuesto D, su composición es

Entonces las fórmulas moleculares para los compuestos A, B, Cson como sigue:

A: C4H404; B: CgH1204; C: C4H504Br

Ecuaciones¡

CH - COOH II + 2 CH CH OH = 2 H 0 + CH
3 2 2
CH - COOH A C00CH2CH3
CH - C00CH2CH3

CH - COOH CH,, - COOH ^ H-0 CH. - COOH I 2


HBr I ¿ * ¿ CH/CH/-COOH
II CHBr - COOH
CH - COOH

CH2 - COOH +. KOH = 2 H,0 + f 2 " C°°K


CH/OH/-COOH CH/OH/-COOK

CH - COOH CH - CO + H20
II
CH - COOH + CH - CO
calor

Compuesto A : ácido maleíco.


EJERCICIOS N° 7 - 8 /experimental/ 4o OIQ

Ejercicio N° 7

Determine las muestras desconocidas en los diez tubos de ensayo


enumerados -usando los reactivos y aparatos dispon! -Bles en el mesón del
laboratorio. Escriba las ecuaciones químicas para las más importantes
reacciones que utilizó para i -dentificar cada sustancia. En caso de que
la reacción tuvo lu gar en solución, escriba las ecuaciones en forma iónica.

Ejercicio N° 8
- 58 -

El J.Q de junio se preparó una mezcla de ácido fórmico con exceso


de etanol. Esta mezcla se mantuvo en una vasija ce rrada aproximadamente por
un mes. Determine cuantitativamente la composición de la mezcla el día de
la competencia, usando solamente los reactivos y aparatos disponibles en el
mesón del la boratorio. Calcule las cantidades de ácido y etanol en % masa
que fueron mezclados inicialmente.
QUINTA

OLIMPIADA INTERNACIONAL DE QUIMICA *

SOFIA 1973

BULGARIA

En la nitración de un hidroxi derivado de benceno se for mó un


compuesto que tiene 49.0% en masa de oxígeno. Una carga de 4350 C se requiere
para la total electroreducción de 0.458 g del compuesto con una eficiencia
de 80%.

Problema:

1. Determine tanto la fórmula estructural como la estequiométri-ca del


compuesto si el producto de la reducción electroquímica es un hidroxi amino
derivado aromático.

F / carga de Faraday / = 96500 C.

------------------------- S O L U C I O N ---------------------------
a/ Fórmula del compuesto:
r C-H O N
6 x y z
El compuesto es un hidroxi nitro derivado de benceno:

CCH, , - , /OH/ _
/NO-/ 6 6-/y-2z/-z/ 'y-2z
2' z

b/ Ecuación de la reducción: R-N02 + 6 H

= R-NH2 + 2 H20 Masa combinada del

compuesto: _ _ M/compuesto/
E " --------- s í ---------- /V

Una cantidad de carga se requiere para la reducción electro -química:


- 60 -

EJERCICIO N° 1 5o OIQ

Q = 435Q C . 0.8 = 3480 C

Masa combinada del compuesto:

TT - m - .-Q 96500 C _
~ 3840. = Q-458 ' 3480 c " 12'7
- 61 -

En relación con /!/: M/compuesto/ = 32.7 /2/

c/ % 0 y . m/0/ . 100
M/compuesto/

49 y o 16 . 100 _ „, _ . . ,_
/3/
_
M/compuesto/ => M/compuesto/ = 32.7 y

M/compuesto/ = 6 M/C/ + x M/H/ + y M/C/ + z M/N/ M/compuesto/

= 6 . 12 + x + 16 y + 14 z

Tomando en consideración la fórmula general del hidroxi derivado de Benceno


desconocido:

x = 6 - /y - 2z/ - z + y - 2z

x = 6 - z /4/ Entonces: M/compuesto/ = 72 + 6 - z + 16 y + 14 z

M/compuesto/ 78 + 16 y + 13 z /5/

Resolviendo las ecuaciones /2/, /3/, /4/ y /5/ se obtiene: M/compuesto/ = 229

g.mol"1

= 3

y = 7

z = 3

La fórmula molecular del compuesto es: C-H-C-N-, ó

CgH2/OH/ /N02/3.

El compuesto es 2,4,6-trinitrofenol
- 62 -

EJERCICIO N° 2 5° OIC

Una mezcla de un hidrocarburo gaseoso y oxígeno está en


3
un recipiente de un volumen de 1 dm a una temperatura de 406.5 K y una presión
de 101325 Pa, Hay dos veces más oxígeno en la mez cía que el necesario para
la reacción con el hidrocarburo. Después de la combustión del hidrocarburo
la presión en el recipien te /a la misma temperatura/ aumentó en un 5%.

Problema:

lo ¿Cuál era el hidrocarburo en la mezcla si la masa de agua for mada por la


combustión fue 0„162 g.?

S O L U C I O N

%r
Cantidades de sustancias reactantes y productos de la reacción:
Ecuación: C H + /x + \-/ 0, = x C0n + H„0
x y 4 2 2 2 2
m/H 0
n/H / = -----L. = °-162 ? = 0o009 mol
M/H20 18 g.mol

/C H / = 0°0Q9 mol _ 0.018 mol /!/ 2

x + ^
n/O,/ = /x + 2/ o °-009 mQl= ---------- i . 0.018 mol /2/
2 4 y_ y
2

/„„ / 0„009 mol x A 0_.0 . ,-,/


n/C02/ = 2 . ------------- = -y— . O0OI8 mol /3/
2

Antes de la reacción:
/ 1
n/mezcla/ =/ £-^r P-V 101.325 kPa . 1 dm3 = _Q.Q3
= ------------------------------------- n m ™oi
mol

r.t 1
8.314 J.mol . K-1 . 406.5 K
- 63 -

n/C H / + 2 n/0_/ =0.03 mol /4/ x y 2

Después la reacción p = 101.325 kPa . 1.05 = 106.4 kPa


/ ■, p.V 106.4 kPa o 1 dm3 n^-c ,
n/mezcla = *—-= = --------------- = --- 2—-= --------- =n 0.0315 mol

8.314 J.mol .K . 406.5 K

n/C02/ + n/02/ + n/H20/ = 0.0315 mol

n/C02/ + n/02/ = 0.0225 mol /5/

Cuando / ! / , / 2 / y /3/ se sustituyen en /4/ y /5/ se obtiene una ecuación con


dos incógnitas que cuando es resuelta:

x = 3; y = 6.

La fórmula estequiométrica del hidrocarburo desconocido es: C3E6-


- 64 -

EJERCICIO N° 3 5C OIQ

Vollímenes iguales /1Q cm / de soluciones 0.0_1 -molar de


CH,COOH y HCIO se mezclaron y diluyeron a un volumen total de 100
— 5
cm3. La constante de ionización del CH^COOH es 1.8 . 10 y la del HC10 es 3.7
. 10~8„

Problemas:

Calcular:

1. Grado de ionización para cada ácido en la solución:

2. Grado de ionización del HCIO si la solución diluida no contie


ne CH3COOH;

3. pH para la solución que contiene al mismo tiempo CH^COOH y


HCIO.

S O L U C I O
N
C
l

. , "3 ,
"1 ^2 10 ^.-3
mol o dm
«2, c 2

H30+ CH3CO
1. K.,
O
|
CH3COOH"] /!/
/a± + a2/a1c /a^ +

a2/c . a^c /l -

ax/c
1-OL,

CIO
/a1 +a2/ a2c 1 - a„
£ HCloJ

>>K2, por lo tanto también >> ct2 y a1 + a2 £

/2/
- 65 -

K^/l-ct^/ = cc^ c

c a1 + K1a1 - = O

o1 = 0.125

Cuando /2/ se divide por /!/:

/I - a2 /l -
a2/ a1

Después de la sustitución de : a2 -4
2.94 o 10
2
2. K„ a2 c
l-a„

a2 « 1

= a2 c

a2 = 6.08 . 10 -3

H3C a^c + a 2 C =/ai + cl2// C

/O.125 + 2.94 . 10 4/ . 10 3 ^ 1.25 . 10 4 mol.dm3

pH = 3.9
- 66 -

EJERCICIO N° 4 5o OIQ

Cuando se mezclan las soluciones de dos sustancias des


conocidas en razón estequiométrica, se forman 1.25 g de un precipitado el
cual contiene la sal de un metal bivalente. Este pre cipitado al ser calentado
a 1100°C se descompone en 0.70g de un óxido metálico sólido MO y un óxido
gaseoso. Después de la eva poración del filtrado, queda un residuo seco de
2.0 g de masa el cual por descomposición térmica a 215°C produce un óxido
gaseoso y 0.90 g de vapor de agua. El volumen total de la mezcla ga 3

seosa es 1.6 8 dm /STP/.

Problema:

lo Determine los compuestos desconocidos y escriba las ecuaciones químicas


para las reacciones arriba mencionadas.

S O L U C I O
N
a/ residuo seco: 2.0 g

0.90 g, i.e, 0.05 mol

: lol g 1.69

dm
n/mezcla/ =

22.4 dm . mol

n/A O /= n/mezcla - n/H.O/ = 0.0 25 mol

M/Ax°y/= -1
m _ 1.1 g = 44 g.mol
n ~ 0 .025
mol
- 67 -

EJERCICIO N° 5 5o OIQ

Utilizando sus conocimientos sobre las propiedades del benceno y


sus derivados, escriba las ecuaciones químicas para las reacciones por las
cuales el etil éster del ácido benzoico y los ácidos o-, m-, y p-aminobenzoico
se preparan en la forma más cor ta.

S O L U C I O N

a/ Síntesis del etil ester de ácido henzoico:


-- . KMn04

<
c2H5OH

(0> CH3 ------------ > <^C00H COOC2H5

0
>
b/ Síntesis del o- y el p- ácidos aminobenzoicos:

(Q >-ch3 C^C00H
0"
< > C00H
^N0_
ÍNO, ¿
SüO, NH,
KMnO, 2 2
HNO „ 4---
Fe►
+ + +4 - — --- >■
CH3 T ^T
N°2"<\^^)"CH3 N°2
H2S0
"\^3/>"COOH 1VC00H
L 4

c/-Síntesis del ácido m-aminobenzoico:

COO
H
N0

- 68

EJERCICIO N° 6 5o OIQ

Una mezcla gaseosa que contiene dos hidrocarburos de la misma serie


homologa es 14c4 veces más denso que el hidrógeno.Un volumen de 16„8 dm3 de
esta mezcla fue hidratada obteniéndose 350 g de solución cuando el producto
de la hidratación fue absorbido en agua. Diez gramos de esta solución fueron
calentados en presencia de óxido de plata/I/ el cual fue preparado de 70 cm3
de u-na solución 1-normal de nitrato de plata /l/. El Ag20 sin reaccionar se
disolvió en una solución de amoníaco acuoso y el preci_ pitado residual se
filtró. El filtrado fue acidificado con ácido nítrico y se le agregó un exceso
de bromuro de sodio dando un precipitado que pesó 9.4 g.

Cuando la mezcla de los hidrocarburos que permaneció sin reaccionar


se mezcló con un exceso de 50% de hidrógeno y se pasó sobre un catalizador
de Platino su volumen disminuyó a 11.2 dm3. Los volúmenes de los gases se
midieron en condiciones STP.

Problemas:

1. ¿Cuáles eran los hidrocarburos en la muestra?

2. Escriba las ecuaciones químicas para las ecuaciones arriba mencionadas.

3. Calcule la composición de la mezcla inicial en % en volumen.

4. ¿Cuánto /en %/ de cada hidrocarburo se hidrató?

-------------------------- S O L U C I O N ---------------------------

1. Mr = 2 . 14.4 = 28.8

Cuando se toma en consideración la reactividad de los hidro -carburos y


el valor de Mr la mezcla solo puede estar formada solamente por CH 3 CH
/Mr = 26/ y CH3~C 2 CH /Mr = 40/.
- 69 -

2. /V CH + H20 = CH3CHO

/2/ CH3COCH3

/3/ CH

CH3 3 CH + H20
/4/
2 AgN03 + 2 NH3 + 2 H20 = Ag20 + NH4N03 CH3CHO + Ag20 = CH3COOH
/5/
+ 2 Ag Ag20 +4NH3 + H20 = 2
/6/ Ag/NH3/2 | OH

/!/ CH-COOH + NH, = CH,COONH


i J j 4

/8/ Ag/NH3/2 OH + 3 HN03 = AgN03 + 2 NH4N03 + H2C CH3C00NH4 +

/9/ HN03 = NH4N03 + CH3COOH


NH4N03
/10/ AgN03 + NaBr = AgBr + NH3 + HN03

/ll/ NaNO

3
/12/
CH

.
CH = CH + 2 H2 = CH3

CH^C = CH + 2 H, = CH,-CH^-CH.

3. De acuerdo a /ll/ y /12/ y en relación al exceso de hidróge no, las


cantidades de sustancias son como sigue:

n/mezcla/ = 11'2 ^m— = 5.6 dm3, i.e. 0.25 mol

26 x + 40 . /Q.25 - x/ = 28.8 x = 0.25


0.2 n/C2H2/ =0.2 mol
n/C3H4/ = 0.025 mol

Antes de la hidratación:

n/mezcla/ 16.8 dm"


0.75 mol
-1
mol
22
3 0.07 dm = 0,07 mol
.4 dm' n/AgN03/ = c.V = 1

mol.dm
- 70 -

De acuerdo a /3/ :

n/Ag20/ = 0,035 mol

n/AgBr/ = -2- = ----- 9-¿i --- T =0,05 mol


" 188 g.mol

De acuerdo a /10/, /7/, y /5/ :

sin reaccionar: n/Ag.,0/ = 0,025 mol

reaccionaron : n/Ag20/ = 0,0 35 - 0,0 25 = 0,01 mol

Debido a la dilución, las cantidades de substancias en reac -ción fueron:

n/CH3CH.O/ = n/C2H2/ =0,35 mol

hidratación hidrogenación total

C2H2 : 0.35 mol 0.20 mol 0.55 mol

C3H4 : 0.15 mol 0.05 mol 0.20 mol


Z = 0.75
100 = 73.33
0„55 mol
0o75 mol °
% C2H2 /voló/ % C3H4 /voló/ 4 o % C2H2 /voló/ % C3H4 /vol./
0.20 mol
EJERCICIO N° 7 - 8 - 9 0.75 mol * /experimental/
0o35 mol 100 = 26.67
0.55 mol ' 63.64
0ol5 mol
100
100 = 75.0
Ejercicio N° 7

Las 0.20 mol siguientes soluciones de sales están


disponibles en doce tubos de ensayo numerados: AgNO-j, BaCl2, /NH4/2C03, NaCl
, KI, ZnCl2, NH4C1, Pb/N03/2, AlN03/3, CrClj, Cr/N03/3 y Hg/N03/2.

La numeración de los tubos de ensayos no corresponde al orden de


las sales arriba dadas. Pruebe el contenido de los tubos de ensayo por medio
del menor número de operaciones. En su respuesta señale la sal correcta con
el número del tubo de ensayo. Escriba las ecuaciones químicas para las
reacciones.

Ejercicio N° 8

Seis tubos de ensayo contienen los siguientes compues -


tos:

Na2C03 ó NaHC03 NiCl2 ó CuCl2


AgN03 ó Pb/N03/3 ZnCl2 6 Al/N03/3
ZnS04 ó KI NH4N03 ó Ba/N03/3
El numero de los tubos de ensayo no corresponde al

den de los compuestos. Pruebe el contenido de cada tubo de


yo con los reactivos disponibles Describa las reacciones p
dio de ecuaciones químicas

Ejercicio NJ 9
- 71 -

Hay tres tubos de ensayo marcados con los números 1, 2 y 3. Pruebe


el contenido de cada tubo de ensayo por medio de los reactivos disponibles
y -escriba la fórmula correcta del compuesto de cada tubo. Escriba las
ecuaciones químicas para las reacciones .
SEXTA

OLIMPIADA INTERNACIONAL DE QUIMICA

BUCAREST 1974

RUMANIA
Para la descomposición electroquímica del agua hay en él circuito
un voltmetro , electrodos de platino y una batería que tiene diez celdas
galvánicas conectadas en serie. Cada una tiene un voltaje de 1.5 V y una
resistencia interna de 0.4 Q. La re sistencia de el voltmetro es 0.5 ü y el
voltaje de polarización de la batería es 1.5 V Q La corriente eléctrica fluye
por 8 ho -ras, 56 minutos y 7 segundos a través del electrolito. El hidró geno
obtenido en esta forma se usó para una síntesis con otra sus tancia formando
así una sustancia gaseosa A, la cual por oxida -ción electroquímica se
convierte, vía óxido, en la sustancia B.

Por medio de la sustancia B es posible preparar la sustancia C, la


cual por reducción con hidrógeno produce la sustancia D. La sustancia D
reacciona a 180°C con una solución de áci do sulfúrico concentrado produciendo
ácido sulfanílico. Por dia zotación seguida de una reacción por acoplamiento
con p-N,N-dime tilanilina, se forma el anaranjado de metilo que es un colorante
azo.

Problemas:

1. Escriba las ecuaciones químicas para todas las reacciones men cionadas.

2. Calcular la masa del producto D.

3. Dé el nombre químico exacto para el anaranjado de metilo. Mués tre por medio
de las fórmulas estructurales los cambios que se producen debido a la
concentración de H^O"** en la solución
- 73 -

EJERCICIO N° 1 6o OIQ

Masas atómicas relativas: A /N/ = 14; = 16;

Ar/C/ =12; Ar/H/ = 1.


-------------------------------- S O L U C I O N

1. N2 + 3 H2 = 2 NH3

/A/

4 NH3 + 5 02 = 4 NO + 6 HjO

2 NO + 02 = 2N02

2 N02 + H20 + 1/2 02 = 2 HN03


/B/

2. m = . I.t F = 96500 C.mol"1

I = b°Eb ~ Ep _ 10 . 1„5 V - 1.5 V ,


Rv + b.Ri 0.5!) + 10 „ 0.4 í!

b - número de baterías;
- 74 -

EJERCICIO N° 1 6o OIQ

- voltaje de una batería; Ep - voltaje de polarización;

Rv - resistencia del voltmetro; R^ - resistencia interna

de la batería;

1 7no1"1.
n/H / = ? • . 3 A . 32167 s = lg 96500 Clmol

De las ecuaciones:

1 g H2, i.e. 0.5 mol H2 = 1/3 mol NH^ - 1/3 mol HN03

= 1/3 mol CgH5N02 » J./3 racl CgH5NH2 /D/


31 g de c,H SHj
Masa del producto D; m = n„M = ------------------------

3.

CH +

3 - H4
- 75 -

EJERCICIO N° 2 6o OIQ

La sustancia G puede ser preparada por varios métodos de acuerdo


al siguiente esquema:
S G HOH HCN
E- D
KOH
NH + HCH
A 3
NH
HOH 4
-
F
3

Compuesto A tiene 48,60% en masa de carbono, 8.10% de hidrógeno y 43.30% de


oxígeno. Reacciona con óxido de plata/I/ recién pre parado formando una
sal no disuelta. Una cantidad de 1.81 g de la sal de plata/I/ se formó de
0.74 g de compuesto A.

Compuesto D contiene 54.54 % de carbono, 9.09% de hidrógeno y 36.37% de oxígeno.


Se combina con NaHSO^ produciendo un compuesto que tiene 21.6% de azufre.

Problemas:

2. Escriba las fórmulas resumidas y estructurales de las sustancias A y D.

2. Escriba las fórmulas estructurales de las sustancias B, C, E, F y G.

3. Clasifique las reacciones en el esquema marcado por las fie -chas y discuta
con más detalles las reacciones B-»-G y D+E.

4. Escriba las fórmulas estructurales de los posibles isómeros e indique el


tipo de isomerismo de ellas.

Masas atómicas relativas: Ar/C/ = 12; A^/H/ = 1; Ar/0/ = 16;


Ar/Ag/ = 10 8; Ar/Na/ =23; Ar/S/ =32.
- 76 -

------------------------- S O L U C I O N ---------------

1, Compuesto A :

R-COOH + AgOH ------ * R-COOAg + H20


/C H O / ' x
y z' n

r, .
-
n
.
; y :

z= n
, -
,n
—jj- :
48.60
^— :
8,10
—3^-
43„30
= 1 : 2 : 0.67

Si n = 3, entonces la fórmula abreviada de A es:

C3H6°2

M/A/ = 7 4 g.mol-1 A -

CH3-CH2-COOH

Compuesto D :

/ C H O / p q
r'n

54.54 9.09 36.37 1 ,


,
n
p : q : r = -33- : = 1 :
-3— s
2 : 0.5

Si n = 3 la fórmula abreviada de la sustancia D es: C2H40

M/D/ = 44g.mol-1 D -

CH3-CHO

Reacción:
E
CH--C + NaHSO, --------- >■ CR,-CH-OH
3 <^ 3 3 |

O S03Na El producto de la reacción contiene

21.6% de azufre.

2. CH3-CH2-COOH CH3-CH-C00H
Cl

/A/ /B/
CH,-CH-COOH ------- — ------ -y CH,-CH-COOH
3 I 3 |
Cl OH
/B/ /G/

NH,
CH,-CH-COOH ---------- - ------ ► CH,-CH-COOH
3 | 3 |
Cl NH,
/B/ ¿ /C/

CH,-CH-COOH — ------- Y CH,-CH-COOH


- 77 -

3 I 3 !
NH2 OH
/G/
/C/

CH,CHO —SS! ------------------CH -CH-CN


OH
/D/ /E/
HOH, H30
+
CH,-CH-CN -------------- -—<-
CH.-CH-COOH
3 , 3 |
OH OH
/E/ /G/

NH-.+HCN
CH,-CHO ----------- - ------ <-CH,-CH-CN
3 3 |
NH2
/D/ /F/ HOH, H30+
CH,-CH-CN ----------------- »■
CH,-CH-COOH
3 | 3 |

/F/ /c/

I - reacción de sustitución;
II - reacción de sustitución nucleofílica
III - reacción de sustitución nucleofílica
IV - reacción de sustitución;
V - reacción de adición nucleofílica;
VI - reacción de adición, hidrólisis;
VII - reacción de adición;
VIII - reacción de adición, hidrólisis:

NH NH2

I ¿ ¿
CH,-CH-COOH 3
I OH OH
CH,-CHo-C00H
isomerismo de posición

CH--CH.-COOH CH..-CH-CHO CH-C-CH,


3 | OH
I 2 I OH OH I II I 2 OH
O OH

isomerismo estructural

H I
I CH-.-C-COOH
3 I OH 3
CH,-C-COOH CH,-CH-COOH
3 I OH
H OH
- 78 -

EJERCICIO N° 3 6o
OIQ

dextro /+/ levo /-/ mezcla racémica

estereoisomerismo

/isómeros ópticos/

-11
- 79 -

Se dispone de las siguientes soluciones 0.2-molar:

A/ HC1 B/ HSO~ C/ CH3COOH D/ NaOH

E/ CO 2 ' F/ CH3C00Na G/ HPO2" H/ H2S04

Problemas:
J.» Determine la concentración de H-jO+ en
Ia solución G.
2. Determine el pH en la solución A.

3. Escriba la ecuación para la reacción química que tiene lugar cuando a las
sustancias B y E se les permite reaccionar y señale el par ácido-base
conjugado.

4. Campare las propiedades ácido-base de las sustancias A, B y C y determine


cual de ellas es la más básica. Explique su deci_ siÓn„

5. Escriba la ecuación química para la reacción entre las sustan cias B y G


y explique el desplazamiento del equilibrio.

6. Calcule el volumen de la solución D que se requiere para neu-


3
tralizar 20o0 cm de la solución H.

7. Escriba la ecuación química para la reacción entre las sustan cias C y E


y explique el desplazamiento del equilibrio.

8. ¿Cuál sería el volumen de ácido clorhídrico presente en un li tro de la


solución A, si estuviese en estado gaseoso a una presión de 202.65 kPa y
a una temperatura de 37°C?

Constantes de ionización: CH3C00H + H20

* CH3COO~ + H30+ H2C03 + H20 + HC0~


-
K = 1.8 .5
+ H30+ HC0~ + H^O t CO2" + 10
K, = 4.4 . 10
H30+
K = 4.7 .
10
a
- 80 -

EJERCICIO N° 3 6o OIQ

HSO" + H,0 SO2 + H,0+ K = 1.7 . 10 2


4 ¿ 4 3 a

HPO2" + H-O -* P03~ + H,0+ K = 4.4 .


10-13
4 2-(- 4 3 a

Masas atómicas relativas: A^/Na/ = 23; A /S/ = 32;

A /O/ = 16
r

------------------------- S O L U C I O N

1. CH3COH + H20 * CH3C00~ + H30+

[CH C00 J 0 [H 0 ] [H 0 ]'


3 3
+
3
+

K
a |CH3COOHJ

r 30+l =
H \| Ka . c = \jl.8 . 10 5 .

2. pH = -log H,0+ = - log 2 . 10-1 ; pH


=0,7

I2- 2- 1 -
3. HSO. + CO, = SO,
+ HCO,
, 4 3 i4 3

A B
l 2 B A2

4. Por comparación de las constantes de ionización se tiene:

K /HC1/ > Ka/HSO~/ > Ka/CH3COOH/


Así la fuerza de los ácidos en relación al agua decrece en el orden arriba
dado.

CH^COO es la base conjugada más fuerte, mientras que el Cl es el más


débil.

5. HSO~ + HPO2- * H P04


2
+ S04~

0.2 [H 0 J =3
+ 1.9 . 10~3 mol.dm3

-11
- 81 -

EJERCICIO N° 4 6o OIQ

K /HSO."/ >> K /HPO2 /


a 4 a 4
- 2-
El equilibrio se desplaza hacia la formación de H2P04 y S04

6. CH3COOH + C02~ + CH3C00~ + HCO~

CH.COOH" + HCO~ ■+ CH,COO~ + H-CO,


3 3 +• 3 2 3

K /CH",COOH/ > K /H-CO.,/ > K /HCO~/


a j a ¿ 3 a J

El equilibrio se desplaza hacia la formación de CH3COO~ y H2C03.

7o n/HnS0./ = c.y + 0.2 molodm"3 . 0.02 dm3 = 0.004 mol 2 4

v70.2-molar NaOH/ = -2- = °'008™01 = °-04 ^


c 0.2 mol dm

o / _ n.B.T 0.2 mol . 8.314 J.mol-1 . K"1, 310 K _


8. V/HC1/ - —p --------------- 203.65 kPa -------------------------
= 2.544 dm3
- 82 -

Una mezcla contiene dos compuestos orgánicos, A y B. Am


Bos tienen oxígeno en sus moléculas y pueden mezclarse en cualquiera
proporción. La oxidación de esta mezcla en frío produce una sola sustancia
C la cual se combina con NaHSO^o La razón de las masas molares entre la
sustancia formada en la reacción con NaHSO, y la sustancia C es igual a 2„7931a

La mezcla de las sustancias A y B es quemada en presen


cia de una cantidad estequiométrica de aire /20% 0^ Y 8Q% N2 en volumen/ en
un eudiómetro produciendo una mezcla de gases con un volumen total de 5.4
32 dm a STP. Después esta mezcla gaseosa se hace burbujear a través de una
solución de Ba/OH/2,su volumen dis minuyó en un 15.46%.

Problemas:

1. Escriba las fórmulas estructurales de A y B.

2. Calcule la razón molar de las sustancias A y B en la mezcla.

Masas atómicas relativas: Ar/C/ = 12; A^/O/ = 16;

Ar/S/ = 3 2 ; Ar/Na/ = 2 3
1.
M/C/ +104 -1 S O L U C I O
M/C/
N
77
M C =58
g.mol
= 2.7931 R-C-H
II 0
C , • . CH-^^j^—CH
o
M/C/
- 83 -

EJERCICIO N° 4 6o OIQ

A ... CH,-CH-CH, J I J OH

B ... CH3-C-Cff3
O

La mezcla gaseosa puede contener en condiciones STP solamente CO2 y N2-


El dióxido de carbono es absorbido por la solución de ñidróxido de Bario
y por lo tanto:

/a/ V/C02/ = 5,432 dm3 . 0.1546 = 0.84 dm3


/b/ V/N2/ = 5,432 dm3 - 0.84 dm3 = 4.592 dm3

/c/ CH3-CH-CH3 + 9/2 (02 + 4 N2) = 3 C02 + 3 H20 + 18 N2 OH


/d/ CH,-C-CH, + 4 CO, + 4 N,) = 3 CO, + 3 H,0 + 16 N,
3 ^ 3 2 2 2 2 2
Se designa la cantidad de sustancias como:

CH3-CHOH-CH3 ... x

CH3-CO-CH3 ... y

De las ecuaciones /a/, /c/ y /d/:

/e/ 3x . 22.4 + 3y . 22.4 = 0.84 De las ecuaciones /b/,

/c/ y /d/: /£/ 18x . 22.4 + 16y . 22.4 = 4.592 Resolviendo

las ecuaciones /e/ y /f/, se tiene: x = 0.0025 mol: y

= 0.01 mol: x/y = 1/4


- 84 -

EJERCICIO N° 5 6 o OIQ

Una mezcla de dos metales que se encuentran en la Tabla


periódica de Mendelejev en grupos diferentes, reaccionaron en 3
caliente con 56 cm de hidrógeno /medido en condiciones STP/ pro duciendo dos
compuestos iónicos. Estos compuestos se hicieron reac clonar con 270 mg de
agua de los cuales sólo reaaccionó un ter -ció. Se formó una solución básica
en la cual el contenido de hi dróxidos era de 3Q% en masa y al mismo tiempo
se depositó un pre cipitado cuya masa es el 59=05% del total de la masa de
los productos formados por la reacción. Después de filtrado, el precipitado
fue calentado y su masa decreció en 27 mg.

Cuando una cantidad estequiométrica de carbonato de amo nio se


agregó a la solución básica obteniéndose un precipitado lige ramente soluble
y liberándose al mismo tiempo amoníaco y el contenido de los hidróxidos en
la solución disminuyó en un 16.81%.

Problemas:

1. Determine los metales y sus masas en la mezcla inicial.

S O L U C I O N

Hidruros iónicos se forman por combinación de metales alcalino o alcalinos


térreos con hidrógeno„ Con relación a las condiciones del ejercicio, habrá
un metal alcalino /M1/ y un metal alcalino terreo /M11/ en la mezcla.

Ecuaciones:

/!/ M1 + 1/2 H2 = MIH

/2/ M lX + H2 = M13:H2

/3/ MrE + H^O = MIOH + H2


- 85 -

/4/ K II H 2 + 2 H20 = yi I1 /OB/ 2 +

2 H2 Reaccionaron: 0.09 g H20, i.e. 0.005 mol.

Sin reaccionar: 0.18 g H20, i.e. 0.01 mol. /5/

Dado que todos los hidróxidos de metales alcalinos son muy solubles en agua,
el precipitado sin disolver es MII/OH/2, pero éste también es ligeramente
soluble en agua.

Así, se disolvió en la solución:

m' (MIOH + MI3:/OH/2l = Z

Por lo tanto:

30 100 2
= Z + o.i8 • = °-077 g

/6/ m' (MIOH + MI]:/OH/2) 0.077 g

Representa el 40.95%del total de la masa de los hidróxidos, i.e.

La masa total de los hidróxidos es como sigue: /!/ m^OH +

M^/OH/,) - °°077 .? • 100 - 0.188 g

La masa del sólido M^/OH/.^:

/8/ 0.188 g - 0.077 g = 0.111 g


Calentando:

(9/ MIX/OH/2 = M11 + H20


Disminución de la masa: 0.0 27 g /H2©/

/10/ Masa de M13^: 0.084 g

En relación a /8/, /9/ y /10/:


M/MI]tO/ = 0 .084 M/M IJ Q/ = 56 g.mol 1

MW + 18 " 0ai1

M/M11/ = M/MI:C0/-m/0/ = 56 - 16 = 40 g.mol"1 M11 = Ca

Precipitación con /NH4/2C03:


/ll/ Ca/0H2 + /NH.4/2C03 = CaC03 + 2NH3 + 2H20

De acuerdo a /5/ y /6/, la masa de la solución era: 0.18 g +

Q.Q77g = 0.257g

Después de la precipitación con /NH4/2C03

16.81 = ■ n/ y°y, . 100


m/solución/
- 86 -

Se designa como n' la cantidad de Ca/OH/2 que está presente en la solución.

M/Ca/OH/2 = 74 g.mol"1

Teniendo en cuenta la condición de la tarea y la ecuación /ll/, se tiene:

ifi si /O.077 - n' . 74/ . 100


• 0.257 - n' . 74 + 2n'.18

n' = 5 . 10"4 mol

La cantidad total de Ca/OH/2 /tanto en el precipitado como en la solución/:

/12/ nCa/OH/,/ = g_-, + 5 . 10_4mol = 0.002 mol,/i.e.


¿ 74 g.mol

0.148 g/.

De acuerdo a las ecuaciones /3/ y /4/: n/H20/ =

0.004 mol /para MI:CH2/


- 87 -

n/H2<V = 0„Q01 mol /para M H/ n/MIOH/

= 0.001 mol

De acuerdo a las ecuaciones ¡1/ y /ll/ : n/MXOH/ = 0.188

g - 0„148 g = Q.Q4 g
M/M^H/ = 2^°S/ = 0.04 g = 4Q r n/M OH/
0.001 mol
MIOH = NaOH

Composición de la mezcla:

Q..0CL2 mol Ca + o. 001 mol Na

O.080 g Ca + 0.023 g Na
- 88 -

EJERCICIO N° 6-7-8 /experimental/ 6o OIQ

Ejercicio N° 6

Tubos de ensayo contienen muestras desconocidas de:

- una sal de ácido carboxílico;


- un fenol;
- un carbohidrato;
- una amidao

Determine el contenido de cada tubo de ensayo utilizan do los


reactivos disponibles en el mesón del laboratorio.

Ejercicio N° 7

Determine los cationes en las soluciones N° 5, 6, 8 y 9 usando la


solución del tubo de ensayo N° 7.

Sin usar ningún indicador, determine si la solución en el tubo de


ensayo N° 7 es un ácido o un hidróxido.
Solución: Tubo de ensayo N° 5: NH*; N° 6: Hg2+; N° 7: 0H~;
3+ 2 +
N° 8: Fe ; N° 9: Cu

Ejercicio N° 8

La solución en el tubo de ensayo N° 10 contiene dos cationes y dos


aniones.

Determine estos iones por medio de los reactivos disponibles en el


mesón de laboratorio.

Solución: La solución en el tubo de ensayo N° 10 contiene:

— 7 — Ba
, Al , Cl , C03
SEPTIMA

OLIMPIADA INTERNACIONAL DE QUIMICA

VESZPREM 1975

HUNGRIA
- 90 -

EJERCICIO N° 1 7o OIQ

Cuántos gramos de alumbre, KAl/S04/2 . 12 H.,0 cristalizan de una


solución saturada de KAl/S04/2 a 20°C que tiene 320 g si se evaporan 16Q gramos
de agua de esta solución a 20°C?

/La solución saturada a 20 °C contiene 5.50% en masa de KAISO^./ Masas Atómicas

relativas: A /K/ = 39.10; A /Al/ = 26.98;

A /S/ = 32.06; Ar/0/ =16.0 ;

Ar/H/ = 1,

S O L U C I O N

a/ 5.5 g KAl/S04/2 = ^ 3 ^,^^2

/lao -J.0.10/ g H^O. ..J.0.J.0 g KAl/S04/2 . 12 H20 160 g

x = 17,98 g KAl/S04/2 . 12 H20

b/ Solución alternativa: Se
designa:
3 - masa del alumbre cristalizado
y - masa de la solución saturada de KAl/SO^/, que pertenece después de
la cristalización.

Masa de la fracción de KAl/S04/2 en el hidrato cristalino es igual a


0.544.

Entonces ; 320 = x + y + 160 i.e.


y ¡= 160 - x

Ecuación del balance de masa:

320 . 0.O55 = x . 0.544 + /16Q - x/ , 0.055 X = 17.9?


g

H20...x g KAl/S04/2 . 12 H20

Una aleación preparada con propósitos experimentales contiene


aluminio, zinc, silicio y cobre. Si J.000 mg de esta alea-
- 91 -

EJERCICIO N° 1 7o OIQ

3
ción se disuelven en ácido clorhídrico se liberan 843 cm de hidrógeno /0°C,
101.325 kPa/ y quedan 170 mg de un residuo sin disolver. Una muestra de
50Q mg de esta aleación se la hace reac-
3
cionar con Tina solución de NaOH produciendo 517 cm de hidrógeno /0°C, 101.325
kPa/ en este caso también queda una fracción sin di solver.

Problema:

I. Calcular la composición de la aleación en % de masa.

Masas atómicas relativas: A /Al/ = 26.9 8; Ar/Zn/ = 65.37;


A /Si/ = 28.09; A /Cu/ = 63.55.
r r

S O L U C I O N

1. HC1 disuelve : Al, Zn


NaOH disuelve: Al, Zn, Si

°-843 ^ = 37.61 mmol H, /Al,Zn/


22.414 dm3 . mol-1 2

2. 0.517 dm3 = 46>13 maQl H /A1¡ Zn? si/

22.414 dm3 . mol 1 2 De la diferencia : 8.52 nmol H2 ...

4.26 nmol Si

Si: m/Si/ = 4.26 mmol . 2 8.09 g.mol-1 = 119.7 mg

% si = -A1^7 TOg . 100 = 11.97


1000 mg ___________________________

Cu: m/Si + Cu/ = 170 mg


- 92 -

EJERCICIO N° 3 7° OIQ

m/Cu/ = 170 mg - 119,7 -mg = 50,3 mg / en 1000. mg de la aleación/

% Cu = 5,03

Al: m/Zn + Al/ = 1000 mg - 170 mg = 830 mg 3 x

x mg Al .... —j— ' 26 gs' 7caao


^- H
2 /830 - x/

83
mg Zn ... ° ^ mmol H2

"I- '26798 + 865,37X =


37'61 /mmo1 V
x = 618,2 mg Al /en 1000 mg de la aleación/ % Al

= 61,82

Zn: m/Zn/ = 830 mg - 618,2 mg = 211,8 mg /en 10Ü0


mg de la aleación/ % Zn = 21.18
- 93 -

Una muestra de 1500 mg de una aleación que contiene pía


+ 2+
ta, cobre y cromo es- disuelta y la solución conteniendo Ag , Cu
3+ 3 y Cr iones, se diluye exactamente a 5Q0 cm . Un décimo del
vo
lumen de esta solución se toma para el siguiente procedimiento:

Después de la eliminación de la plata y el cobre, el ero mo es oxidado


de acuerdo a la siguiente ecuación no balanceada:

OH" + Cr3+ + H202 = CrO2" + H20

Seguidamente se agregan 25.00 cm3 de una solución 0.100-molar de una sal de


Fe/II/. La siguiente reacción /escrita en forma no ba lanceada/ tiene lugar:
H+ + Fe2+ + CrO2- = Fe3+ + Cr3+ + H20

De acuerdo a la ecuación no balanceada:

H+ + Fe2+ + MnoT = Fe3+ + Mn2+ + H.O 4 2

un "volumen de 17.20 cm3 de una solución 0.020-molar KMnO^ se necesita para


la oxidación del Fe/II/ el cual permanece sin oxidar se en la solución.

En otro experimento,un volumen de 200 cm3 de la solución i-nicial


es electrolizada. Debido a reacciones secundarias la efi_ ciencia de la
electrólisis es 90% para los metales en consideración. Los tres metales se
depositan cuantitativamente en 14.50 minutos por el paso de una corriente de
2 A a través de la solución.

Problema:

1. Balancee las tres ecuaciones químicas y calcule la composi ción de la


aleación en por ciento en masa.
- 94 -

EJERCICIO N° 3 7° OIQ

Masas atómicas relativas: A^/CXL/ = 63.55; A^/Ag/ = 107.87;

A /Cr/ = 52.00 r

S O L U C I O N

1. Ecuaciones:

10 OH + 2 Cr3+ + 3 H^ = 2 CrO2 + 8 H20


8 H+ + 3 Fe2+ + CrO2- = 3 Fe3+ + Cr3+ + 4 H,0
4 2
8 H+ + 5 Fe2+ + MnOT = 5 Fe3+ + Mn2+ + 4 H,0
4 2

Contenido de cromo:

Corresponde:
17.20 . 0.02 = 0.344 mmol KMnO^ 5 .

0.3440.78 = 1.72 mmol Te2 +


3
2 +
Reaccionaron : 25 . 0.1 - 1.72 = 0.78 mmol Te
= 0.26 mmol Cr en 150 mg de la aleación

m/Cr/ = 2.6 mmol . 52 g.mol-1 = 135,2 mg en 1500 mg de la aleación.

% Cr = 9.013

Contenido de Cu y Plata: /dos soluciones alternativas/

a/ Q = 2- gg'52¿ gQ°° • 0.9 = 16.23 mF /600 mg


/435 mAh/

/ F - carga de Faraday/

QCr = 2'6 • 3 • ifw = 3'12 **•


/83.6 2 mAh/

Q/Cu 16'23 " 3'12 = 13-21


+ Ag/ = ^
/351 mAh/ m/Cu + Ag/ = 600 - . 135.2 = 545.9 mg

x mgCu + /545.9-x/mgAg

2x
Para la deposición del cobre: —g-^—¡r=- mF

545 9 - x
Para la deposición de la plata: ,, „,' Q- ----- mF

13ai = 6335- + 107787— ^

x = 362.6

m/Cu/ = 362.6 mg en 6QQ mg de aleación m/Ag/ =

183.3 mg en 60Q mg de aleación


- 95 -

% Cu = 60 .4; % Ag = 30.6

b/ Q = 40.575 mF /15Q0 mg /1087.4 mAh/

QCr= 2 . 6 . 3 = 7 . 8 m F / 209 mAh/

Q/Cu + Ag/ = 4Q-575 ~ 7,8 = 32.775 mF /878.4 mAh/ m/Cu + Ag/ = m/aleación/

- m/Cr/ = J.5QQ - 135.2 = 1364.8 mg

2x
Para la deposición del cobre: ^ mF

Para la deposición de la plata: ~^j[07%7 X

___ _ 2x , 1.364 „
8 - x
-i¿"JJT> 53.55 107.87

x = 9.06.26

m/Cu/ = 906.26 mg en J.500 mg de la aleación m/Ag/ =

458.54 mg en 1500 mg de la aleación

% Cu = 6Q.4? % Ag = 30.6
- 96 -

El pH de una solución que contiene 3% en masa de acido _3


fórmico /p = -1,0.049 g.cm / es igual a 1.97.

Problema:

1. ¿Cuántas veces debiera diluirse esta solución para lograr un aumento de


diez veces en el valor del grado de ionización?

Masas atómicas relativas : A^/H/ + 1.01; A^/C/ = 12.01;

Ar/0/ = 16.00.

S O L U C I O N

1004.9 g „ 0.03
1. o, = -i = 46 .03 g„mol^------ = 6.55 . ao^moLdm"3
1
" 1 dm3

pH = 1.97; [H+] = 1.0717 . 10-2 mol.dm"3 ÍHI


a-, = -i^J = 0.Q1636 /1.636 %/
1 c^

Cálculo de Cj después de la dilución/dos soluciones alternativas/:


2
CL, . c-
*/ Ka = -i^r- ™
2 2
a- . c, /10 a./ . c,
- 97 -

EJERCICIO N° 4 1° OIQ

De las ecuaciones /!/ y /2/ :

100 /l - a / cl = —1 - io1— = 117'6

6/ K = iü-L- = 1-071S2 • 10"4 , =


178 . lo"4
c-[H I 0.655 - 1.0715 „ 10

Ka/ 1 - 10a / _3 _3
c, = -------- =---- = 5.56 . 10 mol.&ra
- 98 -

EJERCICIO N° 5 7o OIQ

Un cierto aldehido B es subsecuente a un aldehido A en la serie


homologa de los aldehidos. Una cantidad de 19 g de aldehido se agregaron a
100 g de una solución acuosa que contiene 23 % en masa de aldehido A. La adición
de una solución amoniacal de AgNO^ a 2 g de la solución de aldehidos da como
resultado la pre cipitación de 4.35 g de plata.

Problemas:

1. Determine por cálculos los aldehidos usados.

2. Dé la fórmula estructural de los aldehidos.

Masas atómicas relativas: A^/C/ = 12.01; A^/O/ = 16,00;

Ar/H/ = 1.01; Ar/Ag/=10 7,87.

S O L U C I O N

1. Ecuación: Q Q
s + s +
R-C + 2 Ag + H-0 = 2 R-C% + 2 Ag + 2 H
\ \
H OH

Cálculo de la cantidad de sustancia /dos soluciones alternativas/:

a/ n/Ag/ = -- 4,35 g ----- = 0.04033 mol


107.87 g.mol x

n/A/ + n/B/ = 0.020165 mol en ^9 . 42 = 0.70588 g b/ i¿¿ . 4.35

= 258,825 g Ag = 2.3994 mol Ag

n/A/ + n/B/ = 1.199 8 mol in 42 g de aldehidos


- 99 -

Cálculo de las masas molares /tres soluciones


alternati vas/:
m/B/
A/ m/A/ + m/B/ = m/A/ +
n/A/ + n/B/
M A 77 77
M B ~ M7A7 M/A/ + 14

a/ m/A/ = 0.3865 g; m/B/ = 0.3194 g; n/A/ +

n/B/ = Q.Q20165 mol M/A/ = 30 g.mol"1; M/B/

= 44 g.mol-1;

b/ m/A/ = 23 m/B/ = 1 9 g;
g;

n/A/ + n/B/ = 1.1998 mol


M/A/ = 30 g.mol-1; M/B/ = 44 g0mol-:1;

B/ M/A/ = 30 + 14n; M/B/ = 44 + 14n g.mol ,-


1

m/A/ 30 m/B/
n/A/ + n/B/
+ 14n 44 + 14n

n = O; M/A/ = 30 g.mol 1; M/B/ = 44 g.mol-1;

^/ r, m/A/ + m/B/ -,c .-1


C/ = n/A/ + n/B/ = 35 g-mo1

If R = M = 30 g„mol
H, V 30 < M < 44

R CH3, M = 44 g. mo 1
44 < M < 5!
C2H5, M = 58 g.mol

Dado que:
-1 -1 -1
30 g.mol < 35 g.mol < 44 g.mol

Por lo tanto:

M/A/ = 30 g.mol formaldehido, metanal

M/B/ = 44 g.mol" acetaldehido, etanal


H-C
- 100

B:
O
/
CH--C
3
\
H
- 101 -

EJERCICIO N° 6 7° OIQ

La constante de equilibrio de la reacción H. + I. j 2 HI

a 60Q°C es igual a 70.0

Problemas:

1. ¿Cuánto yodo /en %/ se convertirá hasta alcanzar el equilibrio si los


reaccionantes se mezclan en:

a/ razón molar 1:1 a 600°C

b/ razón molar 2:1 a 600 °C /la cantidad de hidrógeno es dos ve ees mayor
que la del yodo/.

2. ¿Cuántos moles de hidrógeno debieran mezclarse con una mol de yodo, cuando
el 99% de yodo sea convertido en yoduro de hidró geno /ácido yodhidrico/
al alcanzar el equilibrio a 600°C?

S O L U C I O N

a/ /dos soluciones alternativas/


x
c -

H 2 x
K I [HI!2
. c
80 .7%
2 + 70 X
c

= 0.807

C C
^ [H2 J = |VJ = " °[HT] =

2 a
- 102 -

K =

A
2 2 4 a
4 a c
7¿2 /l - a/" c /l - a/'

1 2 a
1 - a a

= 0.807 80,7 %

1. 5/ /dos soluciones alternativas/

i/ H = 2c -
x;

:
1" *

______ 4 x ________
K =
/2c - x/ . /c - x/

x = 0.951 c; 0.951; 95.1

ü/ [H2] =
2 ca

[Hl] -K = A
2 2

4 c a
2c - ca

/2 - a/ . /I - a/c

0.951; 95.1%

2- M xc - Q.9 9 c; [l2~| = c - 0,99


c;

1.98 c;

2 2
J..98
i.asz c

c /l - 0.99/ . /x - 0.99/ x = 6".

59 mol H,

0.01 /x - 0.99/
- 103 -

EJERCICIO N° 7 OIQ

Un cierto hidrocarburo saturado A es transformado por oxi-


dación catalítica parcialmente en un alcohol secundario B y parcialmente en
una cetona C. Cuando la cetona se oxida con ácido nítrico en presencia de un
catalizador produce un compuesto D de fórmula C^E^Q^. Cuando el compuesto
D es calentado en presen -cia de anhídrido acético se forma una cetona E
acompañada de la liberación de C02 Y H2^" Los comPuestos E y C tienen estructu
-ras similares pero el compuesto E contiene un grupo metileno menos que el
compuesto C. El compuesto D es uno de los materiales importante en la
producción por policondensación de fibras sinté ticas.

Problema:

1. Escriba las fórmulas estructurales de los compuestos A, B, C,

D y E.

S O L U C I O
N

1. En general: Cetona Oxidación Acido carboxílico

C6H10°4 = C4Hg/COOH/2 = HOOC-/CH2/4-COOH

El producto D es ácido adlpico el cual es la materia básica para la


producción de nylon.

COOH
COOH
D:
El compuesto C es ciclohexamona, CH
porque .

/CH2 ,CH, XC=0 •CH,


oxidación CH2 ^COOH
CH2 CH2

\CH2 CH„

ciclohexamona -COOH

"CH,
ácido adípico

'CH,
- 104 -

CH,

C=0
CH I
'CH_
CH,
CH,
CH, •CH I
2 CH,
C: ciclohexamona
CH

~CH2 A:
■ CH,
ciclohexano
A .CH,
-CH-OH

CH, CH,

"CH,

B: ciclohexanol

/CH,/ 0 + H,0

H00C-/CH,/ -COOH arpfanhíHriHn if n = 2, 3


2 n

C = 0

+ C02 + H20

if n > 3

compuesto E: ciclopentanona

a/ Marque con el signo "+" en el cuadrado correspondiente a aque lias moléculas


para las cuales las aseveraciones escritas en el lado izquierdo de la
Tabla son verdaderas.

C2H4 N2H4 H2°2 H2F2


~"—Molécula Aseveración
Hay un enlace covalente entre
dos átomos igua -les.
La molécula contiene un doble
enlace.
La molécula es planar
|
La molécula es polar

Hay también un enlace de


hidrógeno en la mo lécula„
- Tiene propiedades básicas en i 1
relación con el agua. í
- 105 -

EJERCICIO N° 8 7°
OIQ

b/ Las siguientes aseveraciones en la Tabla están incompletas. Llene en los


lugares con puntos la palabra y las fórmulas que faltan , respectivamente.

Aseveración: Las estructuras electrónicas' de las moléculas


/iones/, que están en la Tabla una bajo la o-

CH4 C H
2 6 co2~ 2°r
c

NH4 N2H62 + N0Í N2

S O L U C I O N

tra son: ..........................................

C
2H4 2H 4
N H
2°2 H2F2

1 + + +

2 +

3 + +

4 + + +

6 +

B/ Aseveración: isoelectrónica

..c°2.
CK4 C

N2H2
2H6

+
co2"
NO~
.. NO*
C
2°42" *\'
2
N
- 106 -

EJERCICIO N° 9 /experimental/ 7° OIQ

Soluciones acuosas de los siguientes compuestos: AgNO^, HC1, Ag2S04,


Pb/N03/2, NH^ y NaOH están disponibles en botellas enumeradas.

Haga reaccionar las soluciones unas con otras yalíniee el número


de las botellas con la fórmula del compuesto.

Atención: El uso de cualquier otro reactivo no está per


rmitido. Al efectuar las reacciones no use el total del volumen de
3
las soluciones,, Unos pocos cm de solución deberán quedar en la botella después
de finalizado su trabajo. Es aconsejable hacer un resumen al principio de las
reacciones, pero el jurado evalúa rá solo aquellas que están escritas en la
Tabla correspondiente.

Escriba sus observaciones en los cuadrados de la Tabla bajo la


diagonal utilizando los siguientes símbolos uniformes:

precipitado blanco : +

precipitado coloreado : ++

formación de un complejo soluble en agua : { }

liberación de un gas : +

Escriba en los cuadrados sobre la diagonal las fórmulas químicas


del precipitado, del ion complejo o del gas formados por la correspondiente
reacción.

Escriba en la última línea de la Tabla los resultados finales


obtenidos en Base a sus experimentos.
- 107 -

Número de la
1 2 3 4 5 6

X
muestra
1

6 x
Fórmula del
compuesto
- 108 -

EJERCICIO N° 10 /experimental/ I a OIQ

Un compuesto sólido está en cada uno de los tubos de en sayo.


Puede ser cloruxo-,yoduro, óxido, hidróxido, sulfuro, sul fato o carbonato,
cada uno combinado con Tino de los siguientes cationes: Ag+, Pb2+, Cu2+, Cd2+,
Sb5+, Sn2+, Fe3+, Co2+, Ni2+ ' Mn2+, Cr3+, Al3+, Zn2+, Ba2+.

Problemas;

Escriba la fórmula química de cada uno de los compuestos sóli -dos dados como
muestra, Ud puede usar solamente los reactivos disponibles en su mesón
de laboratorio.

Atención! Al nacer las reacciones no use el total de la mués -tra. Una pequeña
cantidad de la muestra debe quedar en cada tu bo de ensayo. Solamente aquellos
resultados escritos en la Tabla respectiva serán evaluados.

Tabla:

C O M P U E S T O
N°de muestra Fórmula N°de muestra Fórmula
1 6

2 7

3 8

4 9

Tres muestras se encuentran en ampollas selladas. To -das ellas


son compuestos aromáticos: un hidrocarburo, un fenol y un aldehido.

Determine el grupo del compuesto al cual pertenece su mues_ tra


numerada, utilizando solamente los reactivos que están a su disposición.

Atención! Sea muy cuidadoso al abrir las ampollas. La


"identificación" de las muestras basadas sobre características físicas
/color, olor/ no es suficiente. Solamente aquellos re -sultados escritos en
la tabla serán tomados en cuenta.

N°de Tipo de
muestra Reactivo Observaciones compuesto
1
- 109 -

EJERCICIO N° 10 /experimental/ I a OIQ

3
- 110 -

EJERCICIO N° 12 /experimental/ 7 o OIQ

El carbonato de sodio cristalino pierde parte de su a -gua de


cristalización cuando está almacenado por largo tiempo,i.e. su contenido de
agua no es constante. Después de un prolongado almacenamiento tiene un
contenido promedio de agua de cristaliza ción.

Una solución de Na^CO^ está en el tubo de ensayo y lana sa de Na^CO^


. xf^O usada para su preparación está anotada sobre el tubo de ensayo.

Determine el contenido de agua de cristalización por mol de


carbonato de sodio de la muestra. Haga sus cálculos con una precisión de 0.01
mol.

Procedimiento:

Transfiera cuantitativamente la solución del tubo de en sayo a un


aforado de 100 cm3 y diluyalo hasta la marca con agua destilada libre de dióxido
de carbono /hervido y enfriado a temperatura ambiente/. Mide 10.00 cm3 de
la solución y transfxéra-
3 3 la a un Erlenmeyer de 100 cm y diluyalo
con 30 cm del agua des,
tilada arriba mencionada. Añada 2-3 gotas de anaranjado de meti_ lo y valórelo
con 0.1-molar HC1 hasta el color de transición del indicador. Hiérvalo por
1-2 minutos, el dióxido de carbono es expulsado de la solución. Si el color
de la solución cambia a amarillo, enfríe y valórelo de nuevo hasta el color
de transí -ción del indicador. Calcule el contenido de carbonato en la nues-
tra del consumo total de la solución de HC1 0.1-molar.

Ar/Na/ = 22.99; A /C/ = 12.01; Ar/0/ = 16.00; A^/H/ = 1.01

OCTAVA

OLIMPIADA INTERNACIONAL DE QUIMICA

HALLE 1976
REPUBLICA DEMOCRATICA ALEMANA
1. Explique por medios de símbolos internacionales generalmente usados y
fórmulas los compuestos denominados como peroxo compuestos. Escriba
las fórmulas resumidas para seis de ellos.

2. Escriba las ecuaciones químicas para dos métodos cuantitati -vos para la
determinación del contenido de peróxido en el per -óxido de calcio/II/s

3. Por medio de ecuaciones químicas exprese las reacciones quími cas


siguientes:

a/ Cr/H^O/g Cl^, disuelto en agua y mezclado con exceso de u-na solución


de hidróxido de sodio. Se forma una solución de color verde. El
color de la solución cambia a amarillo cuando se agrega una solución
acuosa de peróxido de hidrógeno.

b/ Si una solución acuosa de un compuesto de manganeso violeta se mezcla


con una solución de peróxido de hidrógeno, la solución se decolora y
se libera un gas de ella.

S O L U C I O N

2-
J.. Los compuestos peroxo contienen el grupo funcional: 0^

Ejemplos: H 0 ' Na2°2' Ba02' H2S05' H2S2°8' K2C2°6' Cr05'


2 2

vo2"3+

2. El peróxido de calcio/II/ se descompone por un ácido y H202 se libera, este


es determinado por:

a/ método manganométrico;

b/ método yodométrico.
- 112 -

EJERCICIO N° 2 8o OIQ
Ecuaciones:
a/ 5 H"202 + 2 MnO~ + 6 H30+ = 2 Mn2 + + 5 Oj + 1 4 H20

b/ H202 + 2 i" + 2 H30+ = I2 + 4 H20 I2 +

2 S202" = S402- + 2 I-

3. a/ {Cr/H20/6}3+ + 4 OH~ = ICr/OH/4/H20/2}" + 4 H"20

2ÍCr/OH/4/H20/2}~ + 3 H202 + 2 OH- = CrO2- + 12 HjO

B/ Ecuación dada en 2 a.
- 113 -

M A
Una muestra de 2,3793 g de un hidrato cristalino del ti_ po X V .
z 1^0, en el cual M es un metal, reaccionó con un exce so de SOCl^. Los productos
gaseosos formados por la reacción se introdujeron en una solución de cloruro
de Bario que contiene S-cido clorhídrico y peróxido de hidrógeno. Pequeñas
cantidades de SOCI2 fueron arrastradas por los productos gaseosos y removidas
por congelamiento. La masa del precipitado formado de la solu -ción fue de
J.4.004 g. Se encontró que contenía 13.74% en masa de azufre.

En otro experimento, J..1896 g de la sustancia inicial fue


3
disuelta en agua y la solución diluida a un -volumen de 100 cm .
3
Un quinto de esta solución se necesitó para reaccionar con lOcm de una solución
0,2-molar AgNO^. La masa del precipitado formado por la valoración fue
0,28664 g. /El punto final de la valora ción se determinó por un método
conductométrico/.

Problemas:

1. Calcular la fórmula abreviada del hidrato cristalino. /Utilice los -valores


de las masas atómicas relativas dadas en la Tabla Periódica de los Elementos
que se adjunta/.

2. Si Ud. sabe que la muestra puede contener un máximum de siete moles de agua
por un mol de hidrato cristalino, de un ejemplo de otro posible hidrato
en el cual debido a las limitaciones dadas no puede ser tomado en
consideración.

------------------------- S O L U C I O N -------------- : ----------

J., a/ El contenido de azufre confirma que el precipitado es BaSO^

Reacciones:

M A „ z H_0 + z S0C1, = z SO. + 2z HC1 + JA A


x y 2 2 2 x y
- 114 -

EJERCICIO N° 2 8o OIQ

2 S02 + z H202 + z Ba2+ = z BaS04 + 2z H+

m/BaSO./ 14.004 g
n/BaSO / = --------- — = -------------- -y = 0oQ6 mol
M/BaSO / 2 33.4 g.mol

Cantidad de agua en el hidrato:

n/H20/ = Q.06 mol

b/ Cantidad de sustancia A en la muestra: Reacción : Ag+ + A

= agua AgA n/Ag+/ = c.V = 0.2 mol.dm"3 . 0.01 dm3 = 0.002 mol

n/Ag.Y = Q.0.0 2 mol

„ z, m/AgA/ 0.2 8 6 6 4 g .-1


M/AgA/ = = n nn? ^ = 143.32 g.mol

m/AgA/ ______________
n/AGa/ ~ 0.002 mol

A = Cl

El precipitado formado por la titulación es AgCl, de modo que el hidrato es


un cloruro.

1-1^96 gf i.e. 0.23792 g de hidrato contiene 0.002 mol Cl" 2.3792 g del

hidrato contiene 0.02 mol Cl~„

La razón molar de Cl a H?0 en el hidrato:

n/Cl / : n/H20/ = 0.02 : 0.06 = 1 ; 3 Presunción:

i/ MCI . 3 H20

n/Cl / = 0.02 mol


n/MCl/ . 3 H20/ = 0,02 mol

M/MC1 . 3/H20/ = 0^ oz^ol = 118 °965 g ' mol"a M/M/ =

M/MC1/ , 3 H20/ - M/Cl/ - 3 M/HjO =

= 1180965 - 35 ,453 - 54.045 = 29.466 g.mol"1

Los elementos con masas molares similares son no-metales, por lo tanto
la primera presunción no es apropiada.

ii/ MC12 . 6 H20


- 115 -

n/Cl"/ = 0.02 mol

n/MCl2 „ 6H20/ = 0.Q1 mol

M/MC12 . 6 H20/ = lililí = 237.93 g.mol"1

M/M/ = M/MC12 . 6 H20/ - 2 M/Cl/ - 6 M/H20/ =

= 237.93 - 70.906 - 108.Q92 = 58.932 g.mol"1 M = Co

La segunda presunción satisface las condiciones del ejercí^ ció.

La fórmula del hidrato es CoCl, . 6 H,0.

iii/ MC13 . 9 H20

n/Cl"/ = 0.0 2 mol


0.02
—^— mol
n/MCl3 . 9 H20/ =
356.895 g.mol"1
M/MC13 , 9 H20/ = M/M/
H20/ - 3 M/Cl/ - 9 M/H.20/ =
= M/MC13 . 9
-116 -

EJERCICIO N° 3 8° OIQ

= 356.895 - 106.359 - 162.138 = 88.398 g.mol

M = Y

El hidrato YC1, „ 9 H,0, también como el otro hidrato SnCl.


3 2 ' 4
. 12 I^O, no pueden ser tomados en cuenta por las limita -ciones del
ejercicio /un máximum de 7 moles de H^O por una mol del hidrato/.
- 117 -

una muestra de 5 g de sulfuro de hierro/II/ técnico FeS, el cual


contiene 5% de hierre metálico reaccionó con ácido clorhídrico.

Problemas:

1. Calcular el volumen de los productos gaseosos en condiciones STP.

2. ¿Cuál es la composición /en volumen por %/ de la mezcla gaseo sa?

S O L U C I O N

1, Reacciones:

Fe + 2 HC1 = Fe Cl2 + H2

FeS + 2 HC1 = FeCl2 + H2S

03 .85 g.mol

^ - -$t§í - fi7 ?:75 g


,-i - 5 - 4 0 ■ i0 2 01
"-
' ' 87.91 g.mol V/H2/ = n/H / . VQ = 4.48

. 10~3mol . 22.4 dm3. mol-1

= 0 . 1 am3

y/H2S/ = n/H2S/ , VQ = 5.40 . 10~2 mol . 22.4 dm3 . mol 1


= 1.21 dm3
-118 -

EJERCICIO N° 3
8
° OIQ

2. Composición de la mezcla gaseosa: 3


°-3 d"4 „ 100 = 7.S3 % en volumen de H-1,31 da

1.21 dm^ . 10Q _ 92>37% en volumen de H2S


1.31 dm"

3Escriba las fórmulas estructurales de los compuestos A, B,


C, y O»
4Escriba las fórmulas estructurales de los compuestos A, B,
C, y O»
- 119 -

Cuatro sustancias que existen en la naturaleza tienen las


siguientes fórmulas abreviadas /empíricas/:

C2H5N02 /A/ C3H7N02 /C/

C3H7N02 /B/ CgH^NOj /D/

Cuando estas sustancias reaccionaron con una solución de un


hidróxido alcalino produjo sales alcalinas; en-una solución neutral o acida;
sin embargo, la reacción tiene lugar en los átomos de nitrógeno»

Problemas:

2„ ¿Cuál de las sustancias dadas es ópticamente activa y cuáles son inactivas?

3. Escriba la fórmula del grupo funcional la cual es típica de ciertas


substancias naturales y la cual también está contenida en el producto
formado por la reacción de dos moléculas del compuesto A. Dé el nombre
de la substancia natural a la cual pertenece el producto arriba mencionado.

4. "On producto orgánico cíclico, que es de importancia técnica , contiene


también el grupo funcional mencionado en el punto 3.

a/ escriba la fórmula estructural de este compuesto cíclico.

B/ Escriba la parte característica de la fórmula estructural de una


substancia macromolecular la cual puede ser obtenida desde este
compuesto mencionado.

c/ Escriba la parte característica de la fórmula estructural de un


producto isomérico macromolecular el cual es también

4.
- 120 -

EJERCICIO N' 4 8o OIQ


- 121 -

técnicamente importante,

d/ Dé el nombre para el grupo de compuestos al cual pertenecen ambas


substancias macromoleculares mencionadas.

S O L U C I O N

1. CH,-C00H l
(-)
A : 2 NH,

CH,-
COO
C+) NH.

(-)
CH3-CH-COOH
NH,
CH,CH-COO
3 I ( + )
NH3

CH.-CH -COOH I CH,-CH -COO


¿ ¿ NH
2 (+) NH.
2 2

D : CH,-CH-COOH I
NH,
CH-COO
2
Í
( + ) NH

A - ópticamente inactivo B -

ópticamente activo C -

ópticamente inactivo D -

ópticamente activo

3, -NH-CO- péptidos

//CH2 -CV
a/
CH.
:H^^
CO-NH
W -CO-/CH2/5-NH-
c/ -NH-/CH2/6-NH-CO-/CH2/4-CO-

4.
- 122 -

d/ poliamidas
EJERCICIO Nc 5 8o OIQ

a/ Los compuestos B y E se forman cuando una sustancia orgánica A /fórmula


abreviada CcH._0,/ reacciona con una solu ción de hidróxido de sodio
/saponificación/.

b/ La sustancia B puede oxidarse en un proceso de dos etapas pro duciendo


la sustancia C,

c/ La sustancia C cuando reacciona con Bromo produce un produc to de


sustitución D el cual puede ser hidrolizado por reacción con una
solución de hidróxido de sodio produciendo la sustancia E.

d/ La sustancia E cuando reacciona con una cantidad estequiomé trica de ácido


clorhídrico produce un compuesto F /40.0% C, 5.66 % H y el resto es
oxígeno/.

e/ La sustancia F es un producto importante de metabolismo en procesos


biológicos,

f/ El compuesto F en reposo se descompone dando un mol de agua por dos moles


de sustancia la cual resulta en la formación de un compuesto inestable
G,

g/ La sustancia 7 también puede obtenerse de un compuesto H que contiene


nitrógeno por una reacción de una etapa con ácido nitroso. Presumiendo
que la reacción tiene un curso cuanti tativo, 4.5 g de la sustancia F
se forman de 4.45 g de la sus tancia E.

Problemas; \

1. Escriba todas las ecuaciones para las reacciones químicas an

2. Dé las fórmulas estructurales de los compuestos orgánicos de A hasta H„

S O L U C I O N
- 123 -

CH,-CH,-CH,-0-CO-CH-CH-, + NaOH + CHo-CH.-CH--0H + CH_-CH-COONa


3 2 2 | 3 3 2 2 3 |
OH OH

B ^H°0 ■> CH3-CH2-CHO ------------>- CH3-CH2-COOH

C + Br, ------► CH,-CH-COOH + HBr


- 2 3 |
Br

D + 2 NaOH ------ + CH,-CH-COONa + NaBr + H,0


OH

E
~
+ HC1 ----- »■ i
\
■CH..-CH-C0OH
OH
•+ NaCl

2 F --------- >• CH,-CH-CO-0-CH-COOH + H-0


3 i | 2
OH CH3 G

CH,-CH-COOH + HNO, ---- * F + N + H O


3 I 2 — 2 ¿
NH2
H

4.
- 124 -

EJERCICIO N° 6 8o OIQ

En una habitación grande, la temperatura debiera medir se por medio


de un termómetro gaseoso. Para este propósito un tubo de vidrio con un volumen
interno de 80 cm3 se llenó con ni_ trógeno a una temperatura de 20°C y una
presión de 101.325 kPa. Seguidamente el tubo se movió lenta y constantemente
a través de la habitación. Debido a la expansión térmica, el gas a mayor tem
peratura escapa del tubo y es capturado sobre un líquido cuya pre sión de
vapor es despreciable. El volumen total del gas que es capa del tubo fue de
35 cm3 a una temperatura de 20 °C y una pre sión de 101.325 kPa.

Problemas:

.I. ¿Cuántas moles de nitrógeno se utilizaron para llenar el tu bo de


vidrio?

2. ¿Cuántas moles de nitrógeno escaparon del tubo a la temperatura más alta?

3. Calcular la temperatura promedio de la habitación bajo inves tigación si


la expansión térmica del tubo de vidrio es des -preciable.

4. Cambia la situación si en vez de nitrógeno puro se utiliza u na mezcla


que contiene 50% en volumen de nitrógeno y 50% en volumen de hidrógeno?

-------------------------- S O L U C I O N -------------------

1. Llenado del tubo:


p . V 101.325 kPa . 0.080 dm3
n /N / = --------i = ---------------------- -I-------------
R . T 8.314 J.mol . K . 293.15 K

3.3 3 . 10~3 mol

2. Escapado del tubo:

p . T2 101.325 kPa „ 0.035 dm3

£ £ R . T 8.314 J.mol . K . 293.15K

1.46 . 10~3 mol


- 125 -

Quedó en el tubo:

n3/N2/ = n^ - n2 = 1.87 . 10~3 mol

un volumen V^/ es la temperatura promedio de la habitación

3. La temperatura a la cual la cantidad de nitrógeno /n^/tiene un volumen


v^/ es la bajo investigación/:

p •V1 101.325 kPa . 0.080 dm3


R .n3 8.314 J.mol^.K-1. 1.87 . 10-3 mol

= 521 K t =

248°C

4. Ningún cambio puede ocurrir si se compara con la medida ante rior.


- 15 9 -

EJERCICIO N° 7 8o OIQ

La densidad de la solución de ácido sulfúrico en un a-


3
cumulador de plomo cargado debiera ser igual a p = 1.28 g.cm el cual
corresponde a una solución que contiene 35.87 % en masa de
H. SO.. En un acumulador de plomo descargado el valor de p no de
-3
biera decrecer bajo p = 1,1.0 g.cm que corresponde a una solución de 14035%
de ácido sulfúrico.

/Constante de Faraday F es igual a 26.8 Afi.mol X./ Problemas:

I. Escriba la ecuación para la reacción electroquímica total que


tiene lugar en el acumulador de plomo cuando carga y desear
ga.

2. Calcular las masas de HjO y HjSO^ que se consumen o forman de acuerdo


con la ecuación en N°l.

3. Calcular la masa de H2S04 que se necesita agregar a un acumulador de plomo


con una capacidad de 120 Ah si el contenido de HjSO^ está en el rango
dado en el ejercicio.

4. Calcular la diferencia en volumen de las soluciones de ácido sulfúrico


en un acumulador cargado y descargado con una capacidad de 12Q Ah.

------------------------- S O L U C I O N — -----------------------

Descargando
1» Pb02 + PB + 2 K2S04 + 2 PBS0 + 2 H2°
4

Cargando

n/H2S04/ = 2 mol n/H20/ = 2 mol

m/H2S04/ = 196 g m/H2/ = 36 g

Descarga

Carga
- 127 -

m/H2S04/ A =m/H2-S0196
4/ g= + 196 g

A m/H20/ A m/H^O/ = - 36

= + 36 g g

La masa de H2S04 requerida:

26.8 Ah. = 98 g H S04


2 120
Ah. = 438.8 g H2S04

Análogamente:

26.8 Ah = 18 g H20

120 Ah = 80.6 g H20 Acumulador de

plomo descargado: Masa de la solución de ácido

sulfúrico

Masa de H2S04 - m^
Masa de la fracción de H2S04 - =
0.1435
Densidad de la solución de H2S04 ~ pl= lol° SoC™3

Acumulador de plomo cargado:

Masa de H,SO. formado - m,= 438.8 g


2 4 2

Masa de H20 consumida - 80.6 g

Masa de la fracción de H2S04 - w2= 0.36 87

Densidad de la solución de H2S04 _ P 2 = 1.28 g.cm

Porque: = /a/

m
- 128 -

m. + m0
w2 = -i ----- ¿- /b/
m + n^-m^

Se obtiene un sistema de ecuaciones /a/ y /b/ el que resuel to para m^


y m:

= 195 .45 g

m = 1362 g

4, Volumen del electrolito en un acumulador descargado:

,, m _ 1362 g _
-,-,->□ „ 3
y.. = -------------- = ------------------------- 2——sr = 1238.2 cm
p
l 1.10 g.cm

Volumen del electrolito en un acumulador cargado:

m + m_ - m, . „on . ,
-,, 2 3 1720 .2 g ,
, . , „ 3
V2 = --------------- = ------- : - 2 -- ^ = 1343.9 cm

p 2 1. 2 8 g.cm

Diferencia en los volúmenes:


1 V = V2 - VI = 1343.9 - 1238.2 = 105.7 cm3
- 129 -

EJERCICIO N° 8-9-10 /experimental/ 8o OIQ

Ejercicio H° 8

+ 2+
Una muestra contiene Sos de los siguientes cationes: Ag , Pb , Fe2+, Cr3+, Co2+,
Al3+, Mn2+, y uno de los siguientes aniones : SO2-, ClNO3.

Pruebe los cationes y aniones en la muestra por medio de los si-


guientes reactivos: 2 N HC1, HjSO^ concentrado, 2 N H2S04, 2 N
HN03, 2 N CH3COOE, NaOH, NH^OH, Na2C03, KN03/Na2C03, NHjSCN,
Na2B407, Nal", C2H5OH, BaCl2, AgN03, NH4C1, /NH4/2Fe/S04/2 y alizarina S.

Escriba los resultados en la tabla adjunta de la siguiente mane-xa:

a/ en la columna "Reactivo"
escriba la fórmula del reactivo que se necesitó para probar cuando el catión
o el anión estaban presentes o ausentes en la muestra;

B/ en la columna "+/-"
marque la presencia o ausencia de un anión en la muestra por el signo +
y - respectivamente.

Ejercicio N"9

Una solución contiene oxalato de sodio y ácido oxálico.

Determine cuántos mg de oxalato de sodio y ácido oxálico están contenidos en


la solución que se investiga.

Las siguientes soluciones están a su disposición: KMn04

/c = 0.01972 mol.dm-3// NaOH/c = 0.1019 mol.dm-3/, H2S04 concen trado y solución


de fenolftaleína.
- 130 -

Ejercicio N° 10

Cuatro compuestos orgánicos alifáticos desconocidos de fórmula ge neral


A-CI^-B enumerados de 1 a 4, se dan como muestras. Algu -nos de ellos pueden
estar en una solución acuosa.

Ejecute los siguientes experimentos:

1. Determine el valor del pH de la solución.

2„ Haga reaccionar la muestra con ácido clorhídrico,

3. Reacción con hidróxido alcalino /hidrólisis básica - etralli -ción por 5


minutos Bajo reflujo/ y una prueba subsecuente de haluros.

También,están a su disposición los siguientes datos:

a/ Uno de los compuestos Bajo investigación forma un anhídrido intramolecular.

B/ El contenido de carbono e hidrógeno /en % de masa/ también co mo la masa


molecular relativa para el mismo compuesto se cono cen,se dan los ^valores,
sin embargo, en un orden arbitrario el cual no corresponde a la numeración
de las muestras.

Determine el grupo funcional A y B para cada sustancia, use los resultados


de sus experimentos y los datos disponibles.

Escriba sus resultados en la tabla adjunta; marque los resul tados


positivos con el signo "+" y los negativos con el signo
NOVENA

OLIMPIADA INTERNACIONAL DE QUIMICA

BRATISLAVA 1977

CHECOESLOVAQUIA
Campare tres sales de una composición M2^2°x' ^on<^e x representa
tres enteros pequeños y diferentes y M es un metal al calino. A cada una de
las tres sales aplique algunas de las si guientes aseveraciones:

a/ El enlace 0-0 es característico para el anión.

b/ El enlace S-S es característico para el anión.

c/ El enlace S-O-S es característico para el anión.

d/ Se forma por descomposición térmica del sulfato de hidrógeno

e/ Se forma por oxidación anódica del sulfato de hidrógeno.

f/ Se forma por la reacción de una solución acuosa de sulfito con azufre.

g/ Su solución acuosa disuelve el bromuro de plata.

h/ La neutralización de su solución acuosa con MOH produce el sulfato M2S04„

i/ En solución acuosa es capaz de oxidar una sal de Mn/II/a per manganato.

Problemas:

1. Llene la tabla con los valores correctos de x para las fórmu las dadas
e indique en el cuadrado correspondiente con las letras apropiadas
aquellas aseveraciones las cuales pueden ser aplicadas a cada una de las
sales dadas:

Tabla:
- 132 -

EJERCICIO N° 1 9o OIQ

H2S20

M2S20

M2S20
- 133 -

2. Escriba las fórmulas estructurales de los aniones de lastres sales dadas


y señale en ellas los enlaces 6-y 11.

3. Escriba las ecuaciones químicas que expresan los procesos in volucrados


en las aseveraciones usadas en las letras d,e,f, 9, h, i.

S O L U C I O N

M2S2°3 B f g
M2S2°7 c a h
M2S2°8 a e i

2. 0 o
i! I!
0 — "S—o -S"-^0
ll
li
o o 0 0"
S2°7

~
S2°8~

+ S2°7~
a/ 2 HSO. = H20
2 MHSO, H20 + M2S207

e/ 2 HS0~ - 2 e = 2 H+ + S202-2 H+ +

2 MHSO, 2e = M2S20g

f/ SO 2-
+ S + S S2°3~
M2
S03 M2S2°3

+ 2 M2S203 =
g/ AgBr + 2 S203 {Ag/S203/2}J
+ Br = M3
{Ag/S203/2> + MBr
AgBr
h/ S202 + 2 OH = 2 SO2 + H20

M2S207 + 2 MOH = 2 M2S04 + H20

i/ 2 Mn2+ + 5 S2Og" + 8 H20 = 2 MnO~ + 10. S02~ + 16 H+

2 MnSO. + 5 M,S,0o + 8 H,0 = 2 MMnO.+ 4 M,SO.+8 HoS0.


4 2 2 8 2 4 2 4 2 4
- 134 -

A t e n c i ó n

Asegúrese de abrir solamente los


sobres correctos. Ud. pierde puntos por abrir
los sobres incorrectos. Devuelva
los sobres sin abrir junto con su
solución.

Un halógeno X reacciona con una solución acuosa de otro compuesto


halogenado KYO^ de acuerdo a la ecuación:

X2 + 2 KY03 + 2 KX03 + Y2

1. El número atómico del halógeno X es mayor que el del halógeno Y. /Si Ud.
encuentra que esta respuesta es correcta abra el sobre 1./

2. El número atómico del halógeno X es menor que el del halógeno Y. /Si Ud.
encuentra esta aseveración correcta abra el sobre 2./

Escoja la respuesta correcta 1 ó 2, abra el sobre correcto y continúe en


la resolución de acuerdo al texto que trae el sobre abierto.

Texto en el sobre 1:

Su respuesta es correcta. Continúe.

El compuesto KX03 se oxida en solución alcalina por el halógeno


Y formando un compuesto KX04 de 3/VII/ donde el halógeno Y se reduce a haluro
KY:

KX03 + 2 KOH + Y2 = KX04 + 2 KY


+ H20
- 135 -

EJERCICIO N° 2 a 9o OIQ

Una solución acuosa de haluro de potasio KY produce con AgNO-j


en solución un precipitado blanco de AgY insoluble en agua pero rápidamente
soluble en una solución acuosa de amoníaco.

3. Halógeno Y es flúor /sobre 3/„ 4o

Halógeno Y es cloro /sobre 4/. 5.

Halógeno Y es bromo /sobre 5/.

Escoja la respuesta correcta 3, 4 6 5, abra el sobre


correspondiente y continúe de acuerdo con las instrucciones dentro del sobre.

Texto en el sobre 2:
Su respuesta es correcta. Abra el sobre 1.

Texto en el sobre 3:

Su respuesta es incorrecta. Escoja respuesta 4 ó 5

Texto en el sobre 4:

Su respuesta es correcta. Escoja la respuesta correcta final


indicando la alternativa correcta 6 ó 7.

6. X = Br; KXC> = KBrO.


4 4
7. X = I; KX04 = KI04

Texto en el sobre 5:

Su respuesta es incorrecta. Escoja respuesta 3 6 4.

— ----------------------- S O L U C I O N ---------------------------

Las respuestas correctas son las siguientes: 1, 4 y 7.

Nota: El Jurado Internacional no eligió el Ejercicio N° 2a para la 9° OIQ/


Olimpiada Internacional de Química/ pero sí el Ejercicio N° 2b.
EJERCICIO N° 2b - 136 - 9o OIQ

La reacción de iones permanganatos con peróxido de hidró geno en


solución acida produce una sal de Mn/II/ y al mismo tiem po libera oxígeno:

2 MnO~ + 1 H202 + 6 H+ = 2 Mn2+ + 3 02 + 4 HjO

2 Mn04 + 3 H202 + 2 Mn2+ + 4 02 + 6 H20


6 H

2 MnO~ + 5 H202 + 6 H+ = 2 Mn2+ + 5 02 + 8 H20 2 MnO~ + 7 H_202

+ 6 H+ = 2 Mn2+ + 6 02 +10 HjO

Problemas:

J.. Las posibles razones de los reactantes en las ecuaciones arri_ ba escritas
expresan:

a/ todas las ecuaciones Indique la


aseveración
o/ solamente algunas de las ecuaciones correcta por
una cruz en el
cuadrado y
c/ solamente una ecuación explique su
decisión.
d/ ninguna

2. ¿Cuál de los reactivos es un agente oxidante y cuál es un reductor?

3. ¿Cuántos gramos de permanganato de potasio se necesitan para liberar 112


cm de oxígeno a STP de un exceso de peróxido de hidrógeno en solución acida?

S O L U C I O N

-1, Correcto es : C
- 137 -

,» VII L ,.11
Mn + 5 ce = Mn

o;" - 2 e -

, M VII. - _-II - M II , ,
2 Mn + 5 C>2 = 2 Mn
+ 5

Agentes oxidantes: Mn04, 6 MnVI1 Agentes

reductores: ñ^Q^f 6 ^2^ ^ ^

V/02/ = 112 cm3

n/0,/ = ----- °°112, dm3 = 0.005 mol


22.4 d m . mol

n/KMn04/ = Q.0Q5mol . 2/5 = 0.002 mol n/KMnO./ =

0.316 g
- 138 -

EJERCICIO N° 3 OIQ

Las letras A, B, C, D, y E representan isómeros de los ácidos


dicarboxílicos del ciclo butano, siendo uno de ellos la forma racénica.
Se-mostró que:

a/ solamente el compuesto C forma fácilmente un anhídrido cíclico;

b/ B produce un anhídrido cíclico sólo a altas temperaturas;

c/ de todos los ácidos Bajo investigación, solamente A libera dióxi do Se carbono


cuando es calentado;

3/ D y E no sufren cambios a temperaturas altas;

e/ 2 moles de dietil ester del ácido malónico cuando reacciona con etóxido de
sodio /EtONa/, se transforman en una sal sódica la cual por reacción con
yoduro de metileno /CR^I^/ produce un tetraester (^¡.H^Og. Este nuevo
ester da otro tetraester C^g H^^Og en la reacción con dos moles de etóxido
de sodio y un mol de yoduro de metileno» El último de los tetraester
mencionado se transforma por hidrólisis alcalina y subsecuente acidifica-
ción en un ácido tetracarboxílico el cual cuando se calienta da una mezcla
de B y E.

Problemas;

1. Dé las fórmulas correspondientes a las letras A, B, C, D y E.

Marque el grupo que está sobre el plano del ciclo con ■^t , y

Bajo el plano con . Por ejemplo:

2. Exprese los procesos a/, b/ y c/ por ecuaciones químicas.


- 139 -

S O L U C I O N

COOH COOH *"^COOH COOH

TIOOH TCOOH

B C
COOH A*COOH ^OOH

y^/COOH COOH >•,,,„


COOH

1Ll'
O
^COOH n

-
2. a/

O + Ho0 / 2

^ COOH

COOH
b/

O + H,0

COOH

^COOH
COOH .COOH
C/
+ CO.

A
- 140 -

EJERCICIO N° 4a 9o OIQ

Los compuestos A y B tienen la misma fórmula abreviada CyH-^Og, pero


diferentes propiedades físicas/e.g. punto de fusión, rotación óptica
específica/, pertenecen al grupo de los sacári -dos los que contienen
heterociclos de seis -miembros. Cuando una solución al 1% de ácido
sulfúrico se le agrega al compuesto A y B respectivamente, se obtiene un mismo
compuesto que contiene 40.0% de C y 6.71% de H por ebullición. Después de
reducir el compues_ to C / por ejemplo catalíticamente con hidrógeno o con
boro hi -druro de sodio Naj^BH^/se obtiene un producto cristalino D el cual
aislado no redujo al reactivo de Fehling y ni mostró actividad óptica. El
compuesto C fue oxidado con un agente reductor suave/ e.g. solución de
hipobromito de sodio en frío/ produciendo una sal del ácido polihidróxido
monocarboxílico de configuración D /dextro/.

Problemas:

1. Sugiera las estructuras de los compuestos A, B, C y D.

2. Si Ud. no encuentra que el ejercicio no es ambiguo, explique

porqué:

S O L U C I O
N

OCH- OCH

H OH
HO H O H
HO

H• CH20H
CH 2 OR

A /ó B/ B /ó A/
- 141 -

EJERCICIO N° 4b

CHO
H OH H OH

HO H HO H

HO H HO H.

H OH H OH

C D

Hay otra solución similar en la serie de las

D-alosas. Nota:

El Jurado Internacional no escogió el Ejercicio N°4á para la 9"Olimpiada

Internacional de Química,pero sí la alternativa del Ejercicio N° 4b.


- 142 -

Un ester ópticamente activo /11G6 g/ que tiene la fórmu


la abreviada C.H,,0, fue hidrolizada en caliente con -un exceso de o x ¿ ¿
una solución de nidróxido de sodio. Después de terminada la hidrólisis la
-mezcla de reacción alcalina fue extraída varias ve -ees con éter. La solución
acuosa no es ópticamente activa. Las porciones de extracto etéreo fueron
secadas con sulfato anhidro. La solución etérea fue filtrada, el éter destilado
de ella, y el residuo fue redestilado.704 g/100%/ de un líquido de punto de
e-Bullición de 1QQ grados C se obtuvieron.

Problemas:

1. Escriba la fórmula estructural del ester.

2. ¿Cuál sería la estructura de un ester con idéntica fórmula a Breviada,


y.e. C^ñ^O^t si la solución acuosa obtenida después de la hidrólisis
alcalina en la forma arriba mencionada era ópticamente activa?

3. Escriba las ecuaciones para la hidrólisis alcalina de ambos esteres con


una solución de hidróxido de sodio.

S O L U C I O N

1. 0 CH.,
it I 3
Cff3-C-0-C-CH2-CH3
- 143 -

EJERCICIO N° 4b 9o OIQ

7*3

CH3 ---- CH2

O CH,
¡I I 3 fH3 CH3C-0-C-CH2CH3 ^OH_ CH.COoW +
CH^CH^OH

f3 f3
CH3CH2-C-COOCH NA0H E> CH^CH.CH-COO-Na+ + CH,OH
| *
* 3
H
- 144 -

EJERCICIO N° 5 9° OIQ

Dos sales de cobre /I/ de los ácidos orgánicos HA y HB, ligeramente


solubles en agua, forman una solución saturada en un buffer de pH dado.

Problema:

J.. ¿Cuál será la concentración de los cationes Cu+ en la solu -ción si el


producto de solubilidad de las dos sales son Kg /CuA/ y Ks/CuB/ y las
constantes de ionización de los ácidos son k /HA/ y K,/HB/?

------------------------- S O L U C I O N ---------------------------

1. Las ecuaciones para las cantidades totales de las sustancias de las


partículas A, B y Cu son como sigue:

a = n/A-/ + n/HA/ + n/CuA/

b = n/B-/ + n/HB/ + n/CuB/

m = n/CÜ+/ + n/CuA/ + n/CuB/

Las cantidades de precipitados se elimina de la ecuación:

a + b - -m = n/A-/ + n/HA/ + n/B-/ + n/HB/ - n/Cu+/ = 0

porque, cuando se forma un sistema de dos sales sólidas, el número total


de partículas A y B /a + b/ debe ser igual al número total de cationes
Cu+, i.e. al valor de m.

Cuando las cantidades de sustancias de dividen por el volu -men de la


solución, se obtienen las concentraciones, y así:
- 145 -

EJERCICIO N° 6 9° OIQ

ÍA~} + tHA) + {B-} + {HB} =


/a/
{Cu+)

Ks/CuA/
Kg/CuA/ = [Cu HA~J => {A } /b/
lCu+}

Kg/CuB/ = (Cu+}{B~) => (B~) Kg/CuB/ /c/


{Cu+}

ÍH+}{A~} ÍH+} {A~>


Kg/HA/ /a/
ÍHA) K /HA/

=> [HA]
=

C H + K B ~} {H+}
Kg/HB/ /e/
{ HB } {B")

K2/HB/
=> {HBÍ

Sustituyendo /d/ y /e/ en /a/ ÍH+ J

\ [H+}
fCu } = {A ) 1 + + (B ) /f/
K /HA/ 1 + K /HB/

Sustituyendo /b/ y /c/ en /f/

Kg/CuA/ rn+} Ks/CuB/


tCu+] 1 + 1 +
(Cu+} K /HA/, {Cu+} Ka/HB//

/g/

fCu+} = ÍH+) {H+3


Kg/CuA/ 1 + K /CuB/
+ Ka/HA/, 1 + K /HB/
a
- 146 -

Los amino ácidos pueden ser determinados por la cantidad de


nitrógeno liberado en su reacción con ácido nitroso, /mé todo de Van Slyke/,
por ejemplo:

CH3CHNH2COOH + HN02 = Cf^CHOHCOOH + H20 + N2

Otro método consiste en la valoración de amino ácidos con una


solución volumétrica de ácido perclórico en ácido acéti_ co glacial:

CH,CHNH_COOH + HC10„ = CH_.CHNH,COOH+ + C10~


3 2 4 3 3 4

El exceso de ácido perclórico se determina por valoración con una


solución volumétrica de acetato de sodio, /prepara da en un solvente
no-acuoso/.

50.0 cm3 de una solución 0o100-normal de ácido perclórico fueron


agregados a una muestra de glicina NH CH_COOH,el ex-
3
ceso de ácido perclórico fue retrovalorado con 1.6 cm de una solución
0.15-normal de acetato de sodio utilizando el viraje del metil violeta como
punto final.

S O L U C I O N

n/HC104/ = V.c = 0.05 dm3 . 0.100 mol.dm 3


= 0.005 mol
3 "3 n/NaAc/ = 0.Q16 dm . 0.15
mol.dm = 0.00 2 4 mol

Gastados en la reacción:

n/HC104/ = Q.005 mol - 0.0024 mol = 0.0Q26 mol V/0.1 N HC104/

= 0 .026 clm3
- 147 -

EJERCICIO N° 6 9°
OIQ

Porque:

n/HC104/ = n/glycine/ = n/N / = 0.026

mol

entonces:

V/N / = n.R.T _ 0.026 mol . 8.314 J.mol"1. K"1 . 293.1


K
7
27 p 120.658 kPa

= 0..6J.7 dm3
- 148 -

EJERCICIO N° 7 9° OIQ

La fotosíntesis puede resumirse por la ecuación total:

6 C02/g/ + 6 H20/ cl^fila > CgH1206/s/ + 6 0,,/g/

para la cual los valores de AH y AS a 25°C son los siguientes:

AH = 2.816 . 166J; AS = -182 J.K-1 Ó AH = 2.816 . 166J.mol-1,

AS = -182 J.K'^.mol-1 si AH y AS están relacionados a un cam -bio de reacción

de un mol.

Imagine que han sido inventados unos alectrodos que per mitirán una
reducción selectiva de oxígeno a agua y la oxidación de la glucosa a dióxido
de carbono en una celda galvánica, i.e. un proceso reverso al compararlo con
el de la fotosíntesis.

Problemas;

1. ¿Cuál será la fuerza electromotriz de la celda en la cual la energía


luminosa seria transformada en energía eléctrica por medio de la reacción
de fotosíntesis?

Nota: En el sobre Ud. puede encontrar la relación entre la fuerza


electromotriz y la entalpia libre de la reac -ción. /Atención!
Si Ud. abre el sobre pierde algunos puntos./

2. En caso que se desee cuantificar el símbolo "luz" en la ecua ción de


fotosíntesis, se preguntaría: ¿cuántos moles de foto nes con una longitud
de onda por ejemplo de 500 nm toman par te en la reacción arriba dada?
Calcular.

3. ¿Cuál sería la potencia eléctrica de una piscina de natación cuadrada con


un lado de 10 m de longitud, que contiene algas
- 149 -

verdes capaces de reaccionar fotosintéticamente, si bajo ilu minación se


espera una corriente de 1 mA de una área de lcm ?

S O L U C I O N

1. Dos soluciones alternativas:

a/ por medio de las cantidades b/ por medio de las canti dades


relacionadas a un cambio de relacionadas por la
reacción de un mol. Para que la reacción dada: Para que la
reacción tenga lugar en la celda reacción ten ga lugar en la
ccrrrespon derfa: celda correspondería:

A G = -2.87 . ia6J.mol_1 A G = -2.87 . 106J.mol"

Relación entre la fuerza electro motriz


y el cambio de la ental -pía libre de
la reacción que ocu rre en la celda: -A
G = n.F.E.

donde n es el así llamado donde n es el número de


numero de carga. moles de carga que pasa
ron a través del elec -
trodo durante la reac -
ción.

En nuestro caso, n tiene un va -lor


igual a 24, dado que una molécula de
oxígeno se reduce de a cuerdo a la
ecuación:
0 + 4 H + + 4 e =
2 H_0 2 2
F/Constante de Faraday/ = 96487 C.mol-1

Dado que un voltcoulomb en un Joule, en tonces:


„ _ -/-2.87.1Q
6/V. C ,mol 1 _ _ -/-2.87,10
6/V.C
i - ------------------------------------------------------------------------------ — -------- t, - ----------------------------------------------------- -

24 . 96487 C.mol 24 mol.9 6 4 8 7 C.mol

= 1.24 V = lc24 V

2. La energía absorbida por los fotones es la única fuente de e-nergía que


permite el curso de la fotosíntesis, por lo tanto el numero x de fotones
cargados multiplicados por su energía debe ser igual al aumento de energía
del sistema, i.e. al valor de 2.87 . 106 J.

Así: x. ñ o Y o N, = x„h. N, = 2.87 . 1Q


6J
- 150 -

A A A

2,87 . 106 J . X h,C. N A

___________________________ 2„87 o 106 J„ 5Q0ol0 ~9m


_____________ :
6.6256 . lQ ~34J.s. 2 . 9 9 7 9 . 1 0 m.s"1.6 . 6 2 2 . 1 0 23mol-1
8

= 11.99 y 12 mol de fotones

2
3. El área de la piscina de natación es 100 cm . La densidad de
4 -2

corriente a un voltaje de 1.24 "V es igual a 1 mA „ 10 m = -2

= 10 A.m

El total de la potencia eléctrica es: 1.24 V .

10 A.m-2 = 1.24 kW
- 151 -

En la mitad del siglo XIX un químico determinó la masa molar del


nueyo elemento X con el método siguiente dado que no e xistían otros:

El logró preparar cuatro compuestos A, B, C, y D que con tenían el


elemento X y determinó su contenido /en % de masa/en ca da compuesto. A 250°C
los cuatro compuestos estaban en estado gaseoso y fueron individualmente
transferidos a recipientes previa mentes evacuados hasta que la presión alcanzó
el valor de 1.013 . 10^ Pa, seguidamente los recipientes fueron pesados. Después
de restar el peso del recipiente vacío, se determinó la masa del gas. Este
procedimiento fue repetido con nitrógeno. Así se obtuvo la siguiente tabla de
datos:

Gas Masa total de gas Contenido del elemento X en el gas


/% masa/.
N2 0.652 g

A Q.849 g 97.3
B 2.398 g 68.9
C 4.851 g 85.1
D 3.583 g 92.2

Problema :

1. Determine la masa molar probable del elemento X.


n/N_/ 0.652 a
n/N./ = ----- — = ------------ '—^ = 0.0233 mol
M/N2/ 23 g.mol

A la temperatura de 250°C, se considera que las sustancias A, B, C y D se


comportan como gases ideales de acuerdo a la ley de Avo gadro:

n/N2/ = n/A/ = n/B/ = n/C/ =

n/D/ m/A/
M/A/ = --------
n/A/

n/B/
M/B/ = --------
n/B/

La masa del elemento X en un mol de A, B, C ó D:

A: M/A/ . 0 .973 = 35.45 , -1


g.mol
B: M/B/ . Q .689 = 70.91 .-1
g.mol
C: M/C/ . 0 .851 = 177.17 g.mol
D: M/D/ . 0 .922 = 141.78 g.mol
- 152 -

EJERCICIO S8 8a 9o OIQ

Dado que en una molécula de un compuesto debe haber por lo menos un átomo de
X o un múltiplo entero, debemos calcular la masa común más alta de las masas
molares obtenidas. Es en nuestro caso igual al promedio 35.44 g.mol" , el cual
puede ser considerado como la masa molar probable del elemento X. es solamente
el más pro bable dado que su fracción entera no puede ser excluida.

Nota :

El Jurado Internacional no eligió el Ejercicio N°8a para la 9° OIQ/Olimpiada


Internacional de Química/, pero sí la alternativa Ejercicio N°8b.
- 153 -

EJERCICIO N° 8b 9o OIQ

Entre otros factores, la deteriorización del medio am -biente se


manifiesta por la contaminación del aire con monóxido de carbono. Sus más
poderosas fuentes son las máquinas de con -bustión. La toxicidad del monóxido
de carbono es causada por el ñecho que forma con el pigmento-hemoglobina de
sangre, el compuesto carbonil-hemoglobina /HbCO/:

Hb + CO = HbCO

El enlace químico en la carbonil-hemoglobina es alrededor de 200


veces mas fuerte que en la oxi-hemoglobina /Hb02/ ori_ ginada bajo condiciones
comunes. Consecuentemente, la hemoglobi_ na no puede usarse como transportador
de oxígeno. La falta de oxíge no comienza a sentirse desde 50 ppm de monóxido
de carbono en el aire, i.e. 10% de carbonil-hemoglobina en la sangre.

Bajo condiciones comunes el oxígeno del aire se disuelve en la


sangre de los pulmones y su concentración se mantiene a 1.6 . 10 ^ mol . dm
3
por la respiración. La concentración de he
moglobina en la sangre de los capilares pulmonares es también cons
—6 — 3
tante - 8 .10 mol.dm

Problemas:

1. Calcular la velocidad de formación de la oxi-hemoglobina si la


6 3 —1 —1
constante de formación es k = 2.1 . 10 dm . mol . s / a
37°C temperatura normal del cuerpo/.

2. En algunos casos /envenenamiento con monóxido de carbono/ se


necesita un aumento en la velocidad de formación de la hemo -
-4 3 -1 -1
globina de 1.1 . 10 dm . mol . s

a./ Presumiendo que la concentración de hemoglobina en la sangre es


constante, calcular la concentración requerida de
- 154 -

EJERCICIO N° 9 /experimental/ 9° OIQ

oxígeno.

b/ Sugiera una solución práctica sobre la presunción que la concentración


de oxígeno en la sangre es proporcional a la presión Sel oxígeno que
entra en los pulmones.

S O L U C I O N

1. v = k . [Hb] . 2.1 1Q6 dm3 . mol 2

[02"|
tHbJ 10 mol
Co2) 1.6 . 10~6 mol.om"3
3
v = 2.688 10 ^ mo 1. dm

Dado que 1 mol de oxígeno /O^/ se necesita para formar un mol de


oxi-hemoglobina, la velocidad de consumo de oxígeno es la misma que la
velocidad de formación de la oxi-hemoglobina.

4 , „ -3 -1
______________ xr = i i i[) ---------------------- —~~~
k.JHbJ

2. [~02j = —V~-r v = 1.1 . 10 "* mol.dm J . s

k = 2.1 . 106 dm3 . mol"1 . s"1 [Hb")

= 8 . 10"6 mol.dm"3 fo2] = 6.5 . 10~6

mol.dm"3

La concentración de oxígeno debe aumentar a 6.5 . 1 0 mol.dm

La concentración de oxígeno puede ser afectada sólo parcialmente por el


aumento de la presión del aire. Un aumento de cuatro veces la
concentración de oxígeno demandaría un aumen to de cuatro veces la presión
del aire con respecto al valor normal. Esta presión sería muy peligrosa
para los organis -mos vivientes, por lo tanto se debe respirar aire
enriquecido con oxígeno.
- 155 -

Ud.seguirá el cambio de la concentración de uno de los reactivos


por el método colorimétrico visual comparativo. De los datos obtenidos
experimentalmente grafique el cambio de la concentración del reactivo en
función del tiempo.

Procedimiento:

1. Haga una escala colorimétrica comparativa de una solución de bromo.


En diez tubos de ensayo idénticos mida con una jeringa las si_
guientes cantidades de agua de bromo /O,01-molar/: en el pri_
3 3
mero - 10.0 cm ; en el segundo - 9.0 cm ; ................. en el no
3 3 veno - 2.0 cm y en el décimo - 1.0 cm . Seguidamente
agregue a todos los tubos de ensayo /excepto al primero/ agua des_
3
tilada hasta alcanzar un volumen de 10.0 cm en cada caso. Cié rre los
tubos con tapones de goma y mezcle la solución. Ponga los tubos de ensayo
en una gradilla contra un fondo blanco. Finalmente calcule la
concentración /en mol.dm "V de la solución de bromo en todos los tubos
de ensayo.

2. Realice la reacción mezclando 100.0 cm3 de la solución de bro mo con l.Ocm3


de Tina solución 1.00-molar de ácido fórmico. I n mediatamente después de
mezclarlas, transfiera 10.0 cm a un tubo de ensayo idéntico a los de la
escala colorimétrica. Com parando el tono del color de la mezcla de reacción
/a un minu to de intervalo/ con el color de las soluciones de la esca -la,
investigue el cambio de la concentración del bromo en fun ción del tiempo.
Ponga los datos en una tabla que tiene: tiempo /t/ y concentración de [Br}.
- 156 -

EJERCICIO N° 9 /experimental/ 9° OIQ

Ejercicio:

Grafique la concentración de bromo en función del tiem

po y lea en el diagrama el tiempo de vida media de la reacción. Preguntas:

1. Escriba una ecuación para la reacción de bromo con ácido fór mico
presumiendo que los reactivos están en cantidades este-quiométricas.

2. En química analítica una solución volumétrica de bromo puede prepararse


disolviendo una mezcla de bromo y bromato en medio ácido. Explique este
modo de preparación por medio de una ecuación química.

S O L U C I O N

Preguntas:

1. HCOOH/aq/ + Br2/aq/ C02/g/ + 2H+/aq/ + 2Br"/aq/

2. 3 + 5Br + 6 3 Br2 + 3 H20


BrO. H
- 157 -

EJERCICIO N° 10 9° OIQ

Por titulación termométrica de una solución de hipoclo rito con


una solución de propanona Ud. encontrará las cantida -des equivalentes de
los reactivos y consecuentemente los produc tos de la reacción.

Procedimiento:

Para la reacción de propanona con hipoclorito use las soluciones


temperadas a la temperatura del laboratorio /Comprue be/. Ponga lOO.Ocm^de
una solución de hipoclorito en una botella termo, introduzca un termómetro
y agregue continuamente la solución 4-molar de propanona en dosis de
1.Ocm^medidos en la bureta, agitando continuamente con el termómetro
/cuidadosamente, no lo quiebre/. Agite la mezcla de reacción completamente
después de cada adición y lea la temperatura más alta alcanzada. Continué
agregando la solución de propanona mientras la tempera tura siga subiendo.
Entonces agregue tres porciones más y fina lice el experimento. Guarde la
mezcla de reacción para un posi_ Ble uso futuro.

Problemas:

1. Dibuje una curva de valoración de los datos de cambio de tem peratura y


el gasto de la solución de propanona. Exprese las cantidades equivalentes
de los reactivos en moles.

2. Escriba una ecuación para la reacción y denomine el producto formado.

3. Sugiera un método de cálculo para el valor aproximado del ca lor de reacción


de los datos obtenidos.

4. Considere un procedimiento para aislar el producto de la reac ción y de


el método para su identificación.
- 158 -

5. La concentración exacta de una solución de hipoclorito puede ser determinada


también -midiendo el -volumen liberado de oxígeno después de la
descomposición catalítica del hipoclorito. Ilustre el principio de este
método por medio de una ecuación química y muestre esquemáticamente el
procedimiento de cálculo.

— ----------------------- S O L U C I O N --------------------------

CH^-CO-CILj + 3 C10~ = CHC1 + 2 0H~ + CH3COO_


3

Cloroformo

3. Cálculo del calor liberado en el curso de la reacción

Q = m.c.At

calor de reacción; •masa de

la solución;

capacidad calorífica de las soluciones que toman parte en la


reacción;

diferencia de temperatura /elevación de la tempe ratura/.


Sobre la base de los datos obtenidos en el ejercicio es
posible calcular el valor de Q por un mol de reactivo.

4. La mezcla contiene:

Reactivo - CH-jCOCH^ /exceso alrededor de 3 cm3 de la solu -

ción 4-molar/

Productos - CHC13, 0H~, CH-jCOO-

La acetona y el cloroformo se separan de las otras sustancias de la solución


acuosa por medio de un embudo de decantación seguido de una destilación.
CLO

mol -* 1 mol = 22„4 dm


- 159 -

EJERCICIO N° 11 /experimental/ 9o OIQ

Si un exceso conocido de una solución de hidróxido de concentración


conocida se agrega a una muestra pesada de una sal de amonio y el amoníaco
liberado se remueve por ebullición,es posible determinar la cantidad de
hidróxido sin reaccionar por titulación con una solución de un ácido.

Procedimiento:

Tres muestras de sal de amonio, pesadas con una precisión de Q.QCL1


g, están a su disposición. Introduzca cada una de
3 3 ellas en un Erlenmeyer de 250 cm
de volumen. Agregue 50.0 cm
de una solución 0.2-normal de hidróxido de sodio a cada muestra.
Ponga varios trocitos de porcelana para ayudar a la ebullición
en cada una de las mezclas y caliente suavemente los erlenmeyer
sobre una llama pequeña hasta que no se detecte amoníaco en los
vapores liberados. Después de la expulsión del amoníaco, enfríe
la solución a la temperatura del laboratorio, añada 2-3 gotas de
la solución del indicador /azul de bromotimol/ y titule con una
solución 0.2-normal de ácido oxálico hasta el primer color amari_
lio permanente de la solución.

Problemas:

1. Calcular la masa molar de la sal de amonio de los datos exper mentales.

2. La muestra es una sal de un ácido inorgánico monobásico. Considere cual.

3. Calcule el error absoluto y relativo de su determinación.

4. De la» reacciones por medio de las cuales pueden probarse los iones de la
sal en la solución.

DECIMA
OLIMPIADA INTERNACIONAL DE QUIMICA

TORUN 1978

POLONIA
- 161 -

EJERCICIO N° 1 10° OIQ

a/ ün -mineral de cromo que no contiene agua, compuesto de: Fe/Cr02/2,

Mg/Cr02/2, MgC03 y CaSi03.

b/ Se encontró por análisis que elmineral contiene 45.6% de Cr203, 7.98% de


Fe203 y 16.12% de MgO.

c/ Cuando el mineral fue tratado con ácido clorhídrico concentrado, los


compuestos de cromo presentes en el mineral no reaccionaron con el ácido,

d/ Finalizada la reacción, el mineral fue lavado completamente con agua


/hasta que la reacción con Cl fue negativa/ y el residuo sólido s-e secó
a masa constante.

Problemas;

1. Escriba las ecuaciones estequiométricas y iónicas para las reac clones que
ocurrieron cuando el mineral fue tratado con ácido clorhídrico según c/.

2. Calcular:

- El contenido de los componentes /en % masa/ presentes en el mineral;

- Cantidades de las sustancias presentes en el mineral.

3. Calcular el contenido de c ^ 2 ° 3 /en % masa/ en el residuo seco obtenido según


d/.

4. Un tubo de vidrio fue llenado con una cantidad suficiente de CaO granulado.
La masa total del tubo lleno fue de 412.02 g.
- 162 -

El gas formado por la reacción según c/, fue secado y pasado a través del
tubo de vidrio,

Calcular la masa del tubo de vidrio con su llenado después1 que la


reacción terminó„

Masas atómicas relativas: A^/Cr/ = 52.01; A^/Fe/ = 55.85;

Ar/Mg/ = 24.32; Ar/Ca/ = 40.08; Ar/Si/ =

28.09; A /C/ = 12.01; A /O/ = 16.00

-------------------------- S O L U C I O N ------------------------
= M CL. + CO- + H-0
g 2 2 2
1. MgC03 + 2 HC1
= Mg2+ + C02 + H20

MgC03 + 2 H+
CaCl2 + Si02+ H20

CaSi03+ 2 HC1 = Ca2+ + Si02+ H O

CaSi03+ 2 H+

2. La cantidad total de hierro está en la forma de Fe/Cr02/2. Por lo tanto:

Fe203 = 2 Fe/Cr02/2

Mr/Fe03/ = 159 .70 % Fe203 = 7.98

Mr/Fe/Cr02/2/ = 223,87

% Fe/CrO / = 2 ' 223°87 . 7.98 = 22.37


-------- —- 159.79

La diferencia entre la cantidad total de Cr203 y que está con tenida en


Fe/Cr02/2 corresponde a la cantidad de Cr203, que está en la forma de
Mg/CrO,/_.
% Cr203 en Fe/Cr02/2:

Fe/Cr02/2 = Cr203

Mr : 2 2 3.87 15 2.0 2

% Cr2°3 = ÍT37 • 22°37 = 15-19

% Cr203 en Mg/Cr02/2 : 45.5 - 15.19 = 30.41

Contenido de Mg/Cr02/2:
- 163 -

Cr203 = Mg/Cr02/2

M : 152.02 192.34 r

19 2,3
% Mg/Cr02/2 = - 2. . 30 .41 = 38.47

La diferencia entre la cantidad total de MgO en el mineral la que corresponde


a Mg/Cr02/2, está contenida en MgC03.

% MgO y % MgC03 pueden ser calculados análogamente como se zo arriBa.

Mg/Cr02/2 =■ MgO M^ :

192.34 40.32

2
% MgO = 19*2-3 . 38.47 = 8.06

MgO = MgC03
M : 40.32 84.32
r
% MgC03 = !^§! • 8.06 = 16.86

Contenido de CaSi03 se oBtiene como valor complementario 10Q %


% CaSi03 = 10Q - /22.37 + 38.47 + 16.86/ = 22.30
Un kilogramo de mineral contiene:

223.7 g de Fe/Cr02/2 1 mol

384.7 g de Mg/Cr02/2 2 mol

16 8.6 g MgCO, = 2 mol

223.0 g CaSiO., = 2 mol

3. Para simplificar el problema, podemos presumir que el ácido clorhídrico


reacciona con 1 kg de mineral, i.e. con 168.6 g de MgCO.j y con el CaO
que está contenido en 223.0 g de CaSiO^ i.e. con 107 065g de CaO.

Así, 276.25 g de mineral /168.6 g + 107.65 g/ reaccionaron , mientras que


723.75 g no reaccionaron.

Un kilogramo de mineral contiene 456 g de Cx^O-^ /45.6%/ y la misma cantidad


permanece en la parte que no reacciona la cual representa:

456
100 = 63.0
723.75

4„ La masa del relleno del tubo ha aumentado por la masa de CO., formada por
l a descomposición del MgCO^ con ácido clorhídrico. De 16 8.6 g de MgC03
se formaron 87.9 8 g de C02 y así la masa del tubo después de la reacción
es 500 g.
- 164 -

EJERCICIO N° 2 10° OIQ

Una muestra de agua bajo investigación tiene 10°de dure


za temporal y 10. "de dureza permanente. La dureza del agua es a
2+ 2 +
causa de los cationes Fe y Ca solamente.

Un volumen de 1Q.00 dm3 de esta agua está a su disposición. De este


volumen 100.00 cm3 fueron tomados para procedi mientos futuros. El agua se oxidó
con una solución de H2^2 ^ s— guidamente se le agregó una solución acuosa de
amoníaco. Se obtuvo un precipitado café el cual fue secado y después de un ca
-lentamiento apropiado se obtuvo 0.01432 g de un producto anhidro.

Problemas:
2+ 2+
1. Calcular la razón molar Fe : Ca en el agua bajo investiga ción.
2. En otro experimento 10.00 dm3
del agua fue usado de nuevo. La
2+
dureza temporal causada por los cationes Ca fue removido y
2+
luego la dureza permanente causada por los cationes Fe fue removida por
la adición de Na^PO^.
Calcular la masa del precipitado /en su forma anhidro/ bajo la presunción
de que solamente la mitad de los cationes Fe2+ fueron oxidados a Fe3+ en 10.00
del agua analizada. Los cálculos deben ser hechos con una precisión de
una centésima. Dé la razón molar con números enteros.
3

l°de dureza = 10 mg CaO/dm de agua. Masas

atómicas relativas:

Ar/Ca/ = 40.08; Ar/Fe/ = 55.85; Ar/C/ =


12.01;

Ar/H/ = 1.01; A^/P/ = 31.00; Ar/0/ =


16.00
Producto anhidro : Fe203

n/Fe203/ = 0.01432 g Qe 100 cm3 de agua, i.e.

I » 432 g de 10 dm3 1

mol Fe203 = 2 mol FeO

n/Fe O-/ = —1-432 3— ;\, o.009 mol 159.7


g.mol
- 165 -

--------------------- S O L U C I O N — -------------------------

m/Fe.O,/ = n.M = 2 „ 0,009 mol . 71.85 g.mol"1 y 1.293 g l°de


dureza = 10 mg CaO/dm de agua

l°de dureza = . 10 mg = 12,81 mg FeO/dm3 de agua

1.293 g FeO „ ln0, , ~~ 0.1281 g FeO ^


10 de dmreza

Dado que el agua tiene una dureza total de 20°, y 10° de dureza corresponden
al FeO, los otros 10°de dureza correspon -den al CaO que es 1 g de CaO en
10 dm3 de agua.

Razfin Molar:
m/FeO/ m/CaO/ n/FeO/ : n/CaO/ = M/FeO/ :
M/CaO/

= 1.289 . i: = 1 : 1
5d 08
71.85 ' '

Un volumen de 10.00 dm3 de agua contiene tanto hierro el que corresponde


a 1.293 g de FeO . 50% de hierro /0 .6445 g de FeO/ fueron oxidados a Fe/III/,
por lo tanto Fe3/P04/2 también oo mo FePO^ se formaron al mismo tiempo.

3 mol FeO ..... 1 mol Fe3/P04/2

215.55 g ..... 357.55 g

0.6445 g ..... 1.0699 g Fe3/P04/2


- 166 -

EJERCICIO N° 3 10°
OIQ

1 mol FeO 1 mol FeP04

71.85 g 150.,85 g .1.

0.6445 g 354 2 g FePO

Masa del precipitado: 1.0699 g + 1.3542 g = 2.4241 g


- 167 -

El cromado se hace usualmente por electrólisis en solu -ción de ácido


crómico. Los objetos a cromar forman el cátodo.El ánodo es una aleación la
cual es inerte en las condiciones dadas, i.e. no reacciona ni química ni
electrolíticamente.

Una celda electrolítica fue llenada con un volumen de 100.0 dm3 de


una solución acuosa que contenía 0.230 kg de anhi -drido ácido de cromo en
un dm3 de la solución.

Una corriente de 1500 A se utilizó durante la electróljL sis la que


circuló por 10.0 horas a través del electrolito. Después de la electrólisis
la masa del cátodo aumentó 0.679 kg.

Razón de los volúmenes gaseosos:

VC

= 1.603
VA

donde y^ es el volumen de los gases liberados en el cátodo y Vft es el volumen


de los gases liberados en el ánodo. Ambos volúmenes fueron medidos en las
mismas condiciones.

Problemas:

J.. ¿Qué parte de la carga total /en %/ fue usada para un depósito de 0.6 79
kg de cromo?

2. Calcular:

a/ la razón de los volúmenes gaseosos /a STP/ que se libraron como


sub-productos en el cátodo y ánodo;

b/ la eficiencia de la corriente para las reacciones corres -pondientes


tomadas separadamente en el cátodo y ánodo cuan do los gases son
liberados.

Si Ud. encuentra alguna desproporción en los datos calculados


- 168 -

EJERCICIO N° 3 10° OIQ

y aquellos dados en el ejercicio, trate de explicar que proce so tendría


lugar en la celda electrolítica que no haya sido considerado hasta ahora.

Escriba la ecuación abreviada correspondiente para la reac ción en los-


electrodos y corrija si es posible sus cálculos previos.

S O L U C I O N

Total de la carga eléctrica que pasa a través del electroli -to:

«>■ 15°0 -M3gSS • 10 - »»•' r

Reacción en el cátodo:

CrVI + 6 e ------ i- Cr°, ó;

CrO2- + 8 H"+ + 6 e -------- ► Cr + 4 H20

Depositado:

-- 679 g ------ = 13.Q6 mol de cromo


51.996 g.mol

Una carga de 78.36 F se necesitó para depositar el cromo arri_ ba señalado.

Eficiencia de la corriente:

5¡¡:¡6 F • 100 = I 4 - O %

2. La presunción más simple: Hidrógeno se libera en el cátodo y al mismo tiempo


se libera solamente oxígeno en el ánodo. Bajo esta presunción las
cantidades liberadas de oxígeno e hidróge no son como siguen:

n/0,/ = 539 '6 = 139.9 mol


¿ 4

r,/« / 559,6 . 0 , 86 ... ,


n/H-/ = ---------------- = 240 .63 mol
2

La razón molar es:

m/H-,/ V/H-/ _.n ,. .


2' 27 _ 2 40.63 mol .
n/02/ V/O / ~ 139.0 mol -t./¿u

Este valor es diferente al dado en el ejercicio. De modo.que, junto a las


reacciones mencionadas otros procesos tienen lugar en los electrodos. La
eficiencia de la corriente puede ser calculada de la razón de volúmenes
de los gases libera -dos, sin hacer ninguna investigación del tipo de
procesos in_ volucrados.
- 169 -

Balance de los procesos:

el proceso principal: CrO^ ------ ► Cr + 3/2 02 /6F/


cátodo ánodo

n = 14.0% el

sub-proceso: /electrólisis del agua/

2 H20 ----- >• 2 H2 + 02


cátodo ánodo
n2 = ?

Cantidad de hidrógeno liberada en el cátodo es igual a:

Q • ru
n/H / = ¿

2' 2

La cantidad de oxígeno liberada en el ánodo es igual a:

Q . /n- + n,/
n/02/ = -------- \ ---- L—
De acuerdo a los datos dados en el ejercicio:

Q . n2
2' "'"2' Q . /ni + n2/
y/U2/ n/H2/ T/h~7 =
= 1.603
n/0-/
4

Resolviendo la ecuación anterior para n-, se obtiene el valor: n2 = 0.565

/56.5%/

Los volúmenes de hidrógeno y oxígeno liberados: „/H2/ =

559 -6 -2 °-565 = 158.1 mol

V/H2/ = 22.41 dm3 . mol-1 . 158.1 mol = 35 43 dm3

n/02/ = 559.6. /0.140 + 0.565/ = 9g>6

V/Q / = 22.41 dm3 . mol-1 . 98.6 = 2 210 dm3

La eficiencia de la corriente cuando el hidrógeno es liberado en el cátodo


es igual a 56.6 %

La eficiencia de la corriente para la liberación del oxígeno en el ánodo


es igual a 70.5%.

Así, el 29.5% de la carga eléctrica es usada sin un efecto a-


parente. Por lo tanto,algunos procesos cíclicos están ocu
rriendo en la celda electrolítica, los cuales son la causa de 2 —
- 170 -

que el anión CrO^ sea reducido en forma incompleta. Una de las reacciones
que causa una disminución del valor de la eficiencia es la siguiente:
CrO2" + 8 H+ + 3 e ^± Cr3+ + 4 HjO
- 171 -

EJERCICIO N° 4 10° OIQ

Un recipiente de un volumen de 5.0 dm fue llenado con etano a una


temperatura de 300 K y presión normal y sellado. El recipiente con el gas fue
calentado y la presión se midió a dis_ tintas temperaturas. Los siguientes
valores fueron obtenidos:

T /K/ Presión, p, Presión, p',


medida en /kPa/ calculada /kPa/

300 101.325

50Q 169.820

800 276.111

1000 500 .748

Problemas:

1. Calcule la presión del etano, p 1 , en el recipiente de acuerdo a la ley de


los gases ideales y escriba los valores en la columna correspondiente de
la tabla.
/R = 8.314 J. mol"1 . K-1/

2. Explique la diferencia entre los valores teóricos p' y aque -líos /p/
obtenidos por medidas.

3. Escriba la ecuación química para la reacción que probablemente tiene lugar


en el recipiente a altas temperaturas.

4. Calcule el valor para el grado de conversión a del etano y el


valor de la costante de equilibrio K de la reacción que tie-
P
ne lugar a temperaturas de 800 y 1000 K.

5. La razón entre las constantes de equilibrio Kp a dos temperaturas diferentes


de acuerdo a la ecuación de van't Hoff :
- 172 -

/ln = 2.303 log/

Calcule el valor -medio de A H para el calor de la reac ción en


el rango de temperaturas de 800 - 1000 K.

6 . ¿Qué influencia tendrá Tina elevación de la temperatura y la presión en el


grado de conversión del etano?

------------------------- S O L U C I O N ---------------------------

1. La Tabla completa tiene los siguientes datos:

T/K/ p/kPa/ p' APa/


300 101.325 101.325
500 169.820 168.325
800 276.111 269.930
10.00 500.748 337.412

2. Los valores p son a altas temperaturas mayores que los calcu


lados /p'/. De aquí, el número de moléculas /moles/ en el
sistema aumenta. Aparentemente esto ocurre por descomposi -
ción térmica del etano.

3, Los alcanos se descomponen térmicamente produciendo alkenos


e hidrógeno:
C2H6 t C2H4 + H2

La ecuación de Clapeyron para la sustancias que experimentan


descomposición térmica originando dos sustancias gaseosas , tiene la
forma: p.y = n/l + a/ . B.T,

donde a es el grado de descomposición.

De una -mol de C-H,: ¿ o


a moles de CjHg y a moles de H2 se obtienen y 1 - a moles de
C n H , quedan sin reaccionar. ¿ o

De n moles de C-H. : se obtienen n a moles- de C-.H. y n a moles de


¿ O ¿ 4
H.,; quedan sin reaccionar:

n/l - a / moles de C2Hg

4.
- 173 -

Por lo tanto, la cantidad de sustancia de los compuestos en la mezcla gaseosa


será:

Zn = 2 n a + n/l - a/ = n/l + a/

Comparando los valores teóricos y experimentales de presión ob -tenemos:


p' . V = n.R.T. => p' = --r- R.T

p.V = m/1 + a/.R.T => p = n/1 * a/ R.T

p' _ n _ p - p'
p n/l + a/ a p'

276.111 - 269.930 n
a =
800 --------- 269 .930 --- = ° - ° 2 3

= 500.748 - 37.412 =
a
1000 337.412 u - 4 0 ^

La reacción tiene lugar en fase gaseosa y así la constante de e quilibrio


se calcula de acuerdo a la relación: p
C2H4 • PH2
K =
p PT-C2H6

4.
- 174 -

EJERCICIO N° 5 10° OIQ

0
T = 800 K: K = -023 \ • 269
•930 = 0.146 kPa
p 0.977
2
0 u ,484
qs 337 412
T = 1000 K: K = - ° ^ ' ^ =
153.18 kPa
p O . DJ .6

5. De acuerdo a la ecuación de van't Hoff:

Kl

2. 303 log —~ ---- . R


SH = --------------- V -----
2 x2

Después de sustituir los. -valores conocidos: AH = 2

6. Dado que la reacción es endotérmica y el número de partículas ha aumentado


en el curso de la reacción, de acuerdo al princi_ pió de Le
Chatelier-Brown, el equilibrio se desplaza hacia los productos cuando se
aumenta la temperatura y en sentido con trario cuando la presión es
aumentada.

7. Si el valor correcto es AH^ y AH., es el valor calculado, luego el error


relativo es calculado por la relación:

ÍH1 " *H2


100 / % /
AHa

31.36 k J.mol"1
- 175 -

Un cierto compuesto orgánico líquido X /presente en e l alquitrán


del carb.ón/, con una masa de J..06g fue quemado produciendo 0.90 g de agua
y 3.52 g de dióxido de carbono. Sus vapo res son 3.79 mas densos que el
nitrógeno.

El compuesto X fue oxidado por CrO^+f^SO^ en caliente. Una


sustancia incolora cristalina A fue aislada de la mezcla de la reacción. Es
;soluble en una solución acuosa de NaOH ó NaHCO^.

Cuando el compuesto A es calentado pierde agua y se con vierte


en el compuesto B. La condensación del compuesto B con fenol en presencia
de HjSO^ o ZnC^ produce una sustancia Y la cual es a menudo usada como
indicador ácido-base.

Los compuestos A y B cuando se calientan con exceso de 1-butanol


/se agregan algunas gotas de una solución de H^SO^/dan el -mismo compuesto
líquido C.

Si se acepta que un átomo de carbono muestra una ten -dencia para


formar cuatro enlaces en los compuestos orgánicos , es posible escribir
formalmente dos fórmulas diferentes para el compuesto X. Las fórmulas
escritas en esta forma no corresponden, sin embargo, a la estructura química
de la molécula X, por que hasta hoy día nadie ha tenido éxito en la preparación
de los dos isómeros hipotéticos.

Haayman y Witbaut efectuaron en 19 41 una ozonización del compuesto


X en una solución de cloroformo. Después de la hidró lisi's de los ozónidos,
se encontró que la capa de agua contenía tres compuestos orgánicos diferentes
con una razón molar de D : E: F = 3; 2; 1. Solamente dos de ellos formaron
nuevos compiles tos G y H. por una oxidación suave, el tercero permanece sin
cam biar bajo estas condiciones pero con un agente oxidante mas fuer te como
H202, por ejemplo, forma un compuesto líquido muy conocido I con un olor agudo
característico.
- 176 -

EJERCICIO N° 5 10° OIQ

Una muestra del compuesto G puro y anhidro se disolvió en una


solución acuosa 1-molar de H2S04 y la solución resultante se valoró con una
solución 0„05-molar de KMnO^„ Se necesitó 25.6 cm de la solución de KMnO^
para 0.2 88 g de la sustancia G „

Problemas:

1. Escriba la fórmula abreviada para e l compuesto X.

2. Basado en la información y los datos del ejercicio, escriba las ecuaciones


químicas de las reacciones por las cuales se forman los compuestos A , B ,
C„

3. Dé el nombre del compuesto Y y escriba la ecuación de su síntesis. Escriba


su fórmula estructural y su color en solución ácido y básica.

4. Escriba dos fórmulas estructurales para el compuesto X y también una


fórmula más correcta de acuerdo a los últimos descubrimientos.

5. Escriba la ecuación química para la ozonólisis del compuesto X por la cual


puede ser explicado el hecho del porque' los com puestos D , E , y F después
de la hidrólisis presentan en la ca pa acuosa una razón molar de 3 : 2 :
1.

6. Escriba las ecuaciones químicas para las reacciones de formación de los


compuestos G, H , I.

7. Escriba la fórmula estructural formal utilizada anteriormente y una


fórmula estructural correcta y mas moderna de otro compuesto orgánico
líquido si sabe que el compuesto está también presente en el alquitrán de
carbón y es un derivado del com -puesto X.
¿Qué nombre de un conocido químico está conectado en la histo
ría con la fórmula de este compuesto básico? ¿Cuáles son los productos
obtenidos por su ozonólisis?

Escriba los nombres de las sustancias X, Y , A - I, debajo del compuesto


correspondiente en l a ecuación.

Nota:

"Ose en sus cálculos: M/C/ = 12 g.mol ^; M/0/ = 16 g.mol "S


M/H/ = 1 g.mol-1; M/KMnO^/ = 158 g.mol"1
- 177 -

S O L U C I O N

1. La fórmula empírica del compuesto X puede ser calculada de la composición


de los productos de combustión de este compuesto:

n/H2°/ = ---- ° - 9 9 = 0 o 0 5 mol => n/H/ = 0.1 mol


18 g.mol

% H = °'1 „ 100
= 9.4 1.06

n/CO / = ----- 3 - 5 2 g _ - = 0.08 mol => n/C/ = 0.08


44 g, mol i.e. 0.096 g

% C °'96 g . 100 = 90.6 1.06 g

X ; C H. * : y 1 = %Í : ^ = 4 : 5
' x y - 12 1

Fórmula Empírica: C4H5

Fórmula molecular: /C4H5/n


La masa molar del compuesto X se calcula de la siguiente mane
- 178 -

M/X/ = JM/H-/ „ --------- = 28 g.mol . 3.79 = 106 g.mol


P /N2/

La fórmula molecular del compuesto X es C Q H .

La información dada en el ejercicio concerniente al compuesto X , soporta


la presunción de que el compuesto X es 0- x i l e n O o

(6
calentado
•-Lh.tQf"1 .COOH
-H20

r
3 -2H20

Q-xileno; ácido itálico anhídrid


1,2-dime 1,2-benceno- o
til-benceno ácido itálico;
; dicarboxílico;

El ácido ftálico /A/ o su anhídrido /B/ cuando se calientan con un exceso


de 1-butanol con la adición de una cierta canti_ dad de ácido mineral como
catalizador, produce ester dibutíli^ co de ácido ftálico - C:

+ 2 H O
^COOC,Hq
COOH

rr\r C00H + r^-C00C4H9

COOC4H9
ION 2 C4H9OH calentando '
X° + 2 C4H9OH calentando

COOC4Hg
+ 2 H20

C ; dibutil ftalato
- 179 -

3. La condensación del anhídrido ftálico con fenol en presencia de H _ S O . ó


ZnCl- anhidro:

Fenolftaleína

La fenolftaleína es usada como indicador ácido-base la cual se incolora


en una solución acida y rojo púrpura en solución alcalina.
incoloro

OH

01,0
COO

rojo
Fórmulas de Kekulé para el o-xileno:
CH.

CH CH.
permitirían sugerir que este compuesto .
existe en dos formas i-sómericas. Sin CH.
embargo nadie ha tenido éxito en la
obtención de los dos isómeros del o-benceno disustituído. Hoy en día se
sabe que todos los enlaces C-C, también como el enlace C-H en el Benceno
y sus- derivados son equivalentes.
- 180 -

Por lo tanto, la formula para el o-xileno puede escribirse de la siguiente


manera:

Esta forma de escribir las fórmulas expresa que los enlaces ir están
igualmente divididos en todo el anillo bencénico. Por su puesto que tales
fórmulas no sostienen más la existencia de dos formas isoméricas del
o-xileno.

En 1241 Haayman y Witbaut suministraron más evidencia química para la


equivalencia de los seis enlaces C-C en el anillo ben cénico. Ellos
permitieron reaccionar el o-xileno con ozono y obtuvieron dos diferentes
triozónidos con una razón molar de 1 : 1 . Los productos de la ozonólisis
fueron descompuestos por agua formando tres sustancias diferentes:

HC C 3
3 0, n -C C 0 H H H ■
I I ------- — 6 ? I --------- ' ZCHj-C-C ^lc-C 0 0 0 E
HC C H ;C ,-C-H H 2 0
D
H

CH3 H C ^
:H
3

.CH
O"0 \ CH3 1¿ 0 0 H H
HC 3 30-,
-*CHTC - C - C H , +£-C.
H-C 3 Ó'

CH

H H. \ /
C-C D etanodial, glioxal,
/ V
00

0o
IIII
CH^-C-C-H E propanonal, metilglioxal,
- 181 -

00
II II
CH3-C-C-ch3 F butanodiona, diacetil,

Los productos en la mezcla resultante /después de la hidrólisis/ están en


la razón molar 3 : 2 : 1 lo cual prueba la equi valencia de los enlaces C-C
en el anillo bencénico.

De los tres compuestos arriba obtenidos D, E, y F, solamente los dos primeros


son fácilmente oxidables a los correspondien tes ácidos:

V h0cc-c0oh
H H H OH
/ HNO diluido \_/ oxidación
C-C ------------------- *- ---------------------------- ►

0 0 00

ácido glioxálico ácido oxálico

CH -C-C agUa dS br°m° CH,-H-c00h


3 ^ ----------------------------------- e- 3
0

E H
ácido pirúvico
ácido a -ketopropiónico
El compuesto F requiere agentes oxidantes más fuertes, tales
como soluciones de H_0_, HIO„, etc.
2 2' 4'

0 0
II |l solución caliente
CH3-C-C-CH3 ----------------------------- 2 CH3-COOH
H2°2

I
ácido etanóico, ácido
acético.

El ácido oxálico se usa como una sustancia estándar en la pre paración de


soluciones volumétricas de KMnO^:

2 KMn04 + 5 /COOH/2 + 3 H2S04 = 2 MnS04 + K2SC>4 + 10 C02 + 8H20

Los datos experimentales de la determinación del compuesto G por titulación


con 0.05-molar KMn04 demuestran que el compues to es ácido oxálico y asi
ello confirma la exactitud de la solución.

n/KMn04/ = c.V = 0.05 mol.dm"3 . 0.0256 dm3 = 0.00128 mol n/ /COOH/2/ =

5/2 . 0.00128 mol = 0.0032 mol


- 182 -

Lo que corresponde a 0.2 88 g de sustancia G lo que está de a-cuerdo con


el resultado dado en el ejercicio.

En 1865 Kekulé sugirió una fórmula cíclica para el benceno:

Sin embargo,fue probado experimentalmente que todos los átomos


- 183 -

EJERCICIO N° 6 /experimental/ 10° OIQ

de carbono y los de hidrógeno que están en la molécula del ben ceno son
equivalentes. Por las mismas razones dadas en 4., la fórmula del benceno
se escribe hoy en día en esta forma:

La ozonólisls del benceno produce un triozónido el cual por hidrólisis


se transforma en glioxal.

H
l
C

HC eH 3 H2°
H H
3 0 , „ >
------ * 3 C + 3 H2<D2
! I! // 0
HC CH
fe
\ /
c I
H
- 184 -

Cuatro soluciones acuosas están disponibles en el mesón del


laboratorio^ Estas son soluciones de HCl, NaOH, NH^ y NH^CCOH cuyas
concentraciones son aproximadamente 1 mol.dra 3. La concen tración del HCl
es la única que ha sido exactamente determinada y conocida.

Utilizando la solución volumétrica de HCl, determine la


concentración exacta de las otras soluciones. Una bureta, una pipeta, vasos
para titular y los indicadores anaranjados de -metilo y fenolftaleína están
a su disposición. Cada valoración debe e-jecutarse dos veces y se calcula el
valor promedio para la con -centración. Una tercera valoración se ejecutará
solamente en el caso de que los dos resultados previos de las valoraciones
difie ran mas del 2%.

Ahora Ud. efectuará las siguientes medidas termoquími -cas del calor
de neutralización producido en las reacciones de las soluciones de ácidos y
bases arriba dadas:

3
a/ Mida cuantitativa y exactamente 50.0 cm de HCl en un vaso de p.p. Mida en
otro vaso un volumen de la solución de NaOH la cual contiene el mismo número
de moles como las del HCl en el primer vaso. Luego mida la temperatura de
ambas soluciones con una precisión de 0.2K. Vacie rápidamente el contenido
del pri mer vaso en el otro usando un termómetro como varilla de agitación,
y revuelva la solución. Determine la mas alta temperatura de la mezcla.

B/ Haga medidas análogas con los pares siguientes de ácidos y ba ses:

HCl - NH3, CH3COOH. - NaOH, CH3C00H - NH^.


Problemas:

le ¿Que indicadores han sido usados para las determinaciones individuales?


Dé aproximadamente la región de pH en la cual los indicadores antes
mencionados muestran su color de transición. Dé las razones para el uso
de un determinado indicador, usando solamente ecuaciones iónicas para las
reacciones las cuales son características o las propiedades específicas
de las sales formadas en las reacciones individuales de neutralización.Cal_
cular la concentración de todas las soluciones investigadas.
- 185 -

EJERCICIO N° 6 /experimental/ 10° OIQ

2. Escriba la ecuación calorimétrica en su forma general por medro de la cual


se puede calcular en calor de neutralización. Calcule el efecto térmico
para cada reacción de neutraliza ción investigada y dé el valor en relación
a un mol de agua for mada.

Las densidades de la soluciones son las siguientes:

p/HC1/ = 1.02 g.cm-3 p/NaOH/ =

1.04 g.cm-3 p/NH3/ = 0.99 g.cm-3

p/CH3COOH/ = 1.01 g.cm-3

En los cálculos como valor de la capacidad calórica para las


-1 -1
soluciones es igual a 4.19 J.g . K , donde la capacidad ca lórica del vaso
y del termómetro pueden ser despreciables.

3. Ha obtenido Ud. resultados iguales en todos los casos? Si no, ordene los
sistemas de las reacciones pertinentes de acuerdo al valor decreciente del
calor de reacción.
¿Qué reacciones son las causantes de las diferencias anterior mente
mencionadas? Exprese las reacciones mediante ecuacio -nes químicas.

4. El método exacto determina que el calor de neutralización de


un ácido fuerte con una base fuerte /i.e„ el calor de reacción cuando se
forma 1 -mol de agua de iones H y OH / es igual a 57.57 kJ„mol 1„ Calcule
el error relativo de su determina crón.

S O L U C I O N

i. En la valoración de Tin ácido fuerte con una base fuerte, tanto la


fenolftaleína como el anaranjado de metilo pueden usarse como indicadores
ácido-base. La caída en la curva de valo ración cubre el cambio de color
de ambos indicadores /pH en -tre 4 - 10/„

Solamente la fenolftaleína puede ser usada en el caso cuando un ácido débil


se valora con una base fuerte porque la neutra li'zación ocurre a valores
más altos de pH /el calor de transi_ ción de la fenolftaleína es en el rango
de pH = 8 - 10/. La sal formada sufre hidrólisis /más precisamente su anión/
y la solución muestra reacción básica:
CH3COO + OH

+ 2 H20 En la valoración de mn ácido fuerte con


una base débil o vice versa, se usa

NH* + 2 H.O 4
2
- 186 -

metil orange /pH = 3 - 4.5/ y debido a la hi -drólisis resultante la solución


muestra reacción acida:
La concentración exacta de las soluciones acuosas de hidrólisis de sodio
y amoníaco se determinan por valoración con una solución volumétrica de
ácido clorhídrico. La concentración de una solución de ácido acético
se determina por titulación con una solución de hidróxido de sodio.

2. Cuando la capacidad calórica del vaso y del termómetro se des precian, el


calor de neutralización puede ser calculado de acuerdo a la relación simple:

AHneutr, = /ml + V ' C ' /T2 " V


m1 -m2
masa de la primera solución;
-
masa de la segunda solución;

capacidad calórica de las soluciones;

temperatura de las soluciones antes de mezclarlas;

temperatura de las soluciones después de mezclarlas.

Si las temperaturas de las soluciones antes de ser mezcladas no son iguales,


será el valor promedio de la tertperatura de ambas soluciones.
Finalmente el calor de neutralización debe ser referido a la formación de
un mol de agua.

3. Los resultados obtenidos para la concentración de una base fuer te con un


ácido débil y vice versa, también como para la reac ción de un ácido débil
con una base débil, son mas bajos que aquellos obtenidos para la
neutralización de un ácido fuerte por una Base fuerte. Una parte del
calor es gastado por la ionización de un electrolito débil:

CfLjCOOH + H20 + CH3C00~ + H30+

Una ecuación similar puede escribirse por NH...H-0.


- 187 -

EJERCICIO N° 7 /experimental/ 10° OIQ

Los valores de los potenciales estándares de reducción se dan para


los siguientes sistemas redox:

2 S202"/S40g" E° = 0,17 V

2 l"/I2 E° = Q.535 V

2 S02-/S202g- Eo
2.05 V

Problemas;

1. Ponga en orden las formas de oxidación de los sistemas redox arriba dados
desde el agente oxidante mas débil al más fuerte /escriba en la Tabla 1/.

De manera similar, ordene las formas de reducción desde el a-gente reductor


más débil al más fuerte,

2. En las ecuaciones abajo dadas marque con una flecha el curso esperado
/dirección/ de la química /Tabla 1/.

2 I" + S402" = I2 + 2 S202-

2 i" + S202~ = I2 + 2 SO2-

2 S202" + S202g- = S402" + 2 SO2"

3. Bajo la presunción de que las soluciones usadas tienen la mis_ ma


concentración, es posible decir sin hacer el experimento cuáles de las
reacciones dadas tendrían velocidades de reacción más alta y cuáles mas
bajas?
- 188 -

4. Para confirmar las. hipótesis dadas en 3, ejecute los siguien tes- tres
experimentos cualitativos:

Experimento 1
Tacie 20.0 cm3 de una solución 0„10-molar de Na_S.0_ en un Er
2 2 2 2~
lenmeyer y rápidamente agregue agitando violentamente 1.0 cm de Tina
solución 0o10-molar de Yodo.

Experimento 2 ción de cada una de las sustancias iniciales de la


3
Mida 20.0 cm de una solución 0o10-molar de /NH./-S.0o en un
4 2 2 8 ^
Erlerrmeyer y rápidamente agregue bajo fuerte agitación 4.0cm de una
solución 0 »10.-roolar de yoduro de potasio.

Experimento 3
3
Ponga 20^0 cm de una solución 0.10-molar de /NH4/2S.,Og en un
Erlenmeyer y agregue rápidamente bajo intensa agitación 2.0 3
cm de una solución 3e tiosulfato de sodio 0.10-molar.

Dado que ambos reactantes y los productos de la reac ción son


incoloros, el curso de la reacción debe ser seguido indirectamente. Para
este propósito agregue a la solución de£ pues de dos minutos dos o tres
gotas de una solución 0.10-mo-lar de Yodo. Si el resultado de su experimento
es sorprenden te, ejecute el experimento 3 de nuevo, pero permita reaccio
-nar las soluciones de Na2S203 y /NH4/2S2Og por 10 minutos.

Ordene las reacciones de los experimentos N" 1 - 3 /en la Tabla 3/


de acuerdo al aumento de las velocidades de reacción y entonces responda
las preguntas cuando sea posible sobre la Base de los valores conocidos,
los valores de los po -tendales estándares de reducción a adivinar, por
lómenos cualitativamente, la velocidad de reacción para una mezcla de reac
crón que tiene dos pares de sistemas redox.

Las conclusiones hechas en base de los experimentos dados nacen


posiBle investigar la influencia de la concentra
- 189 -

EJERCICIO N° 7 /experimental/ 10° OIQ

velocidad

_ o —
de reacción entre los iones I y S_ , 0 Q ,

Haga el experimento N° 4 de acuerdo a las siguientes instrucciones :

Experimento 4

a/ Mida sucesivamente en Erlenmeyer de 2 5 0 cm3:


2 5 . 0 cm
3 de una solución 0 . 2 0 -molar de yoduro de potasio,
3
1 0 . 0 era de una solución 0 . 0 1 -molar de tiosulfato de sodio,

5.0 cm3 de una pasta de almidón,


y agite el contenido del Erlenmeyer.

b/ Mida 25.0 cm3 de una solución 0.20 -molar de /NH4/2S2Og en un vaso de p.p. de
100 cm3. "Vierta el contenido del vaso rS pidamente en el Erlenmeyer, tome
el tiempo con un cronómetro y agite el contenido del Erlenmeyer. Mida el
tiempo hasta el momento cuando la solución se torna azul.

Ejecute análogamente el experimento N° 4 tres veces, usando los volúmenes abajo


dados de la solución de yoduro de potasio mientras que los volúmenes de Na2S2C>3
y /NH4/2S 2 0g y de la pasta de almidón permanecen sin cambiar. Mas aún, agregue
a las so luciones abajo dadas volúmenes de una solución 0o20 -molar de nitrato
de potasio de tal modo que el volumen de la solución resultante sea siempre
el mismo. Explique el uso del nitrato de potasio en este caso.

4 /ii/ : 15.0 cm3 0.20 -molar KI + 10.0 cm3 0.20 -molar KN03

4 /iii/ : 10 o 0 cm3 - " - + 15.0 cm3 - " -

4 /iv/ : 5.0 - " - + 2 0 . 0 cm3 - " -

Haga Tina lista con los resultados de los experimentos N°l - 4 claramente en
las Tablas adjuntas.

Escriba las fórmulas de las sustancias correspondientes sobre las flechas en


la Tabla 1 /como se solicita bajo 1/ y marque el curso esperado de la reacción
con flechas en la ecuación.

Para una evaluación cualitativa de la velocidad de reacción /Tabla 2/, utilice


términos tales como: muy rápido, rápido , lenta y muy lenta.
- 190 -

Llene la Tabla 3 exactamente de acuerdo con los títulos de las coiumnas.

Calcule la velocidad de reacción de acuerdo a la fórmula:

Ac/S2028-/ 3
v = ----- 1 ----- /molcdm . s /
Ae

2- 2-
Ac/S2Og / - cambio de la concentración del S20g en
un intervalo de tiempo.

Dibuje /sobre el papel mm adjunto/ la dependencia de la velocidad de la reacción


con la concentración de los aniones I a
2-
una constante concentración de los aniones S20g en la solu -ción.

Haga uso del conocimiento obtenido de los experimentos ante -riores y de las
soluciones que están a su disposición para su gerir otro experimento con el
cual sería posible investigar la dependencia de la velocidad de reacción sobre
la concentra ción de aniones SjOg a una concentración constante de anio -nes
I en la solución.

Considerando la Tabla 3, llene la Tabla 4: marque las colum -ñas en la Tabla,


sugiera un plan del experimento y confeccione una lista de los resultados
experimentales y de los valo -res calculados.

Similarmente, como antes,dibuje la dependencia bajo investiga ción sobre el


papel mm.
- 191 -

7. Escriba una relación general para la dependencia de la velocidad con la


concentración de los reactivos y utilizando el diagrama adjunto, calcule
los valores para la constante de velocidad de reacción para ambos casos
y determine el valor promedio.

S O L U C I O N

i. s4o2- i2 s2o2"

Aumento de las propiedades oxidantes de las formas oxidadas

Aumento de las propiedades reductoras de las formas reducidas

2. El curso esperado de la reacción química:

2 I- + S.O2- I_ + 2 S-,02.- /a/

2 I- + S202_ -=*" I + 2 SO2- /b/

2 S202_ + S202_ S402_ + 2 SO2" /c/

3. La formulación de cualquiera hipótesis ya sea apoyando o re -chazando la


posibilidad de predecir la velocidad de reacción debiera ser aceptada como
correcta.

4. Resultados de los experimentos N° 1 - 3:

1 - reacción /a/ muy rápida;


2 - reacción /b/ lenta;
3 - reacción /c/ muy lenta, su curso apenas puede ser observa
do.

so 2- i" s 2 o 2_

Conclusión:
- 192 -

Las diferencias conocidas entre los valores de los potenciales


estándares de reducción de dos pares de sistemas redoy. no permite
adivinar aún cualitativamente, la relación propia en tre las velocidades
de las correspondientes reacciones.

5. Se le solicita completar en la Tabla 3 los siguientes datos:

- volúmenes de las reacciones individuales;


3
- el volumen total de la solución /65 cm /;
_ n —
- concentración calculada para I y S20g ;
- tiempo de reacción;
- valores calculados para la velocidad de reacción.

La adición de una solución 0,20-roolar de KNO^ se necesita pa ra mantener


constante la fuerza iónica de la solución.

Al dibujar la dependencia de la velocidad de reacción con los


valores de [I 3 /a una concentración cons-tante de aniones 2-
SjOg / se obtiene una línea recta que cruza el origen del sis tema de
coordenadas.

6. La Tabla 4 debiera ser completada en forma análoga a la Tabla 3, donde


deben ser especificadas las columnas individuales.

Las soluciones para el experimento se preparan en la misma for ma pero


la solución de KI /25 cm3/ formará una adición cons -tante, mientras
que las de /NH^SjOg y /NH4/2S04 formarán u-na adición cambiante en la
solución resultante. El sulfato de amonio juega el mismo papel que el
nitrato de potasio en el experimento precedente.

En este caso, la velocidad de reacción versus la concentra -


2 _
ción de aniones S_0o se dibuja /a concentración constante de aniones
I-/ obteniéndose también una línea recta que cruza el origen del sistema
de coordenadas.
- 193 -

7 . La velocidad de la reacción:

v = k „ is2o2-] . [rp

a/ I = consta

k' es la pendiente de la línea recta:

5/ [s20g~j = const. => v = k" . Ll~] 2

k» = k . IS202"}

1*2°?']
k" es la pendiente de la línea recta.

Los valores para las constantes de velocidad de reacción obtenidos


por los procedimientos a/ y a/ debieran ser teó ricamente iguales.
Si ellos difieren parcialmente, calcule el valor promedio de la
constante de la velocidad de reacción.
OLIMPIADA INTERNACIONAL DE QUIMICA

LENINGRADO

1979 UNION

SOVIETICA
- 195 -

PROBLEMA N° 1 11° OIQ

Al efectuar este ejercicio programado, encierre con un círculo


aquellas letras las cuales según su opinión corresponde a las respuestas
correctas de cada una de las 20 preguntas»

1. ¿Cuál de los elementos es oxidado en la reacción entre etile no y una


solución acuosa de permanganato de potasio?

a) carbón, b) hidrógeno, c) potasio, d) manganeso, e) oxígeno .

2. Aproximadamente cuántos litros de CO^ se formarán en la reac ción 18 g


de carbonato ácido de potasio con 65 g de ácido sulfúrico al 10%?

a) 1 1 , b) 2 1, c) 3 1, d) 4 1, e) 5 1.

3. ¿Cuál de los siguientes hidrocarburos produce el máximo de ca lor al


comfiustionar completamente 1 litro de gas?

a), propano, b) metano, c) acetileno, d) etileno, e) todos producen lo


mismo.

4. ¿Cuántos isómeros puede tener un compuesto si su fórmula es: C3H5Br?


a) 1, b)_ 2, c) 3, d) 4, e) 5.

5. ¿Cuál de los siguientes hidrocarburos será el mejor combusti ble de motor?

a) cicloctano, b\ 2,2-dimetilhexano, c) octano normal, d).


3-etilhexano, e) 2 , 2 ,4-trimetilpentano.

6. ¿Con cuál de los siguientes compuestos reaccionará una solución acuosa


del oxido de mayor valencia del elemento 33?
- 196 -

a) C02, b) K2S04, c) HCl, d) NaOH, e) Mg„

7» ¿Cuál debe ser la concentración mínima (en % de masa) de una solución de


1 kg de hidróxido de potasio necesaria para la muestralización completa
de 3=57 moles de ácido sulfúrico?

a) 5%, b) 10%, c) 15%, d) 20%, e) 25%'.

8. ¿Cuántos compuestos con la fórmula abreviada C^HgN pueden existir?

a) 1, b) 2, c) 3, d) 4, e) 5.

9. En cuál de los compuestos el contenido de nitrógeno dado en % de masa


es máximo?

a) nitrato de potasio, b) nitrato de bario, c) nitrato de aluminio, d)


nitrato de litio, e) nitrato de sodio.

10. En cuál de los átomos de carbono (indique su número de serie) se


adicionará principalmente el cloro en la reacción de HCl con ácido
2-pentanóico?

a) 1, b) 2, c) 3, d) 4, e) 5.

11. ¿Cuántos moles de agua hay por mol de nitrato de calcio en un cristal
hidratado si el contenido de agua es 30.5%?

a) 1, b) 2, c) 3, d) 4, e) 5.

12. ¿Cuál de estos ácidos orgánicos es el más fuerte?

a) benzoico, b) 2-clorobenzoico, c) 4-metilbenzoico, d)

2-aminobenzoico, e) 4-bromobenzoico.

13. ¿Cuál de todos estos ácidos tiene el mayor grado de disocia-


ción?

a) HC10, b) HC102, c)_ HC103, d) HC104, e) todos tienen


el mismo grado.
14. ¿Cuál de las siguientes sales no sufren hidrólisis?

a) bromuro de potasio, b) sulfato de aluminio, c)


carbonato de sodio, d) nitrato férrico, e) sulfato
de bario.
- 197 -

15. Aproximadamente cuántos litros de aire son necesarios para la combustión


completa de un litro de amoniaco?

a) 11, b) 2 1, c) 3 1, d) 4 1, e) 5 1.

16. ¿Cuál de los elementos se oxida en la descomposición térmica del


hidrocarbonato de sodio?

a) sodio, b) hidrógeno, c) oxígeno, d)


carbono, el ninguno.

17. ¿Cuál de los siguientes cambios en las condiciones no afecta la posición


del equilibrio de la descomposición térmica de CaC03?

al aumento de la temperatura, bl disminución de la presión, cl adición


de un catalizador, d) un cambio en la concentración de CC>2.
el un aumento en la cantidad inicial de sustancia.

18. ¿Cuál de las siguientes sustancias se formarán en el ánodo de Pt en la


electrólisis de una solución acuosa de cloruro de aluminio?

al aluminio, b) oxígeno, c) hidrógeno, d) hidróxido de a-luminio, el


cloro.

-12.. Los aparatos mostrados esquemáticamente en las figuras están diseñados


para preparar amoníaco en condiciones de laboratorio. El tubo de
ensayo que se calienta contiene una mezcla Se NH.C1 y CaCOBl,. ¿Cuál
de las figuras es la correcta?

lO.. ¿Cuál de los aparatos mostrados en las figuras es el mejor para la síntesis
de bromoetano de bromuro de potasio, ácido sulfúrico concentrado y etanol?
- 198 -
199

S O L U C I O N

1. a) ; 6. d). y e) ; lio d) ; 16. e);

2. cí; 7. d) ; 12. b)_ 17. c). y e)

3. a) ; 8. d). ; 13. d) 18. b). y e)

4. el; 9. di; 14. a): y e)_ 19. c).


;

5) el; 10. c). 20. a)..


15 . d) ;

;
- 200 -

PROBLEMA N° 2 11° OIQ

Una aleación contiene los siguientes metales: cadmio , estaño,


bismuto y plomo. Una muestra de esta aleación que pesa 1.2860 fue atacada con
una solución de ácido nítrico concentra -do. Se obtuvo un compuesto individual
"A" como precipitado el cual fue separado, lavado repetidas veces, secado y
calcinado.La masa del precipitado de la calcinación a masa constante fue
0.3265g.

Una solución acuosa de amoníaco se agregó en exceso a la solución


obtenida después de separar el precipitado. Un compuesto de un metal "B"
permaneció en solución, mientras que to -dos los demás metales precipitaron
en forma de compuestos ligera mente solubles. El precipitado fue separado
cuantitativamente de la solución a la cual se le pasó una corriente de ácido
sulfhídrico hasta la saturación. El precipitado resultante que contiene un
metal "B" fue separado, lavado y secado. La masa del precipitado fue Q.6613
g.

El precipitado que contiene los compuestos de los meta les "C" y


"D" fue tratado con un exceso de una solución de NaOH. La solución y el
precipitado fueron separados cuantitativamente. Una solución de HNO^ se agregó
a la solución alcalina hasta pH 5 - 6 y un exceso de una solución de K2Cr04
se agregó a la solución transparente resultante. El precipitado amarillo fue
separado, lavado y cuantitativamente transferido a un vaso de precipitados;
una solución diluida de H.,S04 y KI cristalino se agrega ron „ El yodo producido
como resultado de la reacción fue titulado con una solución de tiosulfato de
sóido en presencia de almidón como indicador. Se necesitaron 18.46 mi de una
solución Q.1512 normal de Na2S203„

El último metal contenido en el precipitado como una sal


ligeramente soluble fue transformado en un fosfato aún menos soluble y su masa
fue de 0.4675 g.

Escriba todas las ecuaciones de las reacciones químicas en las


cuales está basado el análisis cuantitativo de la alea -ción. Nombre los
metales A, B, C, D. Calcule los porcentajes en masa de los metales de la
aleación.

-------------------------- S O L U C I O N ---------------------------
- 201 -

1. La acción del ácido nítrico sobre la reacción: Sn + 4 HN03

= H2Sn03 + 4 N02 + H20

Pb + 4 HN03 = Pb(_N03)2 + 2 N02 + 2 H20 Si + 6

HN03 = Si(N03)3 + 3 N02 + 3 H20 Cd + 4 HN03 =

Cd(N03)2 + 2 N02 + 2 H20

Relación de peso para la determinación del estaño: H2Sn03 = Sn02

+ H20

Cálculo del contenido de estaño en la aleación: M(Sn) =

118.7 g.mol-1 M(Sn02) = 150.7 g.mol-1

BÍS£l= ^Sn) ■ m(Sn) = 113.7 g.mol"1. 0 ,3265 g = Q>2571 g

m(Sn02 W(Sn02) 150.7 g„mol

Porcentaje de masa del estaño (metal A) en la aleación:

VCSN)
= MUH= °-1999 = 19-99%
2. Por la acción de un exceso de solución de amoníaco, tienen lu gar las
siguientes reacciones:

PB(.N03)2 + 2 NH4OH = PbCOH)^ + 2 IJH4N03

Si(Jí03l3 + 3 NH40H = SiC0H)3+ + 3 NH4N03

CdQJ03l2 + 4 NH4Oa = {CdíNH3)4} (N03) 2 + 4 H20


solución
Reacción que resulta de saturar la solución con ácido sulfhídrico.

{Cd(NH3)4> CN031 + 2 H2S = CdS+ + 2 NHjNC^ + (NH^S

3. Cálculo del contenido de cadmio en la aleación: M(Cd) =


-1 -1
112.4 g.mol M(CdS) = 144.5 g.mol

m(Cd) = H2,4 g.nof1 . 0.6613 g = 0_5143 g 144.5 g.mol

Porcentaje de masa de cadmio (Metal "B"l en la aleación

vfCd) = ^•^1¿3 g = 0,3999 = 39.99 % 1.2860 g


- 202 -

4. La acción del exceso de hidróxido de sodio sobre los hidroxi ios de plomo
til) y bismuto flll):

PbtOH)2 + 2 NatOH) = Na2{PbtOH)4}

Csolución)

Si (OH.) 3 + NaOH no reacciona

Acidificación de la solución con ácido nítrico a pH 5 - 6: Na2lPbtOH) 4>

+ 4 HN03 = PbfNO^ + 2 NaN03 + 4 H20 La acción del K2Cr04 en la solución Pb(.N03)2

+ K2Cr04 = PbCr04+ + 2 KN03

Las reacciones de precipitación sobre la cual la determina ción


cuantitativa del plomo en PbCr04 es:

2 PBCr04 + 6 KI + 8 H2S04 = 3 I2 + 2 PbS04 + 3 K2S04 + + Cr2 tS04)3 + 8 H20


I2 + 2 Na2S203 = 2 Nal + Na^Og

Porcentaje de plomo (Metal "C") en la aleación:

c(Na.S,0,). V(Na-S-0.)o M(Pb)


w(Pb) = • ¿ ¿ 5 ¿ ¿ 3

m (aleación). 3
2+ 2-CUn ion Pb corresponde a un ion de CrO^ , el cual acepta
electrones en la reacción redox considerada:

ITrpfO - 0.1512 molodm-3 . 0o01346 dm3 . 207„2 a.mol-1


Wt =
- ' ~ 1.286 g . 3 ----------- * ------

= 0..1429 = 14,99 %

Para convertir el hidróxido de Bismuto en fosfato es necesa rio Ca) disolver


el hidróxido de bismuto en un ácido;

Sl(.OH)3 + 3 HN03 = 3I(N0313 + 3 H20

CB). precipitar los iones bismuto con iones fosfatos: Bi(N03)3 + K3P04 =

BiP044- + KN03

Cálculo del contenido de Bismuto en la aleación:

M<Bi) = 209 g.mol-1 MtBiP04) = 304 g.mol-1

m(Bi) = 209 ^mofl • °'^76 <? = 0.3215 g 304 g.mol

Porcentaje de bismuto (Metal "D") en la aleación:


- 203 -

wCBi)
= i'Jseo l = °- 25QQ - 25 - 00 %

Composición de la aleación : Cd = 40%, Sn = 20%, Bi = 25%,


Pb = 15%.
204

PROBLEMA N°3 11° OIQ

Cuál de los procesos químicos ocurren en la interacción


de:

Ca) sulfato de amonio-aluminio con agua de barita;

(h) cromato de potasio, cloruro ferroso y ácido sulfúrico;

(c) soda calcinada e hidrosulfato de sodio;

(d) 4-bromoetil benceno y cloro;

Ce) n-propil alcohol, fenol y ácido sulfúrico concentrado?

Escriba las ecuaciones iónicas abreviadas para aquellas reacciones


que ocurren en solución acuosa. Para las otras reacciones escriba las
ecuaciones moleculares e indique él tipo de reacción. Para aquellas en las
que pueden formarse varias sus -tancias, indique las diferencias de las

S O L U C I O
N
Caí a - 1 Ba2+ + SO2,- = BaSO. 4-4
4
a - 2 NH* + OH" ---- ► NH3 . H20 ---- «- NH3+ + H20

a - 3 Al3+ + 3 OH" = A1C0H)3+

a - 4 AlfOHl3 + OH" ='IA1(0HI4}~

a - 5 posiBlemente Ha2+ + 2{A1(0H)4}" = BaÍAl(OH)2)4+

(51 B - 1 2 CrO2" + 2 H+ = Cr202" + H20


b - 2 7 + 7- +
3+ 3+ 6 Fe + Cr20^ + 14 H = 6 Fe + 2 Cr +
+ 7 H20
b - 3 concentraciones altas de Cl H2S04:

Cr202" + 4 Cl" + 6 H+ = Cr02Cl2 + 3 H20

condiciones de reacción.
- 205 -

(c) c - 1 exceso de H+: C02~ + 2 H+ = H2O.C02 = H20 + CC>2 + c - 2

exceso de C02~: C02~+ H+ = HCO~

(d) d - 1 sustitución por radicales libres (bajo exposición de luz o


por calentamiento)

o
Br Cl,
CH..C 2 3 Br -CHC1-CH3 + HCl
H & v

pequeña
-
cantidad de Br^ CH2CH2C1 policlorura

ción.

d - 2 sustitución electrofílica en presencia de un catli zadoro

Cl
Br C2H5 +
-<Q>- 2CH3 A1CI.
Ci2
^Cl

+ Br C2H5 + ci- -C2Hj y como productos

cr

colaterales de
C2H5
"C2H5
reacción, Cl-
Cl Br
Br C3H7OS03H + H20
H20
(e) e - 1 1 CH_,CH-CH_OH + H.SO, --
J ¿ Z ¿ 4 t

(C3H70)2S02 + H2°

2 C H_0H + H_SO.
3 7 o 2 4 C3H7OC3H7 (exceso de C3H7OH) +

+ H20

3 CH^CH^CH^OH g-gr* CH3CH=CH2 g^-. CH3CH(OH)CH3


"2 4
2 4
(en e-1 y e-2)
- 206 -

OH OH
SO-H
e - 4 + H2S04 +
S03H

-S03H

OH
OH S03H C3H7
+ C3H7OH poli
e - 5
O
C3H7
alkilación n- e iso-

e - 6 oxidación parcial al C^HjOH. y CgHgOH seguida de cor sensación o


esterificación.
- -207 -

PROBLEMA N° 4 11° OIQ

El compuesto "X" contiene nitrógeno e hidrógeno. Por ca lentamiento


fuerte de 3.2 g de "X" se descompone sin formar resi dúo sólido. La mezcla
de gases resultantes se absorbe parcial -mente por ácido sulfúrico Cel volumen
de la mezcla gaseosa dismi nuye por un factor de 2.81. El gas no absorbido,
el cual es una mezcla de hidrógeno y nitrógeno, bajo condiciones normales,
ocupa un volumen de 1 , 4 1 . y tiene una densidad de 0.786 g/1. Deter mine la
fórmula del compuesto "X"„

S O L U C I O N

Si la densidad de la mezcla de y Nj es conocida, su


composición puede ser determinada como:

0.786 . 22.4 o tn + 1) = 28 n + 2

de aquí n = ' 1 . 5 . La masa de la mezcla es 0.786 g l"1. 1.4 1 = 1.1. g.


Consecuentemente la mezcla de los gases absorbidos por el ácido sulfúrico (esos
gases debieran ser Nff^ y ^H^) tiene Tina masa molar media de:

3.2 g - 1.1 g . _±
---------------- . 22.4 1 mol = 18.6 7 g mol ,
1.4 1 . (2.8-1)

m-ientxas que NH^ corresponde a 17 g mol x.

Esto significa que los productos gaseosos absorbidos co rresponden


a una mezcla de NH^ y N2H"4. La composición de la fracción aEsorBiüa es:
- 208 -

Como retr.'atarto, la razón total de los componentes de la mezcla es


como sigues

N-H. + 8 NH- -í 2 N„ -í 2 H,, la cual coutisponde a Tina


¿ 4 J /¿

composición inicia] de gwsiajieif ^ de S : B> (2 + H 4] ¡ (4 + 2 4 + 4 ) = 1


Í 2.

La sustancia inicial es hidrazina N_H:..


- 209 -

PROBLEMA N° 5 11° OIQ

Un derivado de henceno X tiene la fórmula empírica CgH^-Su


bromuración en presencia de luz lleva a la formación de dos mo-noñromoderivados
de aproximadamente idéntico rendimiento. La bro muración en la oscuridad en
presencia de hierro también produce dos monobromoderivados, si la reacción se
lleva a un alto grado, puede ocurrir la formación de cuatro dibromo derivados.

Sugiera una estructura para el compuesto X y para los pro ductos


de la Bromuración. Esquematice las reacciones.

S O L U C I O N

El compues.-to de fórmula empírica CgBL^ puede ser:

C H
6 5"C3H7

^CH3
C6E4 11 C2H5
V

C6H3CaV3 111

Por la acción del Bromo en presencia de la luz sin cata lizador,


ocurre la bromuración de la parte alifática predominante en el car-bono unido
al núcleo aromático.

Cuando la ^reacción se hace en la oscuridad, en presen -cia de


hierro, éste se convierte en FeBr^ y cataliza la bromuración del anillo
aromático.

El compuesto X no puede sex I (pues se formaría un mono Bromo derivado


en presencia de luzl; tampoco pueden ser los isó-
- 210 .-

meros Illa, IIIB„

Illa, solamente un monobromo derivado es


posible en la bromu-ración de los
CH CH
3 3
grupos- CH3;

Illb. bajo las mismas condiciones son


posibles tres monobromo
derivados.

H2CH3

lie

Asi la selección debe sex hecha de las siguientes es tructuxas:

lia Ilb

La condición para que dos monobromo derivados se f or men en la


oscuridad se cumple con las estructuras lia y- Ilb; 1 condición de la
posibilidad de cuatro di&romo derivados elimi ma la estructura lile De
aquí, la única posibilidad del com ponente X es lie.
- 211 -

esquema para las reacciones de bromuración es:

C2H5
- 212 -

PROBLEMA N° 6 11° OIQ

130 g de un metal desconocido fueron tratados con un ex ceso de ácido


nítrico diluido. A la solución resultante se le~a gregó un exceso de una
solución alcalina caliente y se despren -dieron 1.12 1 de un gas Cen
condiciones normales). ¿Cuál fue el metal que se disolvió en ácido nítrico?

S O L U C I O N

El gas liberado durante la reacción con la solución alcalina es


amoníaco„ Por lo tanto, uno de los productos resultan tes de la disolución
del metal de la pregunta en el ácido es nitrato de amonio. Así, las ecuaciones
de la reacción tendrán la formas

SUs + 10n HNO, = 8Me(N0,) + n NH.NO_ + 3n B\,0


o o n 4 3 ¿

n NH.NO-> + n NaOH = n NH_ + n H.O + NaNO,


4 3 3 2 3

De aquí el esquema;

X 1.12 1
n NH
8 Me 3
8 A g n . 22.4 1
r

Donde n es la valencia del metal (número de oxidación


de Men+J y A es la masa atómica relativa del metal. 1 r

8 A g - 22.4 . n 1
r
13 g - 1.12 1
A = 3-3 g . 22.4 1 . n = 5
Ax = 32.5 . n r 8 g . 1.12 1 n
n =1, h% " 32.5 - ninguna
- 213 -

n = 2, A = 65 -
zinc ' r

n = 3 , A^ = 97.5 ninguno n =

4, A = 130 ninguno

Respuestaiel metal es zinc.


- 214 -

PROBLEMA N° 7 (práctico) 11° OIQ

10 tubos de ensayo numerados de 20 mi cada uno, contie nen soluciones


0.1 Ji de las siguientes sustancias: cloruro de ba rio, sulfato de sodio,
cloruro de potasio, nitrato de -magnesio , ortofosfato de sodio, hidróxido
de bario, nitrato de plomo, hi -dróxido de potasio, sulfato de aluminio y
carbonato de sodio. U-sando solamente estas soluciones como reactivos,
determine en cuál de los tubos de ensayos numerados se encuentra cada una de
las sustancias arriba citadas.

Diseñe un plan de análisis y escriba las ecuaciones de las


reacciones a ser realizadas„ No olvide de dejar por lo me -nos 2 mi de la
solución en cada tubo para control. Si en el cur so del análisis se necesita
una cantidad adicional de solución , TJd., puede pedirla a su profesor quien
se la proporcionará, pero en tal caso TJd. perderá algunos puntos.

-------------------------- S O L U C I O N --------------------------

Usando la tabla, el total de los problemas no pueden ser solucionados


de una vez; todos los precipitados son blancos y hay sustancias que forman
el mismo número de precipitados. Del nú mero de los precipitados solo KC1 (1),
Mg(N03)2 (4) y Pb(N03)2(8) pueden ser determinados inmediatamente.

Posteriormente Na2S04 y KOH (dan tres precipitados cada uno) pueden


ser determinados por medio de la reacción con Mg(N03)2 - Mg(OH)2; Ba(OH)2 y
Al2(S04>3 (dan 6 precipitados ca da uno por medio de la reacción con KOH _ -
A1(0H)3; BaCl2 , Na3P04 y Na2C03 (dan cinco precipitados cada uno): primero, la
reacción con Na2S'04 indica BaCl2. Entonces la reacción BaCl2 -A12CS04>3 produce
A1C13 (después de filtrar el precipitado de BaSO.) . El desprendimiento
de C0- y la formación de Al (OH)., en
- 215 -

la reacción con la solución de A1C1- indica Na.CO.

as

ÍN OCO U (N O tN nO2 O m
O« ÍN« cumCO2 O CO
mO
Di S COIB ¿a ÍNi-H<
OCJ
ÍNtO2
PM
BaCl2 4- 4- 4- + 4- 5
- - - -
Na-SO. 2 4 4-
- - - 4- 4-
- - - 3

KC1 4- — — 1
- - - - - -
MgCN03) + 4- + 4- 4
2
- - - - -
Na-PO. 3 4 4- 4- 4- 4- 5
+
- - - -
Ba(OH) + 4- + + + 4- 6

- - -
Pb(N03)2 + 4- + 4- + 4- 4- + 8
-
KOH 4- 4- 4- 3
- - - - - — 6
A12(S04)3 + - 4- 4- 4- 4- 4-
- -
Na2C03 + + 4- + 4- 5
- - - -
Determine la masa del permanganato de potasio en solución que se
le ha dado: se le proveerá con ácido clorhídrico de concentración dada y una
solución de KOH de concentración deseo nocida, una solución de ácido oxálico
de concentración desconocida y una solución de ácido sulfúrico (2N).

------------------------- S O L U C I O N -------------------------

Equipos y reactivos: una bureta para titulación; indicadores


(anaranjado de metilo, litraus, fenolftaleína...); pipetas de 10 (15 ó 201
mi de capacidad; aforados de 250 mi de capa cidad; 2 vasos de precipitados
de 100 - 150 mi de capacidad para la valoración»
XII. OLIMPIADA INTERNACIONAL DE QUIMICA

LINZ 1980

AUSTRIA
- 217 -

PROBLEMA N° 2 12° OIQ

La disociación (molecular) de cloro es un proceso endo térmico,


AH = 243c6 KJ„mol ^0 La disociación puede llevarse a efecto también por la
luz.

1! Pregunta: ¿A qué longitud de onda de la luz puede obtenerse el efecto de


disociación?

2°. Pregunta: ¿Puede obtenerse este efecto con una luz cuya longitud de onda
es más pequeña o más grande que la longitud de onda crítica
calculada?

3! Pregunta: ¿Cuál es la energía del fotón con longitud de onda crítica?

Cuando la luz que puede lograr el efecto de disocia ción del cloro
incide en una mezcla gaseosa de cloro e hidrógeno, se forma ácido clorhídrico»
La mezcla se irradia con una lámpara U.V. de mercurio (A = 253.6 nm) „ La
lámpara tiene un poder de entrada de 10 watts„ Una cantidad de 2 % de la ener
-gía suministrada es absorbida por la mezcla gaseosa (en un red piente de
10 litros). Después de 2.5 segundos de irradiación se forman 65 milimoles
de HC1„

4; Pregunta: ¿Cuan grande es el quantum obtenido (= al número de moléculas


de producto por fotón absorbido).?

5o Pregunta: ¿Cómo puede explicar el (cuantitativamente) el valor obtenido?


Describa el mecanismo de reacción.
- 218 -

PROBLEMA N° 2 12° OIQ

S O L U C I O N

1. X , AH = N, h o sigue que

c.NA.h ñ ?3 -34
3 . 10 . 6.02 .10 . 6.6 , 10 . , Q-,
AH -,n-7 „
---------------- = ----------- = 4. 91 . lu n
2.436 . 10
= 491 nm

2. La luz de onda corta es efectiva, pues sus fotones tienen Ti-


na energía mayor que la requerida, mientras que los fotones
de longitud de onda más larga tienen una energía muy débil pa-
ra los efectos de disociación.

— 34 ñ
3. E = h . v = h_^_£ = 6.6 • 10 • 3 • 10 = 4.03 . 10"19
J
1 1 Al 4.91 . 10

, . __ , .-, el número de moléculas de HC1 formadas


4. El quantum producido = --------- ,-- 7 ---- = - h—r ----- r- -- r-^ ----
^ el numero de fotones absorbidos

0 nHCl . NA 6.5 . 10~2. 6.02 . 1023


= = g ± ±Q4

Etot 0.2 . 2.5


-34 í 6.6 . 10 .
3 . 10
X 2
2.536 . 10 7

la energía de entrada = 1 0 . 0.02 = 0.2 W

5. El quantum producido observado se basa en un mecanismo de ca dena.

El inicio de la reacción en cadena Cl2 + H", v ---- ► 2 Cl°

La propagación de la cadena Cl" + H2 ----- >- HC1 + H"

H" + Cl2 ----- «• HC1 + Cl'

El término de la cadena principalmente por:


2 H" ---->- H
2
2 Cl* ---- >■ Cl 2
HC1
H* + Cl" ---- ►
- 219 -

Equilibrio del vapor de agua

La reacción gaseosa homogénea


C02(g) + H2(g) -- ► CO(g) . K20(a)

Se le denomina la reacción del gas de agua

a) Calcular la energía de la reacción de Gibbs AG°QQQ para la reac ción de


gas de agua a 1000K de la reacción de entalpia, ^^000 = 35040 J.mol"1 y
la reacción de entropía, AS°00Q= 32.11 J.mol-1. K-1.

b_). ¿Cuál es el valor de la constante de equilibrio, K , de la


reacción de gas de agua a 1000K?

c). ¿Cuáles son los valores de las constantes de equilibrio, K y


-1 x
K (x: fracción molar, c; concentración en mol.l ) a la mis
ma temperatura (1000K)? (nota: el gas se comporta como un gas
ideal). .

dj. Una mezcla gaseosa contiene 35% vol. H2, 45% vol. CO y 20% vol de vapor
de agua se calienta a 1000K. ¿Cuál es la composición de la mezcla después
que se establece el equilibrio del gas de agua?

e). Calcular la reacción de entalpia AH£4QQ del valor de la reacción de entalpia,


AH?rt__ y los valores de los calores molares,
j-UUU
C° (válido para el rango de temperatura, 1000K - 1400K)
ir

*HÍQO0 = 35(140 J

C°(C02) - 42.31 + 1Q„Q9 . 10-3T J.mol"1 . K_1 C|C.H )

= 27.40 + 3„20 . 10~3T J.mol"1 . K_1


- 220 -

PROBLEMA N° 2 12° OIQ

C°(CO) = 28.34 + 4.14 . 10 3T J.mol 1 . K X

C|(H20) = 30.09 + 10.67 . 10-3T J.mol-1 . K_1

se cumple que:

B 2 2
/ (c1+ c2x)dx = c1 (b-a) + 0.5c2(b - a )
3.

f) ¿Qué se puede decir, sobre la base de los valores encontrados para AH°,del
desplazamiento del equilibrio del gas de agua con el aumento de la
temperatura?

S O L U C I O N

AH =
3Í 1QQQ 35040 J
AS1000 = 32 •11 J-iaol~1 ° K-1

AGÍ000 = AH°- TAS°= 35040 - 1000 . 32.11 =


35040 - 32110 = 2930 J

B) 1G° =
- RT Zn K
P
4n K
A C ° 2930 p = ' -W" = -
83TI = - 0-352418

R = 8.314 J.mol-1 . K_1

K = 0.7030
P

c) Como el número de moles- no cambia en la reacción, la reacción


es independiente de la concentración y de la presión; por lo
tanto, K = K = K (sin dimensión). La fracción de volumen x
p c
y la fracción molar son idénticas en un gas ideal.

di. La composición original del gas,


XO,CO=0-45, XO,H =0-35, *O,H2O=0-20, XO,CO2=0-00
2

Si la fracción de CO., formado en el equilibrio se designa por x, entonces las


concentraciones en el equilibrio se obtienen de:

C0: X0,C0 ~ x C02: x

H20; ^0, H 20 " x H2:X0 ,H2+X

XC0 XH 0 U0,C0~X) , H 0~ X
(X0
- 221 -

K = K = ------ — = ------- 7 ---------- = -- = 0.703


XCX0, H
P X XC0
2
XH
2 2 + ^

o,ccrx)- i x o, n 2 o~ x )
& = K(X
O , H 2+x)x' ahora K = K
x

2 2 ^0, C O ^0,^0
X(X0 , H 20+X0,C0' + X _ K ^O.VL + K X

^2(1-K)-X(X0, H 20+ X
0,CO + K X
0, H 2 5 + X0,C0 . X0, H 20 = 0
sustituyendo los valores

numéricos,

x2 (1-0.703)-x (o.20+0 .45 + 0.703 . 0 „35)+ 0.45 . 0.20 = 0 0.297 x2

-0o89605x + 0.09 = 0 x2 -3.01703x + 0.303030 = 0

x1 , = 1.508515 - /2o275618-0.303030 = 1.508515 = /l.972588 x = 1,508515

- 1,404488 = 0.104027

(El signo más indica una solución que no tiene significado fí_ sico, ;x > 1).

x- - CL1Q4

^CO =0-346, xCQ2 = 0.104


x = 0.096, x„ = 0o454
H2° H2

e) AC° = C°(CO) + C°(H-0) - C°(CO_) - C°(H„)


P P p 2 p 2 P - 2

= - 1 1 .28 + 1 „ 5 2 „ 1 0 _3T JoK


_1inol":L

Í H Í 4 0 0 = Í H l 0 0 0 + ^ [ AC P ÍT =AH1000\ ^ ( C 1 + C2T)dT

=AH°Q00+c1( 1 4 0 0 - 1 0 0 0 ) + 0 „ 5 c 2 ( 1 . 9 6 . 1 0 6 - 1 . 1 0 6 ) ; =AH°Q00-llo 2 8 „ 400 +

1.52 „ 10-3 . 4.8 . 105)

^ioQO"4512 + 729 *6 =

=35040 -4512 + 729.6 = 31257.6

1 H 14Q0 = 3 1 2 5 8 J

f)_ Sobre la base de la reacción isóbara de van't Hoff


- 222 -

PROBLEMA N° 2 12° OIQ

3 1n K AH
______ P _ __
"3 T RT2

Kp aumenta con el aumento de la temperatura para (endotérmico) calores


de reacción positivos: i.e. el equilibrio se desplaza con el aumento de
temperatura en favor de los pro -ductos de la reacción CO y H 2 0.
- 223 -

(Química de iones, estequiometría, reacciones redox)

Un sólido cristalino blanco exhibe las siguientes reac


ciones:

1. La llama del mechero Bunsen se colorea intensamente de amari lio.

20 La solución acuosa es neutral; el agregado gota a gota de á-cido sulforoso


(una solución de SO2) produce una solución de color café oscuro, la cual
se decolora por la presencia de un exceso de ácido sulfuroso.

3. Si una solución de AgNO^ se agrega a la solución incolora ob tenida en


2. y se acidifica con HNO^, se obtiene un precipitado amarillo que es
insoluble en NH_, pero se disuelve rápi
- 2-damente al
agregarle CN 6 S203 .

4. Si una solución acuosa del sólido se trata con KI y H.,S04 di^ luido, se forma
una solución café oscura que puede ser decolorada por la adición de
soluciones de ácido sulfuroso ó Na2S20.

5. Una cantidad de 0. .10.00 g de este sólido se disuelve en agua, se le agregan


0.5 g KI y unos pocos mi de H.,S04 diluido, se forma una solución café oscura
la cual se valora con una solución de 0.1000 M de Na2S203 hasta que la
solución se decolora completamente; se gastaron 37.40 mi.

a). ¿Qué elementos contiene el sólido?

bí ¿Qué elementos pueden ser considerados presentes en base a las


reacciones J_. a 4.? Calcular su peso molecular.
- 224 -

PROBLEMA N" 3 12° OIQ

c) Formule las reacciones correspondientes de 2 ) a 4 ).para los


compuestos considerados y escriba las ecuaciones en forma
iónica.

d) Decida sobre la base de 5 ) cuál es el compuesto presente

------------------------.— S O L U C I O N ----------------------------

a) El soluto debe contener Na e I: la coloración amarilla de la


llama del mechero indica la presencia de Na; una sal amarilla
de plata que se disuelve solamente con agentes acomplejantes
_ T -
fuertes tales como CN 6 5^0^ , debe ser Agí.

b) Las reacciones 1 ) a 4 ) indican una sal de Na con contenido de oxígeno


en un ácido que contiene yodo;

Arabos S02 e I~ se oxidan, mientras que en el primer caso I se forma como


un intermedio de I2 (ó 1^ , solución café) y en el segundo caso se forma
I2 (ó 1^).
Como la solución es neutral hay que considerar que es NalO^ 6
NalO.. 4

J«CNaI03I = 22„99 + 126.905 + 3'16.000 = 197.895 = 197„90g/mol

M(NaI04í = 22.99 + 126.905 + 4"16.000 = 213.895 = 213.90g/mol

C) 2 IOT + 6 H_0 + 7 SO- = 7 HSOT + 5 H+ + I-


4 2 2 4 2
2 I0~ + 4 H20 + 5 S02 = 5 HSO* + 3 H+ + I2
I- + SO-, + 2 H-0 = HSOT + 3 H+ + 2 I~
2 2 2 4

1Q4 + 7 X~ + 8 K+ = 4 I2 + 4 H20

ia~ + 5 I_ + 6 H+ = 3 I2 + 3 H20

I2 + 2 S 2 0 2 3 - = 2 i" + S402"
- 225 -

PROBLEMA N° 4 12° OIQ

-3 2-
d) Experimento: 0 .1000 g del compuesto.. 3.740 . 10 moles

1. Hipótesis: El compuesto es NalO^


2-

1 mol NaI03 197„90 g NaI03 6 moles S203

0„10QO gNaI03„„. ^fy^Tj^ 3.032 '10~3 moles S203~ La hipótesis es

falsa.

2. Hipótesis: El compuesto es NalO^


2-

1 mol NaI04 o „ o o 213.90 g NaI04 8 moles S203

Q.10.0Q g NaI04.„. °'^^gQ8- 3.740'10-3 moles S^2-El compuesto es

NaI04.
- 226 -

(Química orgánica, estereoquímica)

Un ácido carbónico A con una fórmula global C_H„0- pro duce dos
isómeros geométricos, cis-A' y trans-A", Por hidroge-nación con Pt/H2, se
obtiene el mismo ácido racémico de los dos estereoisómeros que pueden
separarse en los enantiómeros ( + )-B y (-)-B. A! y A" reaccionan rápidamente
con un mol de bromo en CCl^ en la oscuridad y a 20"para producir C.

a). ¿Cuál es la constitución de A y B?

b) Escriba la estéreo-fórmula par A' y A" y la proyección de Fischer para


los enantiómeros de B (sin considerar los sig -nos (+) ó (-).

c) ¿Cuántos estereoisómeros de C se forman simultáneamente cuan do A' ó A"


son tratados con bromo?

d) Brevemente dé las razones para sus respuestas de c)

e) Escriba las fórmulas de proyección de Fischer y las fórmulas de proyección


de Newman (conformación) para todos los isómeros de C.
Destaque aquellos que son mutuamente enantioméricos y diaste
xeoisoméricos.

----------- ■ ------------ S O L U C I O N --------------------------

a) A CíLj-CH = B CH3-CH2-CH.(;CH3).C00H
C(CH31C00H
- 227 -

PROBLEMA N° 4 12° OIQ

COOH COOH
CH3" -CH3
•H H

| I CH3 CH3

c) Siempre 2 (vea e) : 1 a 4_

d) La adición de bromo a los alkenos da "trans" en las condiciones dadas.


2
En la adición, produciendo 2 = 4 estéreo isómeros de los cua les 2 son
mutuamente enantioméricos„

e) De cis-A' COOH COOH

CÍLy -Br Br CH-

H - -Br -B

r •

CH. CH.
2 6 3 y 4 son enantioméricos. 1 a 3 y 2 a 3 y 4 son

diastereccméricos „
- 229 -

PROBLEMA N° 5 12° OIQ

(Química inorgánica)

De 20 mg de disilano parcialmente metilado Si^g^íCH^)^, se


desprendieron 27.8 mi de hidrógeno durante la hidrólisis alcalina a 294 K
y 97400 Pa.

a) ¿Porqué el enlace Si-Si reacciona durante la hidrólisis?

B) ¿Porqué los enlaces Si-H del disilano reaccionan durante la hidrólisis?

c)_ Calcular el grado de sustitución x del disilano metilado.

di Escriba la reacción para la reacción completa de la hidróli -sis.

e) ¿Cuántos isómeros pueden ser calculados del compuesto formado? Dé la fórmula


estructural para cada isómero.

S O L U C I O N

a) El enlace Si-Si es una coordinación insaturada y por ello tie ne la tendencia


a reaccionar con reactivos nucleofílicos con ruptura de enlace.

b) Similarmente para todos los compuestos con hidrógeno polariza do


negativamente, este enlace también reacciona con los proto nes del agua con
formación de hidrógeno elemental.

c) tCí^Si^ 2. . 28.086
M2 Si
(6-x)H (6-x)'1.008
x CH 15.035

3
PROBLEMA N ° 6 12° OIQ

56.172 + 1.008 (6-x) + 15.035 x

Peso molecular = 62.22 + 14.027 x

20
Peso de la muestra 20 mq
'
= 7?-^=— = m mol
^—=r=—;—=-¡—
62.22 + 14.027 x

Hidrógeno desprendido v

n = P ° „ m mol H. (V en mi)

„ - 0.974 . 27. 8 .
n " 0.08314 . 294 m mo1

(SiH) (SiSi)

(6 - x + 1) . 20 _ 0.974 . 27.8
62.22 + 14.027 x 0.08314 . 294

x = 1.. 9.999 De aquí, grado de sustitución

= 2

d) sSi - H + H20 -^2-^- = Si -- OH + H2

3SÍ ---- Si= . 2 H20 — ► 2 =Si ---- OH + H2

De aquí (para un isómero simétrico)

Si2H (;CH"3) 2 + 6 H20 ----- ► 2 Si (OH) 3CH3 + 5 H2/n

n Si2H4(CH3)2 + 3 n H20 --------- * ÍSi2°3(CH3)2^n + 5 n H2

e) 2
H H CH, H
I I I
3 1
CH_ - Si - Si - CH- , CH, - Si - Si - H
3 I I 3 3 | |
H. H H H
Zn Si(OH)3CH3 -------- ► ísi2°3(CH3)2}n + 3 n H2°
- 231 -

(.Química orgánica, síntesis)

El benzaldehido y el ácido malónico en piridina a 80°C, producen


entre otros, COj y un compuesto A con un rendimiento aproximado (ca.) de 80%
del valor teórico. La hidrogenación ca talítica de 1.48 g de A en Pt, a
temperatura ambiente y presión normal produjo B con un consumo de 0.25 litros
de hidrógeno. La reacción de B con un ácido polifosfórico (en las condiciones
de la reacción de Friedel - Crafts), produce el compuesto C,el cual puede ser
aislado, acompañados de dos subproductos isoméricos a cidicos. Los
subproductos Da y Db pueden formarse en gran cantidad en concentraciones altas
de B en el medio de reacción y puede impedirse por dilución.

El análisis elemental de C produce 81.1% de carbono y 6:01%


deshidrogeno; los valores correspondientes para Da y Db i-dénticos: dentro
del error experimental, son 76.6% y 6.4% respec ticamente o Una cantidad de
2.82 g Da, también como de Db, requiere c a o 100 mi de hidróxido de potasio
0.1 N para su neutralización. C puede ser purificado por destilación (punto
de ebullición 234 - 245°C, P") y luego tiene un punto de fusión de 40°C;
3
su densidad es 1.0.9 g/cm . El peso molecular puede obtenerse por
espectrometría de masa y es 132.

Usando esta información, resuelva los siguientes pro -

.. Blemas:

1. La fórmula estructural de A

2. La fórmula estructural de B

3. La fórmula estructural de C

4. La fórmula estructural de Da y Db.

5. Dé una forma alternativa para la síntesis, de A usando los ma-


PROBLEMA N ° 6 12° OIQ

teriales iniciales más simples de partida y formado al menos un enlace


C-C.

6. Dé una forma alternativa para la síntesis de B usando los ma teriales


iniciales mas simples y formado por lo menos un enlace C-C.

7. Dé la fórmula estructural para los productos de las siguientes


reacciones:

a) C + hidroxilamina (con catalizador ácido) --------- ►

b) C + bromuro de fenilmagnesio (CgH5MgBr) y subsecuente tra


tamiento bajo condiciones acidas -------- ■*■

c) C + benzaldehido + CoHc09Na® -------- ►

S O L U C I O N ------------

COOH
1. Condensación

2. Hi'drogenación

ácido polifosfórico '' "H2°


3. Ciclizacrón intramolecular
- 233 -

En adición a C

H2CH2COOH
CH2CH"2-C

CH2CH2COOH

Da (Isómeros posicionales) Db se
forman.

e.g„ Reacción de Perkin: Tratamiento de benzaldehido con anhí_ drido


acético,,

-CHO + (CH3CO)20 A + CH3COOH

e.g. por síntesis con ácido malónico.

C H O^Na® /(^)-CHnCH(CCCC0H.) +
■CH2C1 + CH2(COOC2H5) ------- - ----- ► y-y 2 2 5
2
+ NaCl
a) NaOH
-C2H5OH
b) HC1,
A -C0„

Reacciones a),b) y c) son reacciones típicas del grupo carbo nilo

a) NOH b) HO C,H,
C,HC
/
OI> Q
6 5
-H20
c) 6 5

(E)
dos estéreo
isómeros (syn y anti). dos estereoisómeros
Análisis Orgánica cualitativo»

En cuatro tubos de ensayo, 4 sustancias diferentes que se


encuentran en la naturaleza están presentes. Encuentre dos sustancias que
- 234 -

PROBLEMA N° 7 12° OIQ

forman compuestos básicos del heno seco y de edi_ bles para humanos. Solamente
dos de esas sustancias serán iden tificadas. Proponga los nombres y las
fórmulas para las dos sustancias, sobre la base de ensayos de combustión,
experimentos de solubilidad, identificación de los grupos funcionales y la de
-terminación del punto de fusión.

Como ayuda se puede usar lo siguiente: Una tabla de pun tos de


fusión. El aparato de Thiel para la determinación del punto de fusión, un
esquema de solubilidad y los siguientes reacti vos:

dietil éter, NaHC03 (5%), NaOH (2 M) , HCl (2M) , H2S04 conc, H-jP04
conc.^etanol, reactivo de Tollens (una solución amoniacal de Ag],
solución de Fehling I y II, clorohidrato de fenilhidra-zina, B-naftol, NaNO^
sólido, Ca(OH)2 sat„, FeCl3 (5%), hielo , 2,4-dinitrofenil hidrazina,
solución de ninhidrina (1% álcali -na), reactivo de Seliwanoff
(resorcinol/HCl), florogiucina.

Se requiere: Una descripción exacta del experimento , las


ecuaciones (o esquemas de las reacciones cuando las ecuacio nes no puedan
darse) que se necesitan para la identificación , los nombres y las fórmulas
estructurales de las dos sustancias problema.
Determinación del punto de fusión por medio del aparato de Thie-le.

Una cantidad de muestra finamente pulverizada se pone dentro de un capilar


que tiene un extremo sellado, hasta una altura de 2-4 mm. Para llenar el capilar
hay que introducirlo en lamuestra. La sustancia es cuidadosamente barrida
de las paredes del capilar y por golpes suaves se le hace llegar al fondo
del capilar. Luego el capilar se coloca en la abertura del aparato de tal
ma ñera que la muestra esté a la altura de la cabeza del termóme -tro de
mercurio. Un transmisor de calor, un aceite de silicona adecuado de alto punto
de ebullición se utilizan en el aparato.

Para determinar el punto de fusión de una sustancia orgánica deseo nocida se


busca primero un rango aproximado de fusión. De tal modo que el calentamiento
se hace de acuerdo a la figura, alrededor de 5°C/min. Para una determinación
exacta, se prepara o-tra muestra y se eleva la temperatura hasta 10°C más
bajo que el rango de fusión determinado a razón de 5°C/min, seguidamente la
temperatura se eleva más lentamente, 1 - 2°C/min hasta lograr la fusión. Como
punto de fusión se toma la temperatura a la cual la sustancia se ve claramente
fundida.
- 235 -

Apéndice 2

Tablas de puntos de fusión (tod


PF PE
ALDEHIDOS PF PE
Acroleína - 52
Pentanal - 103
Furfurol - 161
Benzaldehido - 179
o-Clorobenzaldehido .11 214
Salicilaldehiao - 196
a-Naftaldehido 34 Vanillin 81
o-Nitrobenzaldehido 44

p-Dimetilamino-

benzaldehido 74

Propanol-1 - 97
ALCOHOLES
Etilenglicol - 197
t-Butanol 25 82
Butanodiol - 230
n-Pentanol - 136
Glicerina - 290
Ciclohexilalcohol - 160

Trifenilcarbinol 165

Morfolina -
AMINAS 130

Dretilamina - 56 a-naftilamina 50 300

Ciclohexilamina - 134 p-Bromoanilina 66


n-Nitroanilina 114
Anilina - 184
Difenilamina 54 p-Aminofenol 186Z

o-fenilendiamina 102
s las temperaturas en °C)

ACIDOS Ac. co Ac. Esteárico

Palmíti 63 70
- 236 -

Apéndice 2

ac. ico ac. Oleíco 186

n-Valér 14 222/13

Ac. oxálico (.2^0) Ac. 101 - Ac.mandélico 118

acetilsalicílico Ac. 135 - Ac. benzoico 122

ftálico Ac. antranílico 203 - Ac. malónico 135

Glicina 146 - Ac. S-naftónico 185

232Z - Ac» p-hidroxibenzoji


co. 215
DERIVADOS HALOGENADOS

n-Butil bromuro

Ciclohexenil yoduro 100 p-diclorobenceno 53 179

Tricloroetileño p-bromotolueno 28 185 6 7

Hexaclorobenceno 2 30
CETONAS

Dietil cetona
Ciclohexanona
10 2 Metilisobutil cetona - 118
Benzofenona 156 Acetofenona 20 202

Benzil 4 9 - p-Brcmoacetofenona 51

95 - dl-Alcanfor 178
CARBOHIDRATOS

d-RíBosa

q-d-3rucosa 957 - e-Maltosa 165

Sacarosa 146 Z g-d-Fructuosa 104

180 - a-Lactosa 223


HIDROCARBUROS

n-Heptano

cis-Decalina 99 Penteno-2 - 36

Cumol 194 Ciclohexeno - 84

Antraceno 151 Difenilo 70

216 - Estireno - 146


PF PE PF PE

Tiofenol 126 169


MERCAPTANOS-TIOFENOLES
p-Tiocr p-Bromo
n-Amilmercaptano
esol
- 237 -

tiofenol

200

74

FENOLES

p-Cxe.sol 36 200 o-nitrofenol 45

a-Naftol 94 - Resorcinol 110

Pirocatecol 10 5 - 8-Naftol 123

A. Plcrico 122 - Floroglucina 218

DERIVADOS ACIDOS Bromuro de


77 Acetamida 82
acetilo Cloruro de ac. butírico
102 n-Metilacetanilida 102
4-Nitrobezoil-cloruro 7 Etil
Urea 132
ester ac. Butírico Dietil ester
121 Formiato de sodio 255
ac. malónico -Cetil ester ac.
199 Al-Acetato 200-320Z
palmítico 54
Ba-Propionato ca.300

Z después del número significa punto de descomposición.


- 238. -

Apéndice 3

En H20

soluble insolubl
e

en 5% NaOH

soluble insoluble

en 5% HCl

en éter en 5% soluble |
NaHCO insoluble
,

en H_S0, conc.

r soluble insolubl
e
en H,PO,conco
soluble insol. sol. insol. soluble insolubl
e

52 ^ A2 B M N2 I

: Sustancias de volatilidad más alta:


Todos alcoholes de bajo peso molecular, aldehidos, cetonas , ácidos, aminas,
nitrilos y cloruros ácidos.

S2 : Sustancias con volatilidad más baja, a menudo destilables sin des_ composición:
polialcoholes, sales , hidroxialdehidos y cetonas , carbohidratos, amino e
hidroxiácidos.

: Sustancias con volatilidad baja: ácidos de alto peso molecular,ni trofenoles.

A2 : Sustancias con alto punto de ebullición: fenoles, nitrocompuestos primarlos y


secundarios, sulfonamida, ácidos débiles.

B ; Sustancias con alto punto de ebullición, destilables por arrastre con vapor de agua:
compuestos básicos, aminas (con un máximo de
- 239 -

grupos arílicos), hidrazina.

M : Sustancias poco volátiles:


Compuestos neutros, nitrocompuestos, nitroanilina, azo- y azcrxi-
compuestos, nitrito-, nitrato-, sulfúrico-, y esteres de ácido
fosfórico.

N^: Sustancias con pequeña volatilidad; alcoholes, aldehidos , metilcetonas


y esteres con menos de 9 átomos de C, compues_ tos neutros, éteres,
definas.

N2: Sustancias con muy pequeña volatilidad:


Alcoholes, aldehidos, cetonas, esteres y tioalcoholes con -mas de 9
átomos de C, compuestos neutros, éteres, definas.

I : Sustancias con bajo punto de ebullición:


Compuestos inertes, hidrocarburos, alcanos halogenados.
- 240 -

Apéndice 4

Preparación de los reactivos

Reactivo de Tollen:

Mezclar 0.5 mi 2M NaOH t 1 al CL1M AgNO^ en 2M NH^ Soluciones

de Fehling:

I s J..75 g CuS04 cristo en 25 mi H20

II : 8.5 g sal de Seignette + 2,5 g NaOH en 25 mi H20 Reactivo ñe

Seliwanoff:

125 g de resorcinol se disuelven en 25Q mi HCl dil. (83 mi HCl conc. + 167 mi
H2OJ_, preparar solamente la cantidad necesaria.

Solución de Fenilhidrazina:

d.5 g de fenilñidrazina nidrocloruxo + 0.5 mi acido acético gla cial se agitan


con dos-m-l 3e H20 has-ta obtener una solución clara.

Solución áe 2,4-ainitrofenilñidrazina:

A 0..4 5 de 2,4-dinitxofenilhxdrazina se le agregan 2 mi H^SO^ conc. y luego


agitando y revolviendo se le añaden 3 mi de H20. A la solución tibia se le agregan
10 mi de alcohol de 95 % (eta nolj..

------------------------- S O L U C I O N -------------------------

En los cuatro tubos de ensayo están presentes: cloruro Se sodio puro,


D-fructuosa, ácido palmítico (ácido hexadecanoico) y--vainillina.
D-fructuosa es el bloque basal de la caña de azúcar y- el ácido palmítico, se
identifica como el bloque basal de la mayoría de las grasas vegetales y animales.
Rango de fusión : 120. - 105°C

Fructuosa

Ensayo de combustión: carbonización durante la combustión con olor a caramelo.

Solubilidad: rápidamente soluble en agua, isoluble en dietil éter.

Identificación de los grupos funcionales: Grupo

aldehidico :

1. Con el reactivo de Fehling.

La oxidación del grupo carbonilo con reducción simultánea de Cu(II) a Cu(I)„

^ z- - // ?-
-C + 2{Cu(C„H„0,) _} + 5 OH ■+ -C + CU-0+ +3HnO+4C.H.O^
4 4 o ¿. ^o" 4 4 b
H

2.. Con el reactivo de Tollen

La oxidación del grupo carbonilo con reducción simultánea de Ag Cl 1 a Ag

C + 2 {Ag (NH_) _ } + 2 OH ■+ -C + 2 Ag + 4NH, + H_0


\ 2 2 ^ 3 2
H O"

Formación de Osazona (indicación de monosas

CH2OH HC = 0 HN-NH2 HC=N-NH-^ ^

C = O HC — OH + 3
i: H -------------------------- ► HC=N-NH-<^ ^\4C6H5NH2+NH3+2H.
ketosa aldosa fenil- H osazona
hidrazina

Ensayo para ketohexosas (reacción de Seliwwanoff)


Ketohexosas forman por calentamiento en solución acida,5-hidroxi
- 242 -

metilfurfural que condensa con resorcinol dando una sustancia coloreada.

CH-OH
I2
c=o * coloración roja
HO-CH -3H O I
HC-OH CHO
HOCH2
'¿
HC-OH

CH2OH

Ensayo pra pentosas (reactivo de Tollen)

Las pentosas en contraste con las hexosas forman furfural en solución acida
que condensa con floroglucina dando sustancias coloreadas
HO ~ OH
de rojo. coloración roja
CHO
I
HC-OH: I -3H20
HC-OH I
HC-OH: I
CH^OH

Nombre: Las reacciones arriba dadas, la conducta de solubilidad y el rango


de fusión. Indique que esta sustancia es la D-fructuosa, el
bloque basal de la caña de azúcar.

CH.OH.
I 2
C=0

HO-CH I (Cuando dé la configuración


HC-OH: use la proyección de
I Fischer).
HC-OH; I
CH^OH
:
Rango de fusión : 60 - 6 3°C

Ensayo de combustión: Se quema con una llama amarillenta ligeramente


Humosa =

Solubilidad: insoluble en agua, muy soluble en 2 MNaOH, me -nos soluble


en 5% NaHCO^ o

Identificación de los grupos funcionales: De la solución al-


2+
- 243 -

Acido palmítico

calina una sustancia incolora precipita con Ca

2R-C00H + Ca2+ ----- + (R-COOl2Ca

Sobre la base de la llama de combustión, la conducta de solubilidad y la


precipitación de una sal de calcio de la solu ción alcalina, la sustancia
es un ácido orgánico. El rango de fusión indica ácido palmítico = ácido
decanoíco.

CH3(CH2)14 -COOH
- 244

PROBLEMA N° 8 12° OIQ

Análisis cualitativo inorgánico

En diez botellas de reactivos hay diez muestras diferentes de me tales. Por


evaluación de la solubilidad y por la identifica ción subsecuente solamente
seis de los siguientes elementos se especificarán por sus números de muestra:

Calcio, Hierro, Aluminio, Zinc, Magnesio, Estaño.

A ser especificado:

a)_ El símbolo del metal identificado y el número de la botella


correspondiente.

b) La ecuación para la disolución de cada uno de los seis meta les.

c) Una prueba verbal sin ambigüedades o una prueba en la forma de una ecuación
gnímica.

Los siguientes reactivos químicos están a su disposición:

HCl conc, HCl (2M) , H2S04 (2M) , CF^CCOH (2M) , NaOH(2M) , NH3(2M) , NH4SCN

(0.2M), CH3COO~Na+ (conc), 3% H^, Na2C03 (0.2M), H2S (0.1M); N2HP04 (0.2M),

K4Fe(CN)g (0.2M), K3Fe(CN)5 (0.2M), amina (en CH30H) , quinalizarina (en C^OH)

, urotropina (20%) , ditizona (en CC14) , (NH4)2C204 (0.2M), H20 dest.

------------------------ — S O L U C I O N ----------

a) Vea la lista al final

b) Ca + 2 H20 ------- >■ Ca(OH)_2 + H2

Ca + 2 H30+ ------ ► Ca2+ + W + 2 HjO

Fe + 2 H30+ ------- >■ Fe2+ + H2 + 2 H20

Al + 3 H30+ ------ ► Al3+ + 3/2 H2 + 3 H20


- 245 -

Al + NaOH + 3 H20 -------->■ Na+ + Al (OH) ~ + 3/2 H2

Zn + 2 H30+ ------- ► Zn2+ + H2 + 2 H20

Zn + 2 NaOH. + 2 H20 ------ >■ 2 Na+ + Zn(OH)2- + H2

Hg + 2 H20 ------y H2 + Hg(OH)2


Hg + 2 H30+ ------- >• Hg2+ + H2 + 2 H20

Sn + 2 H30+ ------ + Sn2+ + H2 + 2 H20


Sn + 2 H20 + NaOH ------ «-2 Na+ + Sn(OH)2- + H^

c). Ca : precipitado blanco con (NH4)2C204 (en ácido acético)


Fe : azul con K3 {Fe(CNg)} 6 después de oxidar con H202: a-zul con K4
{Fe(CNg)} 6 rojo con NH4SCN 6 un precipita üo café con NaOH:.

Al : fluorescencia "verde con morina (en ácido acético diluí_ do)_.

Mg ; con quinalizarina: laca de color azul pálido (solución alcalina).


- 246 -

Sn :. con HjS : precipitado café oscuro (solución acida débil).,


fluorescencia azul de la pared exterior de la botella de vidrio llena
con agua fría que se sumerge en la solución de Sn2+

(acidificado con HCl) , en la llama de un mechero Bunsen (_*ensa yo a la


llama*)
- 247 -

PROBLEMA N° 9 12° OIQ

Determinación titrimétrica de perozodisulfato de potasio:

a). Principio

A la muestra (_K2S20g). se le agrega un exceso -medido de una so lución de


FeClIl. El exceso de Fe ("II) se determina con -una solución estándar de
KMnO.„

5) Procedimiento

1) Determinación de la concentración de la solución de Fe(II):


[Fe(NH.4l2(S04) J]
En un vaso de precipitado se agregan 25.0 mi de la solución de Fe CID
, 10 -mi de H3P04 (ca. 3.7 mol/1) y 10 mi de H"2S04
(ca. 1 mol/1) y se titulan con KMn04hasta coloración rosada. La
concentración de la solución de KMn04 es exactamente 0„02 mol/1. Se
efectúan dos titulaciones y se anotan los consumos- de KMn04. Se
utiliza el valor medio (=V^).

2)Determinación del peroxodisulfato en la solución muestra„


a) La muestra disuelta se diluye con agua destilada a lOOml en el
aforado y se mezcla bien.

b.) 25. Q mi de esta solución son transferidos a otro vaso, se mezclan


con 10 mi de H3P04 Cca„ 3„7 mol/1), 10 mi H2S04 (ca. 1 mol/1) y 25.0
mi de la solución de Fe (II).La mez cía se deja reposar por 5 minutos
y se valora con la solución de KMn04 C0.02 mol/1) hasta coloración
rosada. Se efectúan dos valoraciones y se utiliza el valor promedio
c=y21-

c) Cálculos y- evaluación:

I. Ecuaciones de las reacciones: A ser dadas:


- 248 -

Ecuaciones parciales con balance electrónico:


Ecuación total

1) Reacción del peroxodisulfato con Fe(II):

a) S^- .............................

b). Fe2+ .....................................

c) Ecucación total

2). Reacción del Fe (II) con permanganato:

a) Fe2+

c) Ecuación total

II. La concentración de la solución de Fe(II)

1) De el consumo de la solución de KMn04 (mi) por 25.0 mi de solución

de Fe(II) (=V^); ver procedimiento 1.

2) Calcule la concentración de la solución de Fe (III en mol/1.

III. Determinación del K2S2Og

1) De el consumo de la solución de KMnO^ gastados en la retrovaloración


del exceso de la solución de Fe(II) en mi (=V2); ver procedimiento
2.

21 Cuántos mg K^Og : 270.33 g.mol"1

31 Calcular la concentración de la solución muestra de K-S.O. en


mol/1.

b) MnO~ .............................
S O L U C I O N

valor obtenido
experi-mentalmente
.
I/l/a S2Og~ + 2 e" J 2 SO2

b Fe2+ Fe3+ + e"

c s2°8~ + 2 Fe2+ í 2 S04~ + 2 Fe3+

I/2/a Fe2+ j Fe3+ + e~ |.5


- 249 -

2
(V- - V,) . 0.02 o 5 2.70.33
al —= -------------------------------------------------- . ------- = .... mg K S_0,
1000 2

25 . c 2+ - V- o 0.02 . 5 270.33
g) ------ — --------------------- . ------ .... mg K S O
1000 2

(V - V ) . 0.02 . 5 40
al —----------------- - --------------------------------- . — = ....
mol K S O /l
1000 2¿

25 . c - V, . 0.02 . 5 40
S). ----- — -------------------
. -------- — = ----------------- mol/1
100Q 2
MnO^ + 8 H++5 e" J Mn2+ + 4 H20
XIII. OLIMPIADA INTERNACIONAL DE QUIMICA

BURGAS 1981

BULGARIA
La sustancia simple A participa en las transformaciones del equema
1. Solamente los productos que contienen A se mués -tran en el esquema.

Esquema 1

H20 KOH
electrólisis

KOH
A + B: + E
calentado

KOH
A + I + E

1. La sustancia A es un sólido y es insoluole en agua.

2. Las sustancias B é I son gases solubles en agua.

3. Las sustancias E, F, J y K son sólidos solubles en agua.

4 o Las soluciones acuosas de B, C, H, I, J y K reaccionan con F, los productos


en todos los casos son E y D.
- 251

PROBLEMA N° 1 13° OIQ

5. Las siguientes transformaciones con una solución acuosa de yodo.

-+ A A

Plantee las ecuaciones químicas para las interrelacrones y ba lancéelas.


- 252 -

-------------------------- S O L U C I O N -------------------------

(1) S + o2 ----------------- >■ so2

(.2) 2S02 + 02 ---------► 2S03

C3). S03 + H^O --------<■ H2S02

(.41 2KOH + H2S04 --------- ► K2S04 + 2H20

(.51 2S04" - 2e~ --------- >- S2Og~

C6) S02 + 2KOR -------- ■+ K2S03 + H20

(7). K2S03 + S --------- ► K2S203

K S + H S0 K S0 + s + so + H
C81 2 2°3 2 4 ' ------ " 2 4 2 2°

ta) H2 + S ------- ► H2S

(10.) H2S + 2KOH ---- ► K2S + 2H20

Ul,. K2S + XS ------- ► K2S(x + 1}

(121 K 2 S te+l) + H2S04 --------- " K2S04 + xS + H2S

(13) S02 + 2H20 + K2S20g ---------- ► K2S04 + 2H2S04

(14) K2S03 + H20 + K2S20g ----- ► 2K2S04 + H2S04

(151 K2S203 + 5 H20 + 4 K2S2Og ------ ► 5 K2S04 + 5 H2S04

(16) H^S + 4H20 + 4K2S2Og ---- * 5H2S04 + 4K2S04

(17). K2S + 4^0 + 4K2S20g ------ ► 4H2S04 + 5K2S04


253

(18) K2S(x+1) + C4x+l) H20 + 4x K^Og ------ ► 5xH2S04 + (4x+l> K2S04 (+S)

(19) S02 + 2H20 + I2 -------- <■ H2S04 + 2HI

(20) K2S03 + H20 + I2 ------ »- K2304 + 2HI

(21) 2K2S203 + I -------- v 2KI + K2S4°6

(22) H2S + I ------- ► 2HI + S

(23) K2S + I2 --------- ► 2KI + S

(24) K2Sx + I2 ------- ► 2KI + xS

S = SO„ SO- D = H2S04

K2S04 F = K2S204 G = K2S2°3

K2S03 H

H2S K2S K^S L = K2S406


2 x
- 254 -

PROBLEMA N° 2 13° OIQ

El ácido maleteo (H9A)


es un ácido dibásico débil. La co*
- 2-
rrelación entre las cantidades relativas de H.A, HA y A

a a /c,
'l
■H -/c y a2 = C 2"/C
A
0
= c. A

= c.

(c es la concentración total) y el pH de la solución mues_ tra que:

a. aQ = por pH = 1.9 2

al = a2 Por PH =

Encontrar:

1. El valor de las- constantes de disociación para la primera(K^J y la segunda


{K^l Y el grado de disociación.

2. Los valores de aQ, ct^ y para pH = 1.92 y pH = 6.22

3. ¿Cuál es el valor del pH cuando ct^ alcanza su máximo valor? Encontrar el


valor máximo de ct^.

4. ¿Cuál de los valores ácido-base de la tabla son adecuados para la valoración


de una solución 0 , 1 M de ácido maleico con una solución 0.1 M NaOH, como
ácido monobásico y ácido dibási co?

Ponga en la tabla las respuestas correctas.

Todos los coeficientes de actividad deben ser considerados i-gual a 11


- 255 -

Indicador Intervalo de pH

Verde de metilo 0.1 - 2.0


Tropolina 00 1.4 - 3.2
g- dinitrofenol 2.4 - 4.0
Azul de bromofenol 3.0 - 4.6
Rojo Congo 3.0 - 5.2
Rojo de Metilo 4.4 - 6.2
Rojo de Bromofenol 5.0 - 6.8
Azul de Bromotinol 6.0 - 7.6
Rojo Fenol 6.8 - 8.0
Rojo Cresol 7.2 - 8.8
Azul de Tiraol 8.0 - 9.6
Fenoftaleína 8.2 -10.0
Amarillo de Alizarina 10.1 -12.1
Tropolina 0 11.0 -13.0
1, 3 , 5 - trinitrobenceno 12.2 -14.0
1 K
l =

K2 =

2 pH = 1 . 9 2 a o
a
l =

a 2 =

pH = 6 . 2 2 «0

«1 =

a 2 =

3 pH =
a
l =

4 pH =

Primer indicador. Punto 1 . 2 .


de equiva -lencia.
3. 4 .
pH =

Segundo indicador. 1.
Punto de equiva -lencia.
' 2.
3. 4 .
S O L U C I O N

1. aQ = a±

K+ = cH+ = 10"pH = IQ-1'92 = I.2 0 mol/ 1

a
l = a 2

K 2
= CH+ = 1 Q ~PH = 10~6"22 = 6 . 0 2 o 10"7 mol/ 1

2. F = c2+ + Kl CH+ + KaK2


- 256 -

pH = 1.92, cH+ = 1 Q -lo92 = 1 > 2 Q ^ 1 Q -2 r m Q l / l f F = 2 o 8 8 o 1 0 -4 aQ = o1 = c2+/F = (1.20

. 10"2)2/(2.88 . 10-4) = 0.500

a 2 = K iK2/F = 1 , 2 0 ' 1 0 ~2 • 6.02"l0~7/(.2o88 . 10"4) = = 2.51 . 10"5

pH = 6.22, cH+ = 10~6o22= 6.02 . 1Q_7 mol/ 1 , F= 1 . 4 4 5 . 10"8 °0 = CH+/F=C6 . 0 2 . 10"7)

2 /(1.445 „ 10"8) = 2.51 . 10"5

= a2 = K ± K2/F = 1.2Q . 10~2 . 6 . 0 2 . 10~7/.(1.445 . 1Q~8)=0.500

3. f- a i~cH+
= (KlF " KlCH+C2cH+ + K 1))/f2 = 0 c2+ = KaK2
1

c0+ = O..2 0 . . 10"2. 6.0,2 . 10"7)1/2= 8.50 . 10-5 mol/ 1 F = 1.034 . 10~6, pH = 4.07

01 = Kac +/F = (1.20 . 10"2. 8.50 . 10-5)/(1.034 . lo"6) = 0.986

El pH y el valor máximo de a, pueden estimarse o por cálculo de


-3 -4
oc, para un set de valores de C„+ en el intervalo 10 - 10 ± n
mol/1, 6 de la condición que a^, puede alcanzar un valor máxi_
mo solamente cuando a = a_
o 2

4. El primer punto de equivalencia se encuentra en la región de


a., máximo a pH 4.07, donde CT, - = c„ .., = 1/2 = 0.05 mol/1
1 HA NaHA

El segundo punto de equivalencia se encuentra en la región alcalina donde:

COH" = CHA~' CA2" = °-1/3 " COH- = °'0333'

K
CH+ = K2CHA" /CA2" = K2 COH" /CA2~ = 2V ( = H+ CA2"] ' CH+ = (K2Kyv/cA2-)1/2 =

(6.02 . 10_71 . 10-14/0.0333)1/2 4.25 . 10_1° mol/1, pH = 9.37

Indicadores:

Azul de Bromofenol, Rojo Congo, Azul de Tiraol, Fenolftaleí-na.


PROBLEMA N° 3 - 257 - 13° OIQ

El compuesto x ha sido aislado de un producto natural. Se han usado


diferentes reactivos para establecer la estructura de x. Se obtuvieron los
siguientes resultados:

Combustión C0„ y H?0; J..98 g de x producen


1478,4 mi C02, medidos bajo
condiciones norma les y
I0I88 g de H20

II. + fenilhidrazina /fenil hidrazona de x/

III. + NalO 0.189 g de x reaccionan con


21.0 mi de una solu ción
dgjNalO (conco0.05 mol.dm J

IV. + KCN +H20, OH reducción con HI heptai


* D ácido

+ anhídrido acético
La masa molecular de E es 116.67 más
grande que la de x,

¿Qué conclusiones oueden obtenerse sobre la composición y estructura de x sobre


la base de los datos obtenidos en cada u-na de las interacciones arriba dadas
? Las conclusiones deben formularse en la forma mas concisa y clara. Complete
la tabla sin describir como loqró las conclusiones.
l o ........................ „ ............. . .................. „ . , . . . . . . , . « . .

II. .............................................

III. .......................... o ...... o .........

IV. __________ o ..................... „ ..........

V. ............ „ . O o . . . O O .................. O . . .

2. Escriba la fórmula de la sustancia x basado en los datos de la composición y


estructura obtenido en el punto 1.

3. Es-criba la fórmula de las sustancias A, B, C, D y E y la del heptano ácido.


PROBLEMA N° 3 - 258 - 13° OIQ

4. A qué sustancia natural corresponderá esta estructura? Escri_ ba el nombre de


la sustancia y dibuje la fórmula estructural que mejor describa sus propiedades
estructurales.

5 o Dé tres propiedades de este compuesto que no corresponden a la es-tructura


encontrada en el punto 2.

-------------------------- S O L U C I O N --------------------------

I La fórmula empírica más simple


CH20.
II Presencia de un grupo C = O

III Presencia de un grupo -CHO M,


calculado para un solo CHO-180/n, /n-
número de grupos CHO/

IV Cadena continua de 6 átomos de


C 1 CHO, C.H1o0, /M = 180/ 6
12 6

5 grupos OH
1. Reacción Obtención /!/ /2/
- 259 -

HOCH2/CHOH/4CHO

CH=NNHC,HC
I 6 5 COOH CH COOH COOH CHO I
/CHOH/. I I I /CHOCOCH,/.
/CHOH/4/CHOH/5/CHOH/5 /CH2/5 I 6 CH OCOCH3
CH OH CH-OH CH-OH CH-OH CH.
¿ ¿ ¿ j

heptano
ácido

4. D-(.+ )-■
glucosa

CH2OH

O
H
HO
OH
OH
CH2OH

5. No participa en las mismas reacciones típicas de los aldeni-dos (e.g„


con NaHSO^ ó el reactivo de Schiff)

- el fenómeno de la mutarotación
- una fuerte reactividad de uno de las cinco grupos OH /erxhi bido, por
ejemplo en la interacción CH^OH y HC1, guiando la metilación sólo a
un grupo OH/.
260

PROBLEMA N° 4 13° OIQ

La descomposición térmica del agua

H20 + H2 + 0.5 02

puede ser seguida ( a = 10~3) a temperaturas sobre 1700 K. Este proceso puede
ser realizado a temperaturas 800 - 900K, también como a través de las etapas
subsecuentes que se ejecutan en -un ci cío. Sugiere un proceso sobre la base
de las reacciones:

840 K CuCl , , + .MgO, . + (s)


CuO (.si + M9C12(s) + H2°(g) a (s)

+ HCl,. , + O-
, ,
(gl 2
(g)
430 K
Ag. . + HCl. . i AgCl, . +
H-,
y(s) , (g) ^ (s) 2{g)

Satisfaciendo los siguientes requerimientos:

1. Solamente agua debe consumirse durante el proceso.

2. Oxígeno e hidrógeno solos deben ser los productos finales del proceso.

3. Se necesita junto a las sustancias arriba dadas una solución de amoníaco


al 25 por ciento para el ciclo.

4. La temperatura para cada etapa del ciclo no debe exceder 840K.


261 -

S O L U C I O N

(1) 2CuO + 2MgCl2 + H20 840 K > 2CuCl + 2MgO + 2HC1 + 0.5 02

(2) Ag + 2HC1 430 K > 2 AgCl + H2

(3) 2CuCl + 4NH3 ------- >■ CufNH^ J + + 2C1~

(4) 2AgCl + 4NH3 ------- ► |^2 Ag(NH3)2 + + 2C1_

(.5).------------------------------------- 2{Cu(NH3)2}+ + 2{Ag(NH3)2

► 2Agl- + 2 {Cu (NH3) }2+

(6). 2{Cu(NH3)4}2+ + 2MgO ebullición ^ 2CuQ+ + 2yí¡ 2+ + 8NfJ ^ +

(7) 2Hg2+ + 4C1" evaporación ,

(8) H20 -------- ► ^ + 0.5 02


- 262 -

PROBLEMA N° 5 13° OIQ

Los compuestos B y C son isómeros estructurales. Ellos pueden


obtenerse cuando el hidrocarburo A reacciona con cloro. El hidrocarburo A es
un producto básico de una síntesis orgánica industrial. Puede reaccionar
con ozono produciendo un ozóni do.

El isómero B puede ser usado para la producción técnica de los


compuestos- D y E, los cuales son los compuestos ini -cíales para la producción
de la fibra poliamida "nylon" 6,6 - H
-{NH/CHo/cNHC0/CH_/.C0} -OH.
¿ o ¿ 4 n

El compuesto D en bases y E - en ácidos.

La reacción entre el isómero C y una solución alcohol! ca de una


base alcalina produce el monómero F, el cual se usa para la producción de
goma de cloropreno (neoprén) -í-CH2CCl=CHCH2-}r Este método tiene aplicación
técnica.

1. Escriba las fórmulas estructurales de A, B, C, D, E, F y sus nombres según


nomenclatura IUPAC.

2. Escriba el mecanismo de reacción entre el hidrocarburo A y el cloro. ¿Cuál


es el tipo de reacción en términos de su meca -nismo? ¿Cuál de los dos
isómeros se obtiene en mayor cantidad bajo condiciones ordinarias?

3. Escribe las ecuaciones:

- para la producción de D y E del isómero B.


- para la producción del isómero F del isómero C
- para la ozonolisis del hidrocarburo A y la hidrólisis del ozónidQo

4. Escriba el esquema de reacción para un método industrial de la producción


del hidrocarburo A del hidrocarburo el cual es el
- 263 -

componente principal del gas natural.

5. Escriba las fórmulas para las formas isoméricas posibles de la unidad


elemental de la goma neopreno.

S O L U C I O N

1. A: CH^ = CH - CH = CH2 1,3 -


butadieno

B: C1CH2-CH = CH - CH2C1
1,4-dicloro-2-huteno

C: CH2 = CH - CHC1 - CH2C1


3,4-dicloro-l-buteno

D: HOOC/CH2/4COOH ácido
hexanodioco

E: H2N/CH2/6NH2
1,6-hexandiamina

7; CH2 = CC1 - CH = CH2


2-cloro-l,3 butadieno

CH„=CH CH-CH^C1
2. CH2 = CH - CH - CH2 + Cl^c\
+C1
CH2-CH=CH-CH2C1

CH2 =CH1 -CH2C1 r-»CH2-CH-CHCl-CH2Cl

+ + Cl
►C1CH2-CH=CH-CH2C1
CH2 -C '■. -CH2C1
H: CE

1,4-dicloro-2-buteno
ZHO=HD-HO=ZHO < H.°BN-+UZ
LHJ

H—

CH0=H0-H3= H3 < ----- = - H O V HO/OH < -


O H - ¿H+

7 7 oyi
i ---- HO H0-3SO- HOOH+- QK3H+ H3STH3 « ^—
----- ^HO
H~

ZH0=H3-H3=SH3 * --------
HO2HO3H0/HO/HDeHO ---- TN-
02H- Z H+

7 7 -0H r +Z6H
V *
OH3 H3/H0/H0 H3 +--------- 0K3 H3 «--- «—" H3=H3 ■< ----- 7H3
0H0LH3+ O H+ H-

-0 O---- O
ZQZ-& Z + 0H33H0 + OHOH Z -> --------- 02H Z+ ZfíO H0-H3 3H3

\0/ \'
O-
Z UD H3-H3 SH0 < -- e0 Z + 3H3=H3-H3=ZH3

O^H + _I0+2H3 = T33-H3=3H3 -< ------ _0H + T33H3T0H3H3=^H0

ZER^/Zfí3/RZE ■>--------- ZE + NOV^HO/ON

r f7 7 OH/ _
HNZ + HOOO / H3/300H +—¿ ------ ,~g0 06H{' + N.3V H3/3N

N33H3ZH32H3SH30N i— p Z U + N32H3H0=H33H30N

TOX 2 + N3JH3H3=H0JH33N -¡- ----- N3X Z + T32H3H3=H3SH0T3

- ice
_H2 +H2° +H7 -H..-H.0
CH4 ~K CHHCH
'4 "IT- n ¡p+ "Ni—* LH3^2L" cat. t . CHICHO
—CH, CH OH 2 2

CH2=CH-CH=CH2

5- "€\ /CH2" ^C = C -CH


c = c
Cl H Cl \ CH,-
- 265 -

PROBLEMA N° 6 13° OIQ

La descomposición catalítica de isopropanol sobre la su


perficie de un catalizador de , Pr°duce los productos dados
en el esquema y satisface una ecuación cinética de primer orden.

C3HgO (. B
5

C3H7OH ^- ---- kk^ C3H.g (. C )

C3Hg (.
D )

Cinco segundos después del inicio de la reacción a 590K, la


(. A I
concentración de los componentes en la mezcla de la reacción son:

Ca = 2 8.2 rmmol/dm3
Cg = 7.8 mmol/dra3
Cc = 8.3 mmol/dm3 3
C = 1„8 mmol/dm

1. ¿Cuál es la concentración inicial c del C,H,OH en el sistema?


o 3 7
2. ¿Cuál es la velocidad de reacción específica k del proceso?

C3H_,OH ---- —y Productos7

3. ¿Cuál es el intervalo de tiempo /Tj_/2/' en el cua^ concentra ción del CjH^OH


alcanza el valor c = CQ /2?
4. ¿Cuáles son los valores de las concentraciones cB, c^, cQ, a t i1/2.
5. ¿Cuáles son los valores de las velocidades de reacción k^, k2 y k3?
- 266 -

La ecuación que describe los cambios de concentración de A con


el tiempo t para una ecuación de primer orden tiene la forma:

cA = cQ exp. (-kt);

ó log (cQ/c ) = 0.4343 kt

ó ln (cQ /cA) = kt. Complete la Tabla

con los valores obtenidos:


c
1 O =
2 k =
3 T = 1/2
k
4 l =
k2 =
k3 =

5 C3 =
CC =

C =
D
- 267 -

S O L U C I O N

1.

c„ = c, + + c„ + c^ = 28.2 + 7,8 + 8.3 + J..8 = 46.1 mmol/l

2.
1 1 -1
k = -------- lg(c /c ) = ----------- lg(46.1/28.2) = 0 .0983 s
u A
0.4343t 0.4343 . 5

3.

t = Tl/2= 0 .4343 k ^ = 0 . 4343 ! 0 .0983 lo? 2= 7'05 S'

4- Vl = kl CA

V k
2 = 2 CA

Ac
^3 = -rf = k3 CA

v = VjL + v2 + y3 = k cA

CU k^ + k2 + k3 = k = 0.0983 s 1

Ac c - O c k _ .

(3)-^*- = CB " ° - CB _ kl _ UL= 3


4 3
AcD cD - O cD k3 1.8
De las ecuaciones (1) - (.31: k 1

= 0.0428 s-1, k2 = 0.0455 s"1,

k3 = 0.00988 s-1, 5. At t = T1/2

= 7.05 s

(4) cA = c0/2 = cB + cc + c^ = 23.05 mmol/1

De las ecuaciones (21 - (4): cB =

1Q.0 mmol/1, cc =10.7

mmol/1, = 2.32 mmol/1.


- 269 -

PROBLEMA N° 7 (práctico) 13° OIQ

Catorce tubos de ensayo numerados contienen soluciones de


sustancias inorgánicas puras. Cada tubo contiene solamente u na sustancia.

Las muestras contienen los siguientes iones: cationes:


Determine las muestras en los tubos de ensayo usando los siguientes
reactivos: K+, Na+, Hg2++ Ag+, NH*, Ba2+, Sr2+, Fe3+ y aniones

OH", NO^, Cl", SCN",agua, 2~, Cr 02", CrO2", FeCCN)*", Co(N0 )37
i", C02,4-dinitrofenilhidrazina
FeCl^ C2.5% solución ocuosaL 2 , 2

reactivo de Lucas (;znCl2-HCl) , NaOH (.'5% solución acuosa) , NaHCO^


Determine
(5% solución acuosa),elHC1
contenido de los tubos
(concentrado), de de
solución ensayo. Además
Fehling (so dé las
2 +
reacciones entre las muestras, el -único reactivo posible es nina solución
lución
de ácidoalcalina acuosa
clorhídrico que contiene 2 iones
de concentración ml.dm Cu ; esta es prepara da
inmediatamente antes de usarla mezclando volúmenes iguales de las soluciones
Fehling I yEscriba
II), reactivo de Tollen
la siguiente (preparadaen
información inmediatamente antes de usarla
las hojas entregadas :
y una de 2 mol/1 de NaOH; después de mezclar se agrega amoníaco gota a gota
hasta
2.. La disolver el hidróxido
fórmula química de plata formado)..
correspondiente de la muestra individual y el número
del tubo de ensayo.
Escriba los resultados de sus observaciones durante los ensayos
de lasfórmula
2. La sustancias desconocidas
química en lasobre
en forma iónica tabla
ladada.
base por las cua les se demostró
la presencia de un catión individual en la -muestra.
Escriba los (IUPAC) nombres y las fórmulas de las sustancias en
los tubos de ensayo.

Escriba las ecuaciones químicas sobre la base en las cua les las
sustancias individuales fueron identificadas. Cuando e£ to no sea posible
escriba solamente el esquema de reacción.

PROBLEMA N° 8 (práctico) 13° OIQ


- 270 -

PROBLEMA N° 9 (práctico) 13° OIQ

Determinación volumétrica simultánea de carbonato y bi_ carbonato


de sodio:

A/ Procedimiento:

2. Determinación de la concentración exacta de "una solución de HCl (.0..1


mol.¿tai ) usando Bórax NajB^O^ . 10 H20 como es tandar primario.

Principio:

Una solución acuosa de tetraborato de sodio reacciona con ácido


clorhídrico para formar ácido Bórico (H^BO^).

Procedimiento:

25.00 cm3 de una solución estándar de bórax con una concen tración
alrededor de 0„05 mol.dm 3 (la concentración exacta del bórax está
inscrita en la etiqueta del frasco afora do; la solución no necesita
ser diluida con agua a 100 cm ) es transferida a un vaso para la
valoración, se agregan 1 a 3 gotas de rojo metilo y se valora hasta
la primera coló ración naranja de la solución amarilla. La valoración
debe hacerse en duplicado.

El consumo de ácido clorhídrico se designa por "V^.

2. Determinación volumétrica de Na2CO.j y NaHCO^ simultáneamen te.

Procedimiento:

La muestra en el frasco aforado (250 cm3) debe diluirse hasta la marca


con agua destilada la cual ha sido hervida para expulsar el dióxido
de carbono y se mezcla.

ción de HC1 usando anaranjado de metilo (dos gotas) has ta la primera


coloración roja de la solución original -mente amarilla. La muestra
se hierve por 2 a 3 minutos para expeler el dióxido de carbono0 Luego
la solución se enfría y se continúa la valoración hasta un claro
- 271 -

a/ Parte de la solución (.25.00 cm ) se valora con una solu

co lor rojo de la solución. La valoración debe hacerse en duplicado,


se designa por el promedio de las dos valoraciones .
b/ Otra alícuota de la muestra (25.00 cm3) se transfiere a
un Erlenmeyer y se mezcla con 25.00 cm3 de una solución
-3 3 de NaOH de
concentración 0.1 mol.dm . Se agregan lOcm
de una solución al 10% de BaClj y 2 gotas de fenolfta -
leína. El acceso de hidróxido debe ser valorado inme -
diatamente con una solución de HC1 en presencia del pre
cipitado blanco formado. La concentración del HC1 ha si
do determinada con precisión en la parte 1. El consumo
de HC1 se designa por (promedio de al menos dos valo
raciones).

c/ Efectúe la misma valoración de la parte b/ pero sin muestra


(blanco). El consumo de HC1 se designa ahora por (promedio de
dos valoraciones).

Use los resultados anteriores para calcular las cantida des de


Na2C03 y NaHC03 de la muestra.

Resultados

Los resultados deben anotarse en la hoja dada, en la cual se debe anotar


lo siguiente:

a) La ecuación iónica de la reacción de acuerdo a la cual la


concentración del HC1 fue determinada.

b). La ecuación iónica para la reacción usada en la determina-


ción simultánea de Na^CO, y NaHCO,.
- 272 -

c) El volumen de HCl gastado en la valoración de 25.0,0. cm de la


solución de Bórax.
d) Cálculo de la concentración de HCl (en mol.¿tai 3)

e) Gasto de la solución de HCl; V2> V3, y ^.


f) La cantidad de masa de Na.,C03 y de NaHCC>3 en la muestra (en gramos).

M (Na2C03) = J.0.5.989 g.mol"1 M

(NaHCO,) = 84.00.7 g.mol"1


XIV. OLIMPIADA INTERNACIONAL DE QUIMICA *

* *

ESTOCOLMO 19

82 SUECIA
A, El nombre del compuesto {Co(NH3)g} Cl2 según IUPAC es:

a) cobalto [II¡ hexaamoníaco dicloro

b) cobalto (II) hexaamoníaco dicloruro

c) hexaaminocobalto CII) cloruro

d) hexaaminodiclorocobalto til)

e) cobalto ("IIÍ cloruro hexaamoníaco

B. El nombre IUPAC del compuesto

l i l i
l í es
H H H H 0^ H

H BrH H H H

a) 5-Eromo-l-ácido hexanoico bl

5-bromo-2-hidroxi-J.-hexanol c)

2-bromo-5-hidroxi-6-hexanal ñ)
- 274 -

PROBLEMA N° 1 14° OIQ

2-bromo^2-hidroxi-l-hexanal e)

5-bromo-2-ñidraxi-_l-hexanona

C. ¿Cuál de los siguientes pares de ácido-base es el mas apropi do para


mantener el pH a 9 en una solución acuosa?

a)l CH3C00H --- CH"3C00~


-- NH
3

c). H2C03 di 3

H2P0~
-- HCO

-- HPO
- 275 -

Una de las siguientes aseveraciones no puede ser correcta. Indique cuál.

2 + 3+ —
a) Un sólido soluble en agua contiene Mg , Cr y Br .

b) Un sólido soluble en una solución de hidróxido de sodio


3+ +
2-contiene Al , K , S04
c) Un sólido soluble en amoníaco acuoso contiene Ag+, Cu2+ y Cl".
2+ 2
+
d) Un sólido soluble en ácido nítrico contiene Ba , Fe y

<• + 2+ 3-
e) Una solución neutral al litmus contiene Na , Ca y PO^ .

Complete las siguiente ecuación:

H3As04 + Zn ----------- AsH3 + Zn2 +

La reacción se efectúa en solución acida. Complete con las partículas


que faltan y balancee la ecuación de la reacción.

Determine el grado de protólisis del ácido acético de con -centracxón:

0.25 mol/dm3 . K (HAc) =1.9 . 10-5 mol/dm3.


CL

a) 0.021% b) 0.21% c) 0.84% d) 1.3% e) 8„4%

3
Una solución con un volumen de 1.0 0 dm se satura con yoduro de plomo,
Pbl,. La concentración de los iones de yoduro 3
es 2.7 -mmol/dm . Determine el producto de solubilidad del Pbl2.

ai 3.6 . 10~6(mol/dm3)2 b) 2.0 . 10-8(mol/dm3)3

c) 9.8 . IQ-3(mol/dm3)3 d) 2.5 . 10-9(mol/dm3)3

e) 4.9 . 10~9 (mol/dm3)3


- 276 -

H. Se dan las siguientes entalpias de formación:

Compuesto AH°

ácido acético - 0.50 MJ/mol

dióxido de carbono - 0.4 0 MJ/mol

¿\H°de la combustión del ácido acético es:

a) 0.90 MJ/mol ta). -0.90 MJ/mol d) -0.20 MJ/mol

d) -2.1 MJ/mol e) 0.20 MJ/mol

En un recipiente vacío se introduce CoCl2 (g) a una presión a. Se disocia


y se establece el siguiente equilibrio a tem peratura constante.

2COCl2(g) + C(graphite) + C02(g) + 2Cl2(g)

Si x representa la presión parcial del CO^tg) en el equilibrio, ¿cuál


es la expresión del equilibrio?

- —= K b) , = K c) ----- ?*1 = K
(a-2x)2 p (a-2x)2 p (a-x)2 P

4x3 „ . x3
d) --- = K e) -------- ~ = K
(a-x)2 P (a-3x)2 p

J. Para un metal M se conocen los siguientes datos redox:

E° = -0.60 V para M2+(aq) + e~ -+ H+(aq)


E° = 0.40 V para M4+(aq) + 2e~ ■+ Il2+(aq)
4 -f - -f
El E° para M (aq) + 3e + H (aq) es entonces
- 277 -

agua - 0.30 MJ/mol

a) -0.20 V b) -1.0Qy c) 1.0.0 V d) 0.07 V e) -0.07 y

------------------------- S O L U C I O N ----------------------

A. cl B. b) C. b). D, e)

E. H3As04 + 4Zn + 8H+ --- >■ AsH3 +


4Zn2+ + 4H20

F. c) G. c) H. b) I. a)
K. d)
- 278 -

PROBLEMA N° 2 14° OIQ

C A D FG
E
l
análi
sis
cuant
itati
vo
para carbono e hidrógeno se e-fectuó originalmente usando -una técnica y el
aparato (ver figu -ra) desarrollado en 1831 por el famoso químico Justus Liebig.
tina muestra cuidadosamente pesada de un compuesto orgánico (C) se pone en el
tubo de combustión (A) y es vaporizado por calenta miento en un horno (B). Los
vapores son conducidos por una co -rriente de oxígeno a través de un relleno
de óxido de cobre ca -líente (D) y a través de otro horno (E) el cual asegura
la oxida ción cuantitativa del carbono e hidrógeno a dióxido de carbono y agua
respectivamente.

El vapor de agua se absorbe en un tubo tarado (F) que con tiene


perclorato de magnesio y el dióxido de carbono se absorbe en otro tubo tarado
(G) que contiene asbestos impregnado con hidróxido de sodio.

Una muestra pura líquida que contiene solamente carbo -no, hidrógeno
y oxígeno se coloca en una navecilla de platino de 0.57148 g. La muestra se
oombustiona y los tubos de absorción previamente tarados son pesados
nuevamente. La masa del agua en el tubo de absorción aumento de 6.47002 g a
6.503599 g, la -masa del dióxido de carbono aumentó de 5.4611 g a 5.54466 g.

a) Calcular la composición másica del compuesto.

b) Dé la fórmula empírica del compuesto.


- 279 -

Para estimar la masa molar del compuesto 1.0.045 g se ga sifican.


El volumen medido a Tina temperatura de 350K y una presión de 35.0 kPa. fue
0.9 5 dm3 .

c) Dé la masa molar y la fórmula molecular del compuesto.

d) Dibuje las estructuras posibles a las fórmulas moleculares


excluyendo estructuras cíclicas, estéreos i-sómeros, peróxidos
y compuestos insaturados. Hay al_ rededor de 15 posibilidades.
Dé 10 de ellas.

Cuando el compuesto es calentado con una solución de hi_ dróxido


de sodio se forman dos productos. La destilación frac -cionada de la
mezcla de reacción separa una de las sustancias. La otra sustancia es
purificada por destilación después de acidi^ ficar y parece ser un ácido.

e) Cuáles estructuras son posibles para el compuesto C? 0.1005 del


ácido se disuelven en agua y se valoran con una solución de
hidróxido de sodio con una con -centración de 0.1000 mol/dm3.
El indicador cambia de color por la adición de 16.75 de la
solución de hi -dróxido.

f) ¿Cuál era la sustancia original C?

S O L U C I O N

a) Composición en % masa 54.56% C 9.21% H 36.23% O

b) . Fórmula empírica C^H^O

c) Masa molar 88 g/mol

Fórmula molecular C^HgO^

d) Posibles estructuras

1. CH3-CH2-CH2-COOH 11. CH2 (OH)-CH (CH-j)-CHO

2. CH3-CH(CH3)-COOH 12. CH3-0-CH2~CH2-CHO


- 280 -

3. CH3-0-CO-CH2-CH3 9. CH3(OH)CH2-CH2-CHO

4. CH3-CH2-0-CO-CH3 10. CH3-C(OH) CCH3)-CHO

5. CH3-CH2-CH2-0-CO-H 13. CH3-CH2-0-CH2-CHO

6 . CH3-CH (CH"3) -0-CO-H 14. CH3-0-CH(CH3)-CHO

7. CH3-CH2-CH(OH)-CHO 15. CH3-CH2-CO-CH2OH

8. CH3-CH(.0H)-CH2-CH0 16. CH3-CH(OH)-CO-CH3

17. CH2COH)-CH2-CO-CH3

18. CH3-0-CH2-CO-CH3

e) Las estructuras posibles son: 3, 4, 5, 6

f) La estructura ae C es: CH3-CH2-0-CO-CH3


- 281 -

PROBLEMA N° 3 14° OIQ

En Tina industria química en la cual el formaldehído se produce por


oxidación de metanol, se debe analizar una solución acuosa que tiene metanol
y formaldehído. Para probar el método, se efectúan primero análisis con
cantidades conocidas de metanol o formaldehído.

Se utilizan las siguientes soluciones acuosas:

Metanol, 5.0.0 g/dm3

Formaldehído, 5.00. g/dm3


-2 3

Dicromato de potasio, 3.000.10 mol/dm Sulfito de amonio

y hierro CID, 0.200 mol/dm3 Yodo, 0.1000 mol/dm3

Tiosulfato de sodio, 0.2000 mol/dm3

I. Se mezclan 10.00 cm3 de la solución de metanol con lOQ.OOcm3 de la solución


de dicromato de potasio, se agregan aproxima damente 100 cm de ácido
sulfúrico concentrado a la solu ción y se le deja en reposo alrededor de
30 minutos» El ex ceso de iones dicromatos de titula con iones de hierro
(II)
y ácido difenilsulfónico como indicador redox (cambio de co lor rojo -
violeta a verde pálido). El volumen de la solución de hierro (II)
gastados es 43.5 cm3.

3
II. -1Q.00 cm de la solución de formaldehído se mezclan con 50.00 cm de yodo.
La solución de hidróxido de sodio se a-grega hasta reacción alcalina y
la mezcla se deja reposar alrededor de 10 minutos. Acido clorhídrico se
agrega hasta reacción neutral y el exceso de yodo se determina con tio
-sulfato con almidón como indicador. El volumen de la solución de
tiosulfato gastado es 33.3 cm3.
- 282 -

a) . Con los datos del análisis dados en I y II, calcular las


cantidades reaccionantes y la razón molar, metanol/iones dicromato
y formaldehído/yodo,

b) Escriba las ecuaciones- balanceadas para toda las reaccio


nes descritas- en los experimentos I y II.

III„ Se comprueba que el yodo no reacciona con metanol. De la so lución que


contiene metanol y formaldehido, se toman dos muestras de 10.00 cm3.

Una muestra se mezcla con 2QQ„00. cm3 de una solución de ái_ cromato
de potasio y ácido sulfúrico concentrado como en I. El exceso de iones
dicromato consume 4.8 cm de la solución de hierro CU) .

La otra muestra se mezcla con 50.00 cm3 de la solución de yodo y se trata


como en II. El exceso de yodo consume 16.50 cm3 de la solución de
tiosulfato.

c) Dé las ecuaciones balanceadas para las reacciones y cal_ cule el


contenido de metanol y formaldehído en la solución. Dé sus
3
respuestas en g/dm .

S O L U C I O N

a) Cantidad de sustancia:

de metanol 1.56 mmol

de iones dicromato 3,00 mol

de iones de Fe (II) 8.70 mmol


Razón molar metanol/dicromato 1 mol CH^OH j 1 mol Cr207

Cantidad de sustancia

de formaldehído 1.67 mmol

de yodo 5.00 mmol

de iones tiosulfato 6.66 mmol

Razón molar formaldehído/yodo. 1 mol HCHO ■* 1 mol I2

£>I Ecuaciones químicas

CH30H + Cr202~ + 8H+ ■+ C02 + 2Cr3+ + 6H20


- 283 -

PROBLEMA N° 3 14° OIQ

Cr202" + 6Fe2_ + 14 H+ -»■ 2Cr3+ + 6Fe3+ + 7 HjO

I2 + 2 OH"" T0~ + i" + H20

HCHO + I0~ + OH~ HC00~ + I~ + H20

IO~ + I~ + 2H+ -»■ I2 + H20

I2 + 2S202- ■+ 2I~ + S4Og~

En C3I , (151 y (16) , I^ puede participar en vez de I.,. Como

alternativa a (4)

HCHO + I + 20H~ HCOO- + 2I~ + H20 es aceptable

c) Ecuaciones químicas

A las ecuaciones químicas arriba dadas se agrega:

3HCH0 + 2Cr2027~ + 16H+ -»- 3C02 + 4Cr3+ + 11 H20


3

Contenido de metanol 1.9 g/dm

Contenido de formaldehído 10.1 g/dm3

Un átomo o un ion de un metal de transición puede enla zarse a un


número de átomos o moléculas que lo rodean (ligan dos), formando plantillas
características. Esta es la estructu ra esencial notable de una clase
importante de compuestos llama dos de coordinación o complejos.

Si dos o mas átomos de un ligando individual forman en laces con


el mismo átomo central, se dice entonces que el ligan do forma un quelato (del
griego chel = garra de jaiba).

El ion glicinato, NHJ-CHJ-COO , es un ligando quelato bidentado, el


cual puede formar por ejemplo el complejo tris -glicinato-cromo (III).

La figura muestra una sola estructura posible para tal complejo.


Oxígeno y nitrógeno son forzados a coordinar en posi_ clones octañedricas
adyacentes, pues la cadena N-C-C-0 es demasiado corta para "abrazar" el ion
cromo.
- 284 -

a) ¿Cuántos isómeros configuracionales diferentes del complejo son


posible, sin contar los isómeros ópticos?
- 285 -

b) ¿Cuál de estos isómeros pueden resolverse en isómeros ópticos?

Otro compuesto de coordinación de cromo fue analizado y se encontró que tenía


la siguiente composición másica: 19.5% Cr ,
40.00% Cl, 4.5% H, y 36.0% O. Una muestra de 0.533 q del com-
3 3 puesto se disolvió en 100 cm de agua y 10 cm de
ácido nítri-3
co C2 mol/dm í. Se agregó entonces un exceso de nitrato de pía ta y el
precipitado formado fue filtrado, secado y pesado. La masa encontrada fue
de 0o287 g.

Cuando 1.06 g demuestra se calentaron suavemente a 100°C en corriente de


aire seco se liberaron 0.144 g. de agua.

El punto de congelación de una solución de 1.33 g del compuesto en 100 cm3


de agua fue de -Q.18°C (la depresión del punto de congelación molar del agua
es 1082K kg/mol).

Use toda la información experimental para: cj_

derivar la fórmula empírica del compuesto.

d) deducir una fórmula para el compuesto que muestra los ligandos


del ion cromo. Dé las razones molares para apoyar sus resulta
dos.

e)~ dibuje todos los posibles arreglos estéricos del ligando alre-
dedor del ion cromo.
- 286 -

S O L U C I O N

a) Son posibles dos isómeros geométricos del complejo.

i) el facial, el cuales el que ilustra el problema.

el meridional, en el cual se muestran las posiciones con oxigeno


y nitrógeno
0 0

b)~ Se ve claramente que cualquier complejo con tres ligandos bidentados


unidos octañédricamente como se muestra no tiene simetría especular.
De aquí ambos estereoisómeros se pue -den resolver en isómeros ópticos.

cl La fórmula empírica de CrCl.jH^20g

d)_ La reacción con iones de plata indica que:

1 mol CrCl,H,,0, -*■ 1 mol Cl~ 3 J.2


6 *

Calentando suavemente da:

JL mol CrCl,H_,,0, -+ 2 mol H_0 3


12 b ■*■ Z

Estos resultados da la coordinación ÍCrCl2(H20)4> Cl. 2 H20

Esta fórmula es apoyada por el experimento de punto de congelamiento


que muestra que:

1 mol CrCl,H, ,0, — 2 mol de iones en solución


3n12"6
- 287 -

Posibles arreglos es-térrcos- Se los ligandos alrededor del ifin


cromo:

c i s - (FORMA TRANS-(FORMA
) )
- 288 -

PROBLEMA N° 5 14° OIQ

El yodo es soluble en cierta cantidad en agua pura y es más soluble


en soluciones que tengan iones yoduro. Estudiando la solubilidad total del
yodo en función de la concentración de yo duro; las constantes de equilibrio
de las siguientes reacciones pueden ser determinadas:

Ecuación constante de eguilibrio

I2Cs) ¿ I2(aq) ka (1)

I2(s) + l"(aq) ; I~(aq) k2 (.2)

I2(aql+ I~(aq) j I^Caq) k3 (.3)

a). Dé las ecuaciones de equilibrio de (II - (3)

Soluciones de concentración conocida de yoduro de potasio


{I.}. . se equilibraron con yodo sólido. Subsecuentes valora ¿ tot

ciones con soluciones de tiosulfato de sodio sirvieron para de -terminar
la solubilidad total del yodo íI2^tot

Los experimentos aportaron los siguientes resultados:

{I~J /mmol dm~3 10.00 20.00 30.00 40.00 50.00

[I,]L ./mmol dnf3 5.85 10.53 15.11 19.96 24.82


2 tot

El Dibuje {I2}tot versus {I~}tot

cj Derive una expresión algebraica adecuada que relacione íI25tot e <I-}tot


- 289 -

d) Use el gráfico para determinar los -valores de las constantes de equilibrio


k^, k^ y k^.

S O L U C I O N

ai Ecuaciones de equilibrio

Las siguientes relaciones son válidas para las concentrado nes de las
soluciones acuosas:

ÍI2 ) = ^

}/ {!"} = k2

[I3 }/{ I2 }. {I } = k3 = k2/k1

b l Vea el diagrama de la página siguiente

c). La relación entre (I2^tot y ''tot eS

ll 2hot = ki + T^nq • irítot

d) k = 1.04 . 10~3 mol dm-3

k2 = 0.. 910

k3 = 8„6 . 102 mol"1 dm3

(Estos valores fueron calculados por el método de los mínimos cuadrados)..


- 291 -

PROBLEMA N° 6 14° OIQ

Un ácido orgánico sólido de color blanco, A, contiene solamente


carbono, hidrógeno y oxígeno. Para obtener un valor aproximado de su masa
molar, 10.0 g de ácido se disolvieron en agua. Se agregó hielo picado y por
agitación fuerte se logró bajar la temperatura a -2,5°C. El hielo fue
retirado rápida -mente. La masa de la solución fue de 76.1 g y el valor deter-
minado de pH fue de 1.4, En un manual de constantes de depresiones de punto
de congelación molar para agua se encontró el valor de 1.86 K kg/mol. Una
determinación más precisa de lama sa molar del ácido se llevó a efecto. 0.120
g del ácido se va loraron con una solución de hidróxido de sodio con una
concentración de 0.100 mol/dm3. Se usó fenolftaleína como indicador y se
obtuvo su viraje a rojo cuando se agregaron 23.4 cm3 de la solución de
hidróxido de sodio.

a)_ Dé la masa molar y la estructura del ácido A.

El líquido B se disuelve en agua hasta un 10%. El valor del pH: de la


solución es alrededor de 4. B no se oxida fácil -mente, pero después de
la reacción con yodoformo y la subse cuente acidificación se oxida al
ácido A. 0.10 g de B consume 1.5 g de yodo.

Cuando B reacciona con sodio, se libera hidrógeno y se forma un compuesto


metalorgánico. La masa molar de B es aproxi madamente 100 g/mol.

b)_ Escriba la ecuación química para la reacción de yodoformo y


la reacción con sodio. Use fórmulas estructurales para las moléculas
orgánicas.

El compuesto C en solución acuosa tiene una conductividad que difiere


muy poco de la del agua pura. La hidrólisis al calina de C produce
amoníaco. 0.120 g de C fueron tratados
- 292 -

en caliente con una solución diluida de hidróxido de sodio


y el gas formado fue recogido en 50,0 cm3 de ácido clorhí -
3
drico de 0.100 ml/dra de concentración. El exceso de ácido fue valorado
con 10.0 cm3 de una solución de hidróxido de sodio 0.100 mol/dm3.

La hidrólisis acida de C produce dióxido de carbono. De la depresión


del punto de congelamiento, la masa molar de C es timada es de 40 - 70
g/mol.

Dé la estructura de C. Escriba las ecuaciones de ambas reac clones de


hidrólisis.

Si a C se le permite reaccionar con etil ester del ácido A en la presencia


de un catalizador alcalino fuerte se forman etanol y el compuesto D.
La composición de D es 37.5% C , 3.1% H, 21.9% N y el resto es oxígeno.
El compuesto es un ácido.

Dé la estructura para D. ¿Cuál es el átomo de hidrógeno "á-cido"?


Señalarlo con un * en la estructura.
- 293 -

S O L U C I O N

a). Masa molar de A 103 g/mol

Estructura de A HO-C-CR_-C-OH
II ¿ |l O
O

b\ CK-j-CO-CÍ^-CO-CH^ + 6I2 + 80H: -+ 0-C0-CH2-C0-0 +2CHI3 + 61

~0-CO-CH2-CO-0~ + 2H+ ■* HO-CO-CH2-CO-OH

2 CH3-C0-CH2-C0-CH3 + 2 Na -* 2 CH3-CO-CH-CO-CH3+H2+2Na+

cL H2N-CO-NH2

H2N-CO-NH2 + 20H:"' 2NH3 + CO2"

H2N-CO-NH"2 + 2H+ + H20 2NH* + C02

d). /C52
0(j °° * hidrógeno "ácido"
HN NH

O
- 294 -

PROBLEMA N° 7 14° OIQ

El oxalato de calcio, CaC204.H20, es una sal ligeramente soluble de


importancia analítica y fisiológica. Su producto
-9 3 2
de solubilidad es 2.1 . 10 (mol/dm ) a 25°C. Los iones oxala to pueden
protolizarse para formar oxalato ácido y ácido oxáli -co. Los valores- de
pK a 25°C son 1.23 (HoC,0.J y 4.28 (HC,0~). A 25°C el producto iónico del agua
es 1.0 , lo" (mol/dm3)2.

a) Describa aquellas expresiones para las condiciones de equilibrio las cuales


son de interés para el cálculo de la solubili_ dad del oxalato -monohidrato
de calcio.

b) Establezca las condiciones de concentración que son necesarias


3
para el calculo de la solubilidad "s" (en mol/dm ) del oxalato de calcio
en ácido fuerte de concentración C.

3
cj. Calcular la solubilidad (en g/dm ) del oxalato monohidrato de calcio en
la célula de una planta que tiene un sistema buffer que regula el pH a 6.5.

3). Calcular la solubilidad (en g/dm3) del oxalato monohidrato de calcio en ácido
clorhídrico con una concentración de O.OlOraol/dni" Dé la concentración de
los iones hidrógeno de la solu -ción.

e). Calcular las concentraciones en el equilibrio de todas las e£ peeres de


la solución d).

-------------------------- S O L U C I O N ------------------------

a) {Ca2- ). tC202") = KG ÍH+ ) . {OH-} = (2)

(H) . {HC,0T} {H ) o ÍC,0.2~)


---------- — = Kal ------------------------- -±— = Ka2 (4>

tH2C204) {HC204]t

b) s = ÍCa2+} = ÍC202-} + £HC204 } + {H2C2041 (5) C = ÍH+} +

{HC204} + 2 . ÍH2C204} - {OH-} (.6)


- 295 -

Ecuación (5) 6 (6) puede reemplazarse por:

ÍH+} + 2 . {Ca2+} = ÍHC20¡} + 2 . ÍC202"} + COH_}+ C (7)

c) La solubilidad del oxalato de calcio monohidrato es 6.7 . 10 3 g/dm3 (calculada


por ecuación (8)).

d) Eliminando el término de la concentración de las especies oxa lato, usando


las ecuaciones (1), (3) y (4) se obtienen las si guientes expresiones para
(5) y (6) (la concentración de los iones de hidrólisis pueden ser
despreciadas).

2 ÍH+}.K ÍH+}2-K
= KG + ---------2. + ---------- §. (8)
Ka2 Kal-Ka2

Í H +}.K 2 ÍKV'K
C = H + ------- 2. + ----------- i (9)
S.KA2 8.KAL.KA2

La eliminación de "s" de (8) y (9) resulta en una ecuación de cuarto


orden. Por esta razón se prefiere un método ite
- 296 -

rativo. La primera aproximación es (H }= C. Este valor de IH J puede


ser usado para calcular:

i) s de (.8)

ii)_ Los dos últimos términos en (9) son correcciones.

Ahora, el nuevo valor de lH+}obtenido de (9) puede ser usado como


valor de partida para la siguiente aproxima ción o

Dos operaciones repetidas dan el siguiente valor para

s = 6.6 „ 10~4mol/dm3 = 9.6 . 10 ~2 g/dm3 {H+} = 9.3

. 10~3mol/dm3

ÍC2oJ J = 3.2 . 10 mol/dm' -4


(Ca2+) = 6.6 . 10~4mol/dm3 [Cl") =

(HC20~} =5.7 . 10 mol/dm .


0.010 mol/dm3 {OH-} = 1.1 .

ÍH2C20.}= 9.0 10~5 mol/dm


10~12mol/dm3
- 297 -

PROBLEMA N° 8 (práctico) 14°

Una solución buffer de pH tiene una acidez Bien especi_ ficada,


la cual cambia levemente por la adición de cantidades-moderadas de ácido o
Base fuerte. A mayor cantidad de base o de á cido que debe agregarse a un
cierto volumen de buffer para cambiar su pH, se dice que mejor es su capacidad
buffer o tampón . Una solución buffer se prepara mezclando soluciones de un
ácido débil y de su base conjugada en cantidades apropiadas. Un ejem pío de
un sistema buffer útil es el sistema fosfato.

Su tarea consiste en preparar una solución Buffer de fosfato con


las siguientes condiciones específicas.

(1). pH. = 7.20 en la solución Buffer

C2) pH = 6.80 en una mezcla de 50.0 cm3 de la solución buffer


3 3 y 5.0 cm de ácido clorhídrico 0.100 mol/dm .

Equipos y reactivos.

Solución acuosa de ácido fosfórico, solución de hidróxido de sodio


de concentración conocida, ácido clorhídrico (0.100 mol/dm3), solución de
verde de bromocresol, agua destilada.
3 3 3
Buretas, pipetas (25 cm y 5 cm ), Erlenmeyer (100 cm
3 3 y 250 cm ), aforados (100 cm ), vasos de precipitados y embudo.

Procedimiento.

Determine la concentración de la solución de ácido fos_ fórico por


valoración con una solución de hidróxido de sodio u-tilizando verde de
bromo-cresol como indicador (rango de pH,3,8-pH 5.4)

OIQ

Prepare la solución buffer mezclando los. volúmenes cal-


culados de ácido fosfórico e hidróxido de sodio en un aforado llenándolo
hasta la marca con agua destilada.
3 3 Mezcle 50.0 cm de la solución buffer
con 5.Q cm de á-
cido clorhídrico en un Erlenmeyer.

Entregue su hoja de respuestas al arbitro quién también


medirá el pH de sus dos soluciones y anotará sus resultados.
- 298 -

Los valores de pK para el ácido fosfórico son pK ,=1.75,


a ai
pKa2 = 6,73 y pKa3 = 1:L-50°

------------------------- S O L U C I O N ---------------------------

La solución buffer debe contener H_PO. (concentración a


3 2- 3
mol/dm ) y HPO^ (concentración b mol/dm ). Las condiciones de -

ben satisfacer la condición

b/a = 10-6-73/10-7'20

Después de la adición de HC1, la condición será:

(50.0 . b - 0.50)/(.50 .0 . a + 0,50) = 10~6 * 73/10-6'80

De las ecuaciones:

a = 0=0122 b = 0.0361

Concentración total del sis-tema fosfato = 0.0483 mol/dm Concentración

total de Na+ = (a + 2.b) mol/dm3 = 0.0844 mol/dm3

Si la concentración de las soluciones de ácido fosfórico e hi -dróxido


de sodio es 0.500 mol/dm , entonces se necesitan 100.0 cm3 de solución
buffer.
- 299 -

volumen de la solución de H,PO. =


3 4

0.0483 o 0.1000/0.500 dm volumen de la

solución de NaOH = 0.0844 . 0.1000/0.500

dm
PROBLEMA N° 9 (práctico) 14° OIQ

Cada -uno de los 8 tubos de ensayo numerados contiene la solución


de -ana sal„ En las soluciones se pueden encontrar los siguientes cationes
Cun -máximo de uno en cada tubo) :

Ag+, Al3+, Cu2+, Na+ y Zn2+ y los siguientes aniones (a

lo mas uno en cada tubo):

Br", Cl", i", NO~, OH" y S202"

Se provee de: una placa test, una gradilla con tubos de ensayo,
pipetas gotario, papel indicador y un mechero a gas.

Determine mediante reacciones mutuas cuál es la sal di_ suelta en


cada tubo de ensayo. Confirme sus conclusiones efec tuando tantas reacciones
como sea posible. Puede ser necesario usar combinaciones de soluciones como
reactivos.

Dé una lista de números correspondientes a las fórmu -las de las


sustancias. Indique la formación de precipitados por una flecha apuntando
(.+ ) hacia abajo y hacia arriba ( + ) cuando haya desprendimiento gaseoso
en el cuadrado correspondiente de la matriz dada para las reacciones.

Escriba las ecuaciones químicas para todas las reaccio nes


efectuadas.-
-------------------------- S O L U C I O N ---------------------------

Número de soluciones Ecuaciones químicas para las reacciones


mezcladas. observadas.

1 + 2 NH^+OH--* NH3(g) + H20

2 + 3 20H~+ 2Ag+ ■* Ag2OfsJ + H20


- 300 -

2 + 3 + 1 Ag20(s) + 4NH* + 20H~ ■* 2Ag(NH3)+2 +3H"20

2 + 4 Zn2++- 20H~-- Zn(OH)2(s) = Zn(OH) (s)+20H~+

Zn(OH)2~

2 + 5 Al3++ 30H"+ Al(OH)3(s)= Al(OH)3(s)-OH~-- Al(OH)~

2+ 3+
Reacciones para distinguir Zn de Al

2 + 4 + 1 Zn(OH)2~+ 4NH* + Zn(NH3)2+ + 4H20

2 + 5 + 1 AlCOH)~ + NH* + Al(OH)3Cs) + NH3 + H20

2 + 6 Cu2+ + 20H" -> Cu (OH) 2 (s)

2 + 6 + 1 Cu COH) 2 (s) + 4NH* + 2OH."* Cu(NH3)2+ + 4^0

3 + 4 Ag+ + Cl" * AgCl(s)

3 + 6 Ag+ + Br~ ■* AgBr(s)

3 + 7 Ag+ + I~ + Agí(s)

2Ag++S202~->- Ag2S203(s)= Ag2S203(s)+3S202~ +

2Ag(S203)2_
Reacciones para distinguir Cl~ de Br"

3 + 4 + 1 + 2 AgCICs) + 2NH* + 20H~->- Ag(NH3l2 + Cl~+ H20

3 + 4 + 8 AgCl(s). + 2S202 * Ag(S203)2++ Cl"

3 + 6 + 1 + 2
AgBr(s) + 2S203~ + Ag(S203I2~ + Br"

AgBr(s) no se

disuelve 3 + 6 + 8

3 + 7 + 8 Agl(s) no se disuelve
- 301 -

6 + 7 2Cu2+ + 4l" + 2CuI(s) + I.

6 + 7 + 8 I2 + 2S202- - 21- + S402-

1 2 3 4 5 6 7 8

1 +

-2 + + 4- +

3 + + 4- + 4-

4 +

6 +

7 +

8 +
302

Lista de números y fórmulas correspondientes de las sustancias.

1. NH.NO,

4 3

2. NaOH

3. AgN03

4. ZnCl2

5. A1(N03).3

6. CuBr2

7. Nal

8. Na2S203
- 303 -

PROBLEMA N° 10 (práctico) 14° OIQ

DETERMINACION DEL PRODUCTO DE SOLUBILIDAD DEL CLORURO DE PLOMO (II) :

Agite cloruro de Pb(II).: a) con agua y b) con tres soluciones de


cloruro de sodio de diferentes concentraciones hasta alcanzar el equilibrio.
Luego determine la concentración del ion plomo por titulación con EDTA.
Calcule el producto de solu Bilidad del cloruro de plomo (II).

Equipo y reactivos.

3 3 3
Aforado (J.0.0. cm ) , pipetas (20 era y 10 cm ) , probetas
graduadas (100 cm3 y 25 cm3), 4 Erlenmeyer (200 - 250 cm3) con
tapas, espátula, 4 embudo filtrantes, papel filtro, termómetro,
3 3 4 Erlenmeyer (100 cm í, vasos de precipitados
(200 - 250 cm ) ,
soporte con bureta, frasco lavador con agua destilada, varilla
de agitación de vidrio.

Soluciones estándares de cloruro de sodio (0.1000 mol/


3 3 dm ) y de EDTA (0.010Q0 mol/dm ), cloruro de plomo sólido, solu
ción de anaranjado de Jxilenol en una botella gotario (0.5% en
agua!, hexamina sólida (urotropina), ácido nítrico C2.5 mol/dm3)
en una botella gotario.

Procedimiento.

3
yer con tapa. Ponga 1QQ era de agua en un cuarto Erlenme -yer con tapa.
Agregue con la espátula 5 porciones de cloru
ro de plomo sólido (alrededor de 2 g) a cada Erlenmeyer, tápelos y agite
vigorosamente. Déjelos en reposo por 30 minutos. Agítelos ocasionalmente.
Prepare la filtración y valo ración mientras tanto.

2. Mida las temperaturas de las soluciones y anótelas en la ta -bla de


resultados. Filtre las soluciones a través de fil tros secos en
Erlenmeyer pequeños y secos.
304

3. Usando una pipeta, transfiera 10.00 cm3 del filtrado al vaso


3
para valorar. Diluya aproximadamente con 25 cm de agua, a-gregue 3 gotas
de anaranjado de xilenol (indicador) y 5 go -tas de ácido nítrico. Luego
agregue con la espátula 5 por -ciones (alrededor de 0.5 g) de hexamina
sólida (una base débil) revuelva suavemente hasta que la solución esté
clara» Valore con EDTA.

4. Calcule la concentración de los iones plomo y cloruro en la solución y


el producto de solubilidad Ks. Anote los resulta dos en la tabla.

5. Responda las preguntas en la hoja de respuesta.

Preguntas;

a). JDé la estructura del EDTA. Marque aquellos átomos los cua -les pueden
coordenar un ion metálico con *.

B) Dé la ecuación para la reacción de titulación. EDTA puede 2-


eseribirse H.X
- 305 -

S O L U C I O N

Un resultado típico

C(NaCl) Temp. Vol. EDTA ÍPb2+] (Cl")

mol/dm °C mol/dm" mol/dm"

0.0600 18.721 -4
Q.0187 Q.Q974
(mol/dm3
)3
J..77 .
10

0.0400 21 22.7 0.0227 0.0854 1.66

0.0.20.0 21 27.8 Q.Q278 0.Q.756 1.59 . 10

-4
21 34.2 0.0342 0.0684 1.60 .
10

Respuestas a las preguntas:

a) HOOC - CH2 ,CH2-COOH


N * * /
.H-CH2-CH2-H
HOOC-CH, CH2-COOH

b) H2Y2" + Pb2+ -»■ PbY2


+ 2H+
OLIMPIADA INTERNACIONAL DE QUIMICA

* * *

TIMISOARA 1983

RUMANIA

A) Describa la descomposición térmica de las siguientes sales a-mónicas en


términos de ecuaciones químicas.

t°c

a) NH4C104 »

b) (NH4)2S04 tOC >

c) (.NH4)2S208

d) NH4N02 t0C >

C) Señale la respuesta correcta

a) ¿Puede determinarse la masa molar midiendo la densidad de un compuesto


gaseoso, a nina presión y temperatura dada?

1) Sí, bajo ciertas condiciones.

2). Sí, si el compuesto gaseoso no se disocia y asocia.


- 307

PROBLEMA N° 1 15° OIQ

3) Sí, si el compuesto gaseoso no disocia.

4). Sí, si el compuesto gaseoso no asocia.

b) ¿Es un líquido que hierve a temperatura constante (a una presión dada)


una sustancia pura?

11 Sí, si el líquido no es azeotrópico 2) Sí,

si el líquido es azeotrópico

D) Complete y balancee la siguiente ecuación (en agua)

K2Cr207 + SnCl2 + ..... —► CrCl3 +.......... KC1 + .......


E) La solubilidad de Hg2Cl2 en agua es 3.0 . J.0 g/lÜO mi solución.

a) ¿Cuál es el producto de solubilidad?

b). ¿Cuál es la solubilidad de esta sustancia en O.OJL M NaCl en


mol/1?

c) ¿Cuál es el volumen de una solución 0.01 M NaCl la cual di-


suelve la misma cantidad de cloruro mercuroso que se disuel^
ve en un litro de agua pura?

Ajjg = 200.61 Acl = 35.45

Fl ¿Cuál de los siguientes grupos de compuestos son sólidos a 10°C?

a) H20, NH3, CH4

b) F2, Cl2, Br2

c). S03, I2, NaCl

d) Si, Sg, Hg

G) ¿Cuál de las siguientes sales forman Tina solución acuosa áci -da?

a)l CH3C00Na

b) . NHjCl
- 308

PROBLEMA N° 1 15° OIQ

c) Na2HP04

d) Na2C03

e). NaHC03

H) Escriba las fórmulas electrónicas de los siguientes compuestos tal que la


naturaleza de los enlaces químicos sea evidente.

al NaC103 b) HC103 cl SiF4 d) NH3 e) CaF2


f) H20
- 309 -

I) El ácido perclórico sólido se escribe usualmente como HC104 H20.


Basado en los datos experimentales los cuatro enlaces mostrados son
idénticos; sugiera una estructura que tome en cuenta los resultados
experimentales.

J) Los compuestos de la segunda fila de elementos con hidrógeno se formulan


como sigue;

LiH, BeH2, B.2Hg, CH4, NH-j, H20, HF.

a) ¿Cuál de los compuestos son sólidos a temperatura ambiente? Explique.

b) ¿Cuáles son iónicos?

c) ¿Cuáles son poliméricos?

d) ¿Cuál de ellos no reacciona con agua bajo condiciones normales?

e) ¿Qué reacción ocurre entre:

BeH2 + H20 --- f

B2H6 + H20 —-»

B2Hg + LiH -- *

f) Suponiendo que NH^, H20 y HF son ácidos bajo algunas condi ciones,
escriba sus bases conjugadas correspondientes y or dénelos en forma
creciente de su fuerza básica.

2
K) Dados: JlnO. + 8 IT+ 5 e = Mn

f)
o
E 1.52 V
+ 1
t'
MnOT + 4 H+ + 3 e~ = MnO_ + 2 H.O o
2

Calcule E° para la reacción:

1.69 V

o
3
t' ->
------------------------- S O L U C I O N ---------

A)a) 4NH4C104 t0C » 4 HC1 + 6 HjO + 2 N2 + 5 02

b)„ 3CNH4)"2S04 fc°C > S02 + N2 + 4 NH^ + 6 H20


- 310 -

C) 2(NH4)2S20g t0C > 4 S02 + 2 N2 + 8 H20

tOC
d) NH4N02 - N2 + 2 H20

C) a) 1) 2) 3Í

4} b ) 1) 2)

D). K2Cr207 + 3 SnCl2 + 14 HC1 -------- *■ 2 CrCl3 + 3 SnCl4 +


+ 2 KC1 + 7 H20

-4
E) a) S = 3oOCL . 10-5 g/10Q mi = 3 . 10-4 g/1 = 3- 10

472
-7 3 = 6.3 . 10 mol/dm-

Hg2Cl2 t Hg2+ + 2 Cl"

Kg = 4 S3 = 4(6.3 . 10"7)3 = 1 . ÍO"18 mol3 . dm"3

b) Ccl- = 0.01 mol/dm3

10 18
S = —— = 1' = i . io"14 mol/dm3
C^- (0.01) ¿

c) El volumen de una solución 0.01 M NaCl disuelve la misma


cantidad de Hg2Cl2 como 1 dm3 de agua pura es:
-14 -7 3
1. 10 ±H V = 6.3 . 10 . dm
V = 6.3 . 107 dm3
F) c) S03, I2,
NaCl
!Fl

H) a) Cl c) IF -
Na Si - F|
IQ I I
Oj 101 IFJ

b) Cl d) H: - N - H
r7 I \ I
0 0-H N
- 311 -

e) {[T~ Ca2+ IF )

f) 10

Id
I) (H30)+ + (^C104]" 6 (H30) |5 - c l - g|
10]

hidronium perclorato

J) a) LiH, (BeH2)n polímero b) LiH

c) (BeH_) ¿ d). CH,


n

e) BeH2 + 2 H20 Be (OH). 2 + 2 H2

B2Hg + 6 H20 ------ *■ 2 B(OH)3 + 6 H2

B2Hg + 2 LiH 2 Li

f) NH2 >■ OH > F


- 312 -

MnO~ + 4H+ + 3e~ = Mn02 + 2 H20 E° = 1.69

Mn02 + 4H+ + 2e" = Mn2+ + 2 H20 E° = x

MnO~ + 8H+ + 5e" = Mn2+ + 4 H20 E° = 1.52

5E° = 3E° +

2E° 7.60 = 5.0 7 + 2

x x = 1.26 V
- 313

PROBLEMA N° 2 15° OIQ

En una mezcla gaseosa de CO y CO.,, una razón de masa de carbono:


oxígeno = 1:2, fue determinada.

a) Calcular la composición en porciento de masa.

b) Calcular la composición en porciento de volumen.

c) Indique el valor de la razón carbono: oxígeno para la cual ambos


gases no pueden estar presentes simultáneamente „

S O L U C I O N

Escriba x = número de moles de CO


en lOOg
y = número de moles de CO^
28x + 44y = 100

12(x -y) 1 x = 1.389 moles de CO

16(x + 2y) 2 y = 1.389 moles de CO.

1 3R9 44

a) JOO • 100 = 61.11 % C02


100 = 38.89 % CO
100
1. 389 2 8

b) x .=. y 50 % C02 + 50 % CO (en volumen).

c) Los dos gases no pueden estar simultáneamente presen tes si:

Masa carbono _ 12 corresponde a co puro

Masa oxígeno 16

12
•JJ corresponde a C02 puro

A) Una muestra que contiene una mezcla de cloruro de sodio y cío ruro de potasio
pesa 25 g. Después de su solución en agua se agregan 840 mi de 0.5 mol/dm
- 314 -

PROBLEMA N° 3 15° OIQ

AgNO^. El precipitado se fil -tra y luego una lámina de cobre que pesa
100.00 g se sumerge en el filtrato. Después de un intervalo de tiempo,
la lámina pesa 101.52 g. Calcular la composición en porciento de masa de
la mezcla.

S O L U C I O N

Al Acu = 6 3 . 5 AAg = 108

Escriba : Cu + 2 AgNO^ ------ ► CuC.NO^l 2 + 2 Ag

y *

x = la cantidad de plata depositada y =¡ la

cantidad disuelta de cobre

63.5 2 . 108

x - y = 101..52 - 100 x = 1.52 + y

63.5 2 . 108 . c- - 1C , +
----- = Í—E-SÍ—;— y = 0.63 x = 2.15 g Ag
y J..52 + y 1 ^

Masa del nitrato de plata

840
1000
0.5 . 170 = 7.1.4 g AgNO,
- 315 -

PROBLEMA N° 4 -15° OIQ

170 q AgNO, 71.4


----------- i = -------- x = 45.36 a Aq
10 8 g Aq X

Plata consumida por precipitación

45.36 - 2.15 = 43.21 g Ag+

Masa total del cloruro

10 8 a Ag+ 4 3,2
x = 14.2 g Cl
35.5 g Cl

MNaCl = 58'5 MKC1 - 74°6

Escriba x = masa del NaCl en la mezcla y = masa del

KC1 en la mezcla

Masa del Cl en NaCl : 3-¿5gX


08.5

Masa del Cl" en KC1 : 3^56y

35.5 x 35.5 y _ ?
58.5 + 74.6 " ± ' ¿

x + y =25

x- = 3.7,6. g NaCl 7a. 4 % NaCl y =

7.4 g KC1 29.6 % KC1


- 316 -

FISICO QUIMICA

Los siguientes flatos fueron reunidos para la hidróli -sis alcalina


de ciertos compuestos clorados:

a) Un cierto volumen de una solución de la sal neutra de pota -sio del ácido
clorosuccínico se mezcla con un volumen igual de una solución de hidróxido
de potasio. La concentración í nicial de cada solución es 0.2 mol/dm3. La
concentración del hidróxido de potasio en la mezcla de reacción se
determinó a intervalos de tiempo diferentes a 25°C. Se obtuvieron los si
guientes datos;

t (minutos) 10 20 30 45 60 80 100

CJJ^J(mol/dm )_ 0.085 0.074 0.065 0.056 0.049 0.042 0.036

El experimento fue repetido con las mismas soluciones iniciales a


35°C. La concentración de hidróxido se redujo a la mitad después de 21

minutos.

b) En la hidrólisis de 3-cloro 3-metil hexano con hidróxido de potasio, la


concentración del hidróxido de potasio se redujo a la mitad después de 32
minutos a 25°C ó 11 minutos a 35°C, sin importar la concentración inicial de
los reactivos (idén tica).

cl En la-hidrólisis alcalina de 3-cloro-2,4-dimetil-3-isopropil-pentano, se


encontró un mecanismo idéntico al de b), pero la 'Velocidad de reacción
fue alrededor de 100 veces mas rápida bajo las mismas condiciones.
- 317 -

PROBLEMA N° 4 -15° OIQ

Con los datos anteriormente dados, responda las siguien tes


preguntas;

I » ¿Cuál es el orden de la reacción en los tres casos a, b y c?

2. ¿Cuál es la velocidad de reacción a 25°C para la reacción a? Indique las


unidades.

3. Calcule la energía de activación para la reacción a y b

4. Si en la reacción a, se usa. la sal dipotásica del L-ácido cío ro succínico


- el cual es levorotatorio qué tipo de rota ción Óptica exhibirá la
sal correspondiente del ácido mélico formado por hidrólisis?

5. Si también se usa el isómero levorotatorio en la reacción b, ¿qué rotación


óptica será inhibida por el 3-metil-3hexanol formado en la reacción de
hidrólisis?

6. ¿Porqué la velocidad de reacción c es mas rápida que la reac ción b, cuando


ambas reacciones son del mismo tipo y ocurren bajo las mismas condiciones
de presión y temperatura?

S O L U C I O N

_1. Para la reacción a, el orden de la reacción se estima como si


gue:

1.505

- presumiendo una reacción de primer orden:


30 J..436
45 JU288
60 1.189
80 1.0 84
íaa 1.Q22

k no es constante, por ello la reacción no es de primer orden - para Tana


reacción de segundo Orden (en concentraciones de los reactivos iguales-
en el tiempo cero):

k - 4. i-jl. - tea k

10 0.176
20 0.176
- 318 -

30 0.179
45 0.175
60 0.173
80 0.173
100 0.178
Como k tiene un valor constante, se cumplen las para una condiciones
reacción de segundo orden.

La vida media de la reacción b es independiente tración de la


inicial, i.e. es una reacción de primer concen-orden:

a
k = -L- in = —i - 4n = -i- Jl-n 2
t . a - x t . ,, a t,
1/2 a- y 1/2

La reacción c tiene el mismo mecanismo de la reacción b; por lo tanto será


una reacción de primer orden

2. La-velocidad de la reacción a es el promedio de los valores calculados


anteriormente.

k = 0.176 dm3 . -mol X . -rain X

3. Con el fin de determinar la energía de activación, calcule la constante


de la velocidad de reacción, k', a 35°C.

Para reacciones de segundo orden la reacción entre la constan te de


velocidad de reacción y la vida media es: k _ _A_ ( _i L_) = 1
(
L_ _ = _1_ _A_
t a-x a t1/2 a - a a t^' a

La vida media a 35 °C y la concentración inicial, a = 0,1 mol/dm3


(por mezcla de volúmenes iguales de las dos soluciones,la concentración
de cada especie reactante se reduce a la mitad) son conocidas. Calcular
la velocidad a 35°C.

1 1

21 - 0 . 1 - 0.476 dm3 .
mol-1 . min"1
La energía de activación de la reacción a será:
a k

E = E 4 n Jil . 8314 in 0 " 476 30 8


- T - T

« 295

" 0.176 * 303 - 298


- 319 -

Ey = 7.592 . 107 J.kmol"1

Para la reacción b, que es de primer orden la constante de la velocidad de


reacción a las dos temperaturas se calcula de las vidas medias:

k = I"-2- : A. 25°C : k = ^J- = 2.166 . 10~2 min"1 tl/2 i¿

-2 1
A 35°C : k'= = 6.301 . 10 min"

De aguí, la energía de activación es:

6 301
= 8314 * n ' • 10 '1 . 108
• 298 = 8.149 . 107 J.kmol"1
lD)
2.166 . 10 308 - 298

El producto de la reacción de hidrólisis a será dextrorotato-rio como


res-altado de la configuración de inversión.
- 320 -

e
CH2COO ;H2COO CH2COO
i

i
,9 -
,

HO + ,C H COO Lo' HO COO

Cl --- ► HO...C. ....... Cl

'H

Como una reacción de tipo SN2, involucra un estado de transición en el cual


ocurre la inversión de la configuración del car Bono asimétrico. De aquí,
si el sustrato es levorotatorio,el producto será dextrorotatorio.

5. La reacción b es una reacción unimolecular SN^ e involucra la transformación


transiente de un ion carBonio casi estaBle en el paso determinante de la
velocidad de reacción.

La estructura más proBable del ion carbonio es planar. El ion carbonio


puede así ser atacado por un reactivo nucleofílico Cel ion OH ) en ambos
lados del plano con la -misma probabilidad. El producto resultante como
una mezcla racémica sin actividad óptica e inactivo por compensación .
intermolecular„

6. Lo mismo es verdadero para la reacción c, la única diferencia es una


repulsión mas marcada entre los sustituyentes mas gran des. La tendencia
hacia la formación del ion carbonio con u-na es-tructura planar y repulsión
reducida es aumentada.
- 321 -

PROBLEMA N° 5 15° OIQ

La velocidad de reacción en la etapa de formación del ion car bonio y la


de la reacción total son consecuentemente aumentadas .
- 322 -

Al pasar etanol sobre un catalizador a 40QK, ocurre una reacción


de deshidratación en la formación de etileno:
C2H5OH(g) ----- * C2H4(g) + H20(g)
A la temperatura arriba dada y P° = 101.32 5 kPa, la con

versión del alcohol etílico es de 90.6 mol %.

-1. Calcular la constante de equilibrio para la reacción bajo las-


condiciones dadas-.

2. Calcular el valor de las constantes de equilibrio K Y K a


x c

la temperatura dada.

3. Calcular la conversión del etanol a las siguientes presiones:

5. P°; 10. P°; 5a. P°; 100. P° y 200. P°.

4. Haga un gráfico para la -variación de la conversión vs. pre


sión.

S O L U C I O
N

La reacción C.HcOH

+
moles

iniciales 1 0 0

en el equilibrio 1-x x x Total = 1 + x

Fracción molar Presión parcial

etanol 1-x
1+x
1-x
1+x

etileno
1+x
1+x
- 323 -

PROBLEMA N° 5 15° OIQ

agua
1+x 1+x

V. .. presión total

P . . . 101.325 kPa

P P rC2H4. FH20 ( * .p)( x


K ._) . P
P " ■C2H5OH
1-x
l-x p
0
1+x

1. P'= 10J..325 kPa

0.90.6' 0 .906.
P
0.906
1-x2 1-0.9062
1-0.82Q8 0.18

= 4.56

2. K = Kp.P~An? P'= 101.325 kPa; y An = 1; = 4.56

c .OT" \An , , _3
Kc = Kp . (-2-^— I R = 8.314 J mol K fC°= 1 mol tu ;T=400K

K = 0.139 c
- 324 -

3.

x2 KP -1.56
1-x

aj 4,56
0.912

1-x 2

x 1-x2
b) = 0.456

2
x
-- 1
O 1-x" 0.0912

d) x 1-x2 0.0456

e) _¿_ = 0.0228
l-x¿

0.912 1.912 0.60


a) x =

= 0.559 - 0.56
'0.4 56
b) x =
1.456
b.0912 = 0.289 = 0.29 =
c) x = 1.0912

u .0456 0,208 - Q.21


d) x = 1.0456
0.0228
1.0228
= 0.149 - CU 15
e) x = x
10 50
Un mol de un compuesto A reacciona sucesivamente con 3 moles del
compuesto B en solución acuosa en presencia de un cata lizador básico (tal
como Ca(OH>2).

A + B -- ► C ; C + B —+ D; D + B ---- ► E;

La hidrogenación del compuesto E produce el compuesto

E + H2 --- >- F

1 -
- 325 -

PROBLEMA N° 6 15° OIQ

F tiene la composición C = 44.18%, H = 8.82%, 0 = 47% y masa


molar M = 136.

Sabiendo que 1.3.6 g de F reaccionan con 40.8 g de anhí drido acético


para formar el producto G y ácido acético solo, e jecute A, B , C , D, E, F ,
y G y escriba las ecuaciones químicas que ocurren.

S O L U C I O N

La fórmula molecular de F es:

11^=3.68; ^=8.82; 11=2.94;

C:H:0 = T3I: fifí' £|í- L25: 3 .

1 - 5 : 12

(C H
3 12°4)n

M p „- n = 5 . 12 + 4 . 16 + 12 . 1 = 136
C5H1204
- 326 -

136
136

C5H12°4

Como F reacciona con anhídrido acético, puede ser un mo no o un


polialcohol. Si fuera un alcohol monohidroxílico, 136 g de F podrían
reaccionar con 102 g de anhídrido acético. De he -cho 13.6/136 = Q'.l mol
de F reaccionan con 40.8/10.2 = 0.4 moles de anhídrido acético, i.e. F es un
poliol (tetrolí.

F se forma por reducción de E de modo que E tiene un car bonilo con


tres grupos OH.

E se forma de 3 moléculas de B y una molécula de A. El compuesto


E tiene tres grupos OH y un CO, sabiendo que las condi_ ciones de reacción
que se usan para la condensación aldólica está claro que A es un acetaldehído
y B es formaldehído. C y D son los productos de las sucesivas
aldocondensaciones de acetaldehído con formaldehído. C y D son los productos
de las sucesivas aldocondensaciones de acetaldehído con formaldehído„

H2C = O + H3C-CH = O ------ » NO-CH2-CH2-CH = O


B A C

HO-CH2-CH2-CH = 0 + H2C = 0 —► (HO-CH2). 2CH-CH. = O

C B D

(H0-CH2) 2CH-CH = 0 + H2C = O ---- r (HO-CH2).-jC-CH. =


O
D B E

CH0-CH2) 3C-CH = O + H2 - ► (H0-CH2). 4C

E F

(_HO-CH2).4C + 4 (.CH3CO)20 - r (CT^COO-CH^C + 4CH3COOH


- 327

PROBLEMA N° 7 -15° OIQ

Sabiendo que los compuestos A y B son isómeros de fórmu la molecular


C^H^NO y que el compuesto M tiene una masa molecular de 93, determine las
fórmulas de los compuestos de A y S y es criba las reacciones.

HN0,/H„S0
0 [o]
H0N0 U" HOH 3'V °4 , 0
-H20
-1. HN0,/H,S0. [[°]
^n D ---- 1 2 4
HOH
r-i
-H20
HOH i»
0
NaOBr

HN0,+HC1 HOH
® ------ 2 --- , g
s
-co„ HN02/HC
1
HOH [0] HOH
LH S
- 328 -

S O L U C I O N

[0]
C6H5-CH2-OH CHO

HOH HN( tu
C6H5-CH=N-OH C,HB-CH0 VV°¿ - [O
0 [o]'

-H20 COOH
C,Hr-C=N JEL c „..CO0H -¿y* 0
6 3
} m
-4/ 0
-H20 H2/cat.
HOH COOH
C6H5-CH2-NH2 C.H,-CO-NH„ b
ji z
0

NaCBr
NH2 NaN02 .
NaNO.+HCl +
C6H5-NH2 -------- - --- > C6H5-N=N Cl HC1

(D 0 COOH
HOH LU
C6H5-0H N=N+Cl"

-C0„

.CH=NOH i 01 !H0 COOH


H0H
H0H r -, C°l
V0H * ÍOl 0H - KL» ' -----------------
LU
- 329 -

PROBLEMA N9 8 (práctico) 15° OIQ

I, En los tubos de ensayo A , B, C y D hay cuatro derivados del


benceno que contienen uno o dos qrupos funcionales de tres tipos distintos.
Identificar los grupos funcionales de los com puestos A , B, C y
D utilizando los reactivos disponibles.

Justifique su elección escribiendo las reacciones de identifi_


caclón. Utilizando como reactivos los cuatro compuestos A ,
B, C y D sintetice cuatro colorantes orgánicos y escriba las
ecuaciones efectuadas-.

S O L U C I O N

I. Los cuatro compuestos son

NV \LJ7-°H' \U7"0H'
H2
N-\Uy-C00H
Las reacciones de identificación son:

a) Con H2SO

:3H+-<^)-C00"

H
.
4 H3N+" {^3/>~C0°H HS°4

b) Con

:OO "OONa
H
NaOH.:
- 330

H3N'-(( +
Nn-COO
~/(^)\-COO~ + NaOK
+ NaOK ---------
------ ► ►
H„N-(( H2N)-(Y^)VcOO
)-COO Na Na++ HOH

c) Con NaHC03 :

,-- r- COOH . - _ COONa

(Cy) —°H + NaHC03 --- ► (Cy)—°H + C02 + HOH

d); Con H2N"\( )rs03H

S
°3 "^0^~^H3 + NaN0-5
+ H2P°4 ---------------- * ~Q3S"{^^)-^=N + NaHSO .+2HO
H 4

OH

naranj-a

<Q -<g -<p


, -- ^-COOH , - v , -- ^-COOH
-03S-^)-S=» + )-^H H03S >-N=N \-OH

naranj a
H.SO,

naranja

II. a) Con g-naftol

<^^NH2 + H2S04 + NaN02 ------------- ► HS°"

(RVfeN HSO¡ + O)" \0)-N=N^> + NA2S°4

V)
+H0H
<P)
amarillo-naranj a
331 -

HCOC-(( )}-NH2 + H2S04 + NaN02 —* 00C-(^ J ^-N=N +

NaHS04+HOH. H0 HO

Los siguientes colorantes pueden obtenerse

<^feN HSO¡ + (g ^OH M_ (g )-N=N-<Q)-0H

rojo-anaranjado

y - v . - <-COOH ,-- . --------- _COOH

rojo-anaranjado

HOOC-(^^\-NEN HSO~ + (^)\0H -------------►HOOC-^^^)-N=N-^^^>-OH

rojo-anaranj ado

--- , , -- ^-COOH .- . , --5 -C00H


HOOC-/(^)\-feN HSO~ + (Cy)- 0H ----- " H00C"\C3/ N=S-/(^)\-OH
rojo anaranjado

PROBLEMA N° 9 (practico). 15° OIQ

Un aforado contiene una solución que es una mezcla de á


cido oxálico y oxalato de amonio.

Una de las Botellas- marcadas X, Y y Z contiene una solu ción de una


sustancia para calibración de carácter reductora y con una concentración de
0,10.0(1 mol.dm 3.

Se le solicita:
- 332 -

------------------------ S O L U C I O N ------------------------

a) Determinar la cantidad de ácido oxálico y de oxalato de amo -nio que contiene


el aforado, (el resultado será dado en gra -mos) .

b) Escriba la fórmula de la sustancia reductora y la ecuación pa ra la reacción


química que permite su determinación.

Para hacer los análisis se dispone de las siguientes soluciones ;

HCl 0.100Q H r NaOH-2M, KMnO^ -0.02 M, H2S04~25%

HN03-2 M, BaCl2 5%, AgNO^ 5%, H^CNO^ 5 % , fenolf taleína 0.1% y rojo de metilo
1%. c) Describa el procedimiento usado en las etapas individuales,lo
indicadores empleados y los resultados parciales.

= 90.04

M = 124.11
(NH4)2C204

n(HnO.)
1 5
n'(reductor
desconocido!
- 333 -

A^ - Identificación de la solución con la sustancia reductora

X, Y, Z : Fe(NH4)2(S04)2

A 2 - Reacciones de identificación para los iones de la sustancia

- Fe2+ + 2NaOH ------ ■* Fe(OH)2 + 2 Na+

- NH* + NaOH ----- ► NH3 + + H20 + Na+

- 4 NH3 + 2 Hg2(N03)2 + HjO --------- >■ 0 HN2.N03 + 2Hg+3NH40H

2-
- S04 + BaCl2 --- ► BaS04 + 2C1

B. - Preparación de la solución de 0.1 M NaOH


3 3
......................... o . . . . o cm en 20.0 cm

B2 - Concentración de la solución de NaOH


indicador usado . .. „ ...... «...........

C - Concentración del KMnO


4

- Masa del ácido oxálico en la solución inicial .................. g


indicador usado

D2 - Masa del oxalato de amonio en la solución inicial

1-2 cm3 de las soluciones X, Y y 2 se ponen en tres tubos de ensayo.


Se agrega 6 N H2SC>4 y una gota de KMnQ4„ La solución que pierde color
es la que tiene carácter reductor.

Determinación de la fórmula;
2+
..... + NaOH - precipitado blanco-verdoso -------------->-Fe

........... + NaOH., en la parte superior del tubo, poner


papel filtro con Tina gota de Hg2 (NO-j) 2 -------- ► mancha negra:
NH*
4
2-
. . . . . . . o . o o + BaCl2 - precipitado blanco: SO^
- 334 -

------------------------------ S O L U C I O N ------------------------

........... + AgN03 + HN03 ------- ► - Cl~ ausente

La sustancia usada es Fe tNH^)2(SO^)2

Las reacciones químicas:

Fe2+ + 2(Na+ + 0H~). ----- »- Fe(OH>2 + 2Na+

Nff+ + (Na+ + OH") ----- >• NH3+ + H20 + Na+

4NH3 + 2Hg(N03)2 ^ / \ + +
\ 2 3 3 4 3
XH

SO2" + (Ba2+ + 2C1 ) ----- 1- BaS04 + 2Cl"

3 3
5 cm solución 2 M ----- ¡- 100 cm solución 0.1 M.

V cm3 0.10.0Q N HC1 + 0.1 N NaOH en presencia de fenolfta-


leína.
V cm sorución -X + J.0.Q cm H S04 + H2° se valora a 311:3 te!B peratura
2

con KMn04
Se determina la concentración del KMnO..
4

La solución que -va a ser analizada se llena hasta la marca; 3


V cm- de esta solución se valora con NaOH en presencia de rojo ele
metilo. Se calcula la cantidad de ácido oxálico £mo les y g) .

3 3
V cm de la solución a ser analizada + 10.Q cm H.,S04 6N +H20

se calientan y valoran con Mn04.

Se calcula (moles) la cantidad total de oxalato.

La diferencia dá la cantidad de oxalato de amonio (moles y g) .


- 335 -

PR03LEMA N° 10 (práctico) 15° OIQ

Seis tubos de ensayo tienen soluciones acuosas de FeSO,,


4'
H2S04, Mn(N03)2, H202, Pb(N03)2, NaOH.

aF Identifique el contenido de cada tubo de ensayo, sin usar o-tros reactivos.


Escriba los resultados en forma tabular. Es criba las ecuaciones de las
reacciones químicas usadas para las identificaciones.

b)" Después de la identificación ejecute cuatro reacciones cada vez utilizando


tres de los compuestos identificados y escriba las ecuaciones.

S O L U C I O N E S ----

2 3 4

H S0
FeS04 2 4 Mn(N03)2 H^ Pb(N03)2 NaOH

1)
FeSO. Fe(OH)S04 +PbS04
+Fe(CH)2
amarillent blanco |
o blanco-verdoso
+Fe(OH)
3
Café-ro
jizo

2)
H2S04 +PbS04
blanc
o
Mn(N03I2 -Mn(CH)
2 blanco
4-
4MnMn03
café-negr
o

3)
- 336 -

1 2 3 4 5 6
Fe(OH)SO.
amarillento
5)Pb(N0 )
3 '2 +PbS04 +PbS04 Pb(0H)2 blanco blanco
blanco +
Pb(OH)4"
6>NaOH Fe(OH)2 Mn(.0H)2 +Pb(0H)2
blanco blanco
blanco- t i
y +
verdoso + +MnMn03 PbCOHl2" café-negro
+ Fe(OH)
café-ro-

j izo

Observaciones
cambio de ,
---- >- 2Fe(OH)S04 color (Fe )
(l) + (4) 2FeS04 + H202 amarillo.
aparición de

un p.p. blan-

(l)+(5) FeS04 + Pb(NO 3)2 ---- »• +PbS04 + Fe(N03)2 co (+PbS04)


(l) + (6) FeSO.+2NaOH ■+• + Fe(.0H) +Na2S04 Aparición de un precipi
4 tado blanco verdoso

2Fe(OH),+1/20 2+H20 -»■+ 2Fe (OH) 3 (Fe(0H)2 el cual por oxi

z dación con aire se tor-ma café


rojizo +Fe(0H)3

(2)+ (5) H.SO. + Pb(NO ) -* +PbS0. + 2HN0, Aparición de un precipi


2 4 J 2 4 J tado blanco (+PbS04).

(3)+ (6) Mn (NO,) _+2NaOH-H-Mn (OH) ,+2NaN0, Aparición de un


precipita-
J z do blanco +Mn(0H) , el cual
2Mn (OH) ,+l/20?*+MnMnO +2H20 por oxidación con aire se
convierte en un precipita-
Mn (OH) -+1/20 ■* +MnO-+H20 do café-negro +MnMn03 el
¿ cual con el tiempo cambia
- 337 -

Observaciones

a -t-MnO- un precipitado negro


-café.

(5) + (6) Pb(N03) 2+ 2Na0H-> Pb(CH)-o2NaN03 Aparición de un precipitado


blanco +Pb(OH)2 el cual se
disuelve en exceso de reactivo,
Pb(OH)2+ 2Na0H ■* Na2Pb(OH)_4

b)

(l) + (2) + (4) 2FeS04+H2S04+H202 ■* Fe (S04) 3+2H20 —► Cambio de color3+


amarillento (Fe )

(l) + (4) + (6) 2FeS0,+H_,0,+4NaOH -H2Fe(OH) +2Na_,S0. Aparición de un


precipita
4 z ¿ J 4
do (+Fe(OH)3)
café-rojizo.

(3)+ (4)+ (6) Mn(N0,)_+2NaOH+H,0, -M-MNO,. 2H-0+2NaN0 Aparición de un


precipita

(5)+ (4)+ (6) Pb(NOj. +H00_+2NaOH -»4PbO-+2H-C+2NaNO, Aparición de un


precipi-
5 ¿ ¿ ¿ ¿ A J
tado café (+Pb02).
6 ¿ ¿ ¿ ¿ 1 J d o café (+Mn0 ) .
2

APENDICE

Cantidades y sus Unidades usadas en la Colección.

Nota : Los nombres y símbolos para las cantidades y sus unidades están dados
en el siguiente orden:
cantidad, símbolo de la cantidad, nombre de la -unidad,símbolo para
la -unidad.

i/ Unidad base SI

longitud 1 metro m
masa m kilogramo kg
tiempo t segundo s
corriente eléctrica I ampere A
temperatura T kelvis K
cantidad de sustancia n mol mol

ii/ Nombres y símbolos especiales para ciertas Unidades deriva-das SI.


- 338 -

fuerza 7 newton N /kg.m.s 2/


presión P pascal Pa /Noirf2/
energía E joule J /N.m/
potencia P watt w /J.s"1/
carga eléctrica Q coulomb c /A.s/
diferencia de
potencial
eléctrico u volt y /J.A^.s"1
resistencia
eléctrica R ohm Q /V.A"1/
iii/ Otras Unidades Derivadas SI usadas en química.

2
área S metro cuadrado m
3
volumen y metro cúbico m
3
también: dm , (.1)

Nota: La palabra litro se considera ahora como


un nombre especial para el decímetro
cúbico. Ni la palabra litro ni su sím
bolo 1 debieran ser "usados pa ra
expresar resultados de alta precisión.

densidad p kilogramo por ^


metro cúbico k.g.ra
también: kg.dm^3;
g.cm
concentración c mol por metro _^
cúbico mol.m
muy a menudo mol.dm
; mol„l

masa molar M kilogramo por -1


mol kg.mol
muy a menudo m
s
g.mol
-1

iv/ Algunas otras cantidades y constantes y sus símbolos.

masa atómica relativa A


de un elemento

masa molecular relativa M


de un compuesto

fracción molar x

fracción de masa w

fracción de volumen y

entalpia H

entropía S

energía de Gibbs G
- 339 -

carga elemental, e. 1.6021892 , lO"19 C 10~34 J.s

constante de Planck, h , 6.626176 . 104 C.mol-1


8.31441
constante de Faraday, F , 9.648456 .
J.mol~1.K~1
constante de los gases, R,
273.15 K /exacto/ 1.01325 .
cero de la escala Celsius T .

presión normal, P , 105 Pa /exacto/

volumen molar estándar de los 2.241383 -2 3 -1 10 m


.mol
gases ideales, V ,
6 .022045 1023 mol-1

constante de Avogadro , v/ Otras unidades empleadas aunque ellas no


son Unidades Si pero son importantes y ampliamente usadas:

tiempo: minuto min hora h grados 60 s

Celsius °C 3600 s

temperatura:

vi/ Algunos prefijos SI

Fracción -1 deci
Prefijo
lO"9 kilo
nano
10 Símbolo

10 micro
n V m

10 mili
c d k
-3 centi
-2

10

10~

You might also like